Skin Lesions 01-22 Flashcards

1
Q

A 79-year-old farmer presents with a lesion on the back of his neck. Examination shows the lesion is flat and pink and rough to the touch. Which of the following is the estimated rate of malignant transformation for an actinic keratosis?

A) 1%
B) 10%
C) 25%
D) 50%
E) 75%

A

The correct response is Option B.

While the true rate of malignant transformation of actinic keratosis remains unknown, best estimates report the progression rate to squamous cell or basal cell carcinoma is 10%. As a result, treatment for these lesions is recommended through a variety of modalities. The most commonly used method of treatment for patients with solitary or few lesions is cryotherapy. Other options include curettage and topical therapies. For multiple lesions of field cancerization of an anatomical area, topical fluorouracil (5-FU), imiquimod, or photodynamic therapy with photosensitizers are more often used.

How well did you know this?
1
Not at all
2
3
4
5
Perfectly
2
Q

A 52-year-old man presents with a 1-cm invasive cutaneous squamous cell carcinoma of the penis. Which of the following is the most appropriate treatment?

A) 5-Fluorouracil therapy
B) Mohs micrographic surgery
C) Penis transplantation
D) Radiation therapy
E) Total amputation

A

The correct response is Option B.

Mohs micrographic surgery involves the removal of cutaneous malignancies with concomitant circumferential histologic examination. This technique helps preserve normal tissue and has low rates of recurrence. It is most appropriately used for aggressive lesions, lesions at high risk for recurrence, and in areas where maximal preservation of normal tissue is critical for form or function, such as the “mask” areas of the face or other sensitive areas. Appropriate use criteria for Mohs micrographic surgery were developed by an ad hoc committee representing several dermatologic professional societies. This set of recommendations categorizes basal cell carcinoma, squamous cell carcinoma, lentigo maligna, and melanoma in situ as appropriate for Mohs micrographic surgery, of uncertain benefit, or as inappropriate, based on lesion characteristics and location. Squamous cell carcinoma on the genitalia, whether aggressive or nonaggressive, falls in the appropriate for Mohs micrographic surgery category. Furthermore, studies have shown excellent results with Mohs micrographic surgery of the penis, with low recurrence rates. Sexual and urinary function is better preserved by Mohs micrographic surgery than with traditional partial or total amputation.

5-Fluorouracil is a topical chemotherapeutic agent which may be used to treat actinic keratosis, superficial basal cell carcinoma, or squamous cell carcinoma in situ, but it is not appropriate in this patient. Radiation therapy may be useful for treatment of nonmelanoma skin cancer in patients for whom surgery is a poor option. However, when compared with Mohs micrographic surgery in this patient, radiation would likely be less effective and lead to increased morbidity. Total amputation of the penis may have a high cure rate but carries with it devastating morbidity. It is not an appropriate first-line treatment in this patient. Penis transplantation remains an experimental procedure and currently requires life-long systemic immunosuppression. It is not the correct answer.

How well did you know this?
1
Not at all
2
3
4
5
Perfectly
3
Q

A 75-year-old man presents with an abnormal skin lesion. Biopsy confirms Merkel cell carcinoma. Which of the following best describes the characteristics of this skin cancer?

A) The behavior is aggressive with high rates of lymph node spread and local recurrence
B) The plantar surface of the foot is the most common location for this lesion
C) This disease is more common amongst patients of African descent
D) This lesion frequently arises within the setting of a chronic wound
E) Topical chemotherapeutic agents are the primary method for managing this condition

A

The correct response is Option A.

Merkel cell carcinoma (MCC) is an uncommon skin cancer which has aggressive behavior. MCC usually presents as a painless red nodule that is rapidly growing. Elderly patients are more commonly affected by MCC, and it is believed that sun exposure is a major risk factor. There are several studies that suggest involvement of the polyomavirus in the pathogenesis of MCC. Patients with immunosuppression are also at relatively higher risk for MCC. This is an aggressive skin cancer with early and frequent involvement of the regional lymph nodes. Patients with large tumors or regional spread are at high risk for distant metastasis. This disease is significantly more common in lighter-skinned patients than in dark-skinned patients. The head and neck region is the most common site for MCC. A Marjolin’s ulcer is a rare and aggressive type of squamous cell carcinoma arising in the setting of a chronic wound. Once a diagnosis has been established, treatment of MCC typically involves wide local excision and lymph node mapping. Many of the treatment strategies for MCC are similar to melanoma given the similar aggressive clinical behavior.

How well did you know this?
1
Not at all
2
3
4
5
Perfectly
4
Q

A 63-year-old woman with a history of ulcerative colitis presents after being diagnosed with invasive ductal carcinoma of the right breast. Her BMI is 23 kg/m2. She undergoes unilateral mastectomy with immediate reconstruction with a free deep inferior epigastric perforator flap. Her initial postoperative course is uneventful. Five weeks after the procedure, the patient returns to the clinic. Examination shows erythema and skin ulcers surrounded by violaceous discoloration around the abdominal and breast incisions. No underlying fluid collections are appreciated. Punch biopsies of the ulcers are performed and show neutrophilic dermatosis. Which of the following is the most appropriate next step in management?

A) Broad-spectrum antibiotic therapy
B) CT scan of the chest/abdomen/pelvis
C) High-dose steroid therapy
D) Surgical exploration with washout and debridement
E) Ultrasound of the breast and abdomen

A

The correct response is Option C.

Pyoderma gangrenosum (PG) is a rare skin disorder of unknown cause that is believed to be part of the spectrum of neutrophilic dermatoses. Specific characteristics of postoperative PG include dramatic deterioration of surgical wounds after a period of 4 days to 6 weeks, with predominant involvement of the breast and abdomen. Numerous reports of PG following reduction mammaplasty and breast reconstruction have been published. Additionally, patients with a history of inflammatory bowel disease are at an increased risk for developing PG.

Clinically, postoperative PG is characterized by fever, chills, cellulitis, and wound breakdown. The hallmark finding is a rapidly enlarging area of central skin ulceration surrounded by violaceous skin with irregular borders. Systemic signs of inflammation along with laboratory abnormalities (leukocytosis, hyponatremia, and hypoproteinemia) can accompany these findings. The diagnostic challenge of postoperative PG is related to its clinical resemblance to necrotizing infection and, hence, a high index of suspicion is required. However, the presence of neutrophilic dermatosis supports the diagnosis of PG. Treatment consists of immunosuppression and frequently consists of a combination of high-dose steroids and topical tacrolimus.

Additional imaging would not aid in management in this case. Broadening antibiotic coverage or surgical debridement would be indicated for treatment of infection. Local wound care does not address the underlying cause of the clinical condition and, hence, would not be the most appropriate next step.

How well did you know this?
1
Not at all
2
3
4
5
Perfectly
5
Q

A 43-year-old African American woman presents after undergoing Wise pattern reduction mammaplasty 6 months ago. Examination shows bilateral medial inframammary scars that are raised and pruritic but have not extended beyond the original incision sites. Intralesional injection of triamcinolone (TAC) is planned. Which of the following drugs, when administered in combination with TAC, will improve long-term efficacy?

A) 5-Fluorouracil
B) Diphenhydramine
C) Methotrexate
D) Tamoxifen
E) Verapamil

A

The correct response is Option A.

This patient has hypertrophic scars. Injection of 5-fluorouracil (5-FU) and TAC has been shown in randomized trials to be superior to TAC alone in the treatment of hypertrophic scars. Studies have used a regimen of 4 mg TAC and 45 mg 5?FU once a week for 8 weeks and showed reductions in Patient and Observer Scar Assessment Scales. In low doses, methotrexate (MTX) has anti-inflammatory effects mediated by adenosine A2 receptors. Oral MTX has shown promise in preventing recurrence of keloids after excision, but no definitive randomized data yet exist. Verapamil has been shown to be inferior to intralesional TAC in a randomized trial. Tamoxifen and diphenhydramine have mechanistic interest in the treatment of hypertrophic scars, but no data currently exist.

How well did you know this?
1
Not at all
2
3
4
5
Perfectly
6
Q

An 80-year-old man presents with a primary basal cell carcinoma of the left upper lip. Which of the following factors is associated with the highest risk for recurrence?

A) Desmoplastic subtype
B) Perineural involvement
C) Poor differentiation
D) Primary lesion
E) Well-circumscribed borders

A

The correct response is Option B.

Perineural involvement is the factor most associated with high risk for recurrence of basal cell carcinoma.

Poorly defined borders, a recurrent lesion, an immunosuppressed patient, the site of prior radiotherapy, perineural involvement, and an aggressive histologic subtype (morpheaform, basosquamous, sclerosing, mixed infiltrative, or micronodular) are the factors associated with a high risk for recurrence of basal cell carcinoma.

Poorly defined borders, recurrent disease, an immunosuppressed patient, the site of prior radiotherapy or chronic inflammation, a rapidly growing tumor, neurologic symptoms, perineural or vascular involvement, poor differentiation, or adenoid, adenosquamous, or desmoplastic subtypes are the factors associated with a high risk for recurrence of squamous cell carcinoma.

How well did you know this?
1
Not at all
2
3
4
5
Perfectly
7
Q

A 60-year-old woman presents with worsening painful ulceration of a prior healed surgical site of the left lower extremity after undergoing hidradenitis excisions 10 years ago. She reports no trauma and has had no fevers or chills. Examination shows some drainage at the wound site. She undergoes treatment with debridement, antibiotics, and multiple types of dressings without progress of wound healing. Photographs are shown. Which of the following is the most appropriate next step in management?

A) Debridement and biopsy
B) Debridement and dermal allografting
C) Debridement and negative pressure dressing
D) Excision and coverage with a flap
E) Excision and skin grafting

A

The correct response is Option A.

Marjolin ulcers are malignancies (most commonly aggressive squamous cell carcinoma) that arise from areas of chronic irritation, injury, or unstable scar. These malignancies frequently occur over many years. This malignant degeneration is most commonly associated with burns; however, it can be associated with a multitude of other types of nonhealing wounds such as traumatic wounds, osteomyelitis, pressure sores, any type of ulcer, laceration, venous stasis ulcer, or fistulas. The rate of incidence is approximately 2% in burn scars, with a predilection for the extremities, especially in the lower limbs. There is often a delay in diagnosis, since the appearance of a nonhealing ulcer with heaped-up edges mimics other types of ulcerations. Marjolin ulcer should be ruled out in a chronic nonhealing ulcer in the setting of a long-term scar/injury/burn.

Diagnosis of Marjolin ulcer is typically made by tissue biopsy with pathologic evaluation. Squamous cell carcinoma is the most common type of histology seen in Marjolin ulcer, although basal cell carcinoma, malignant fibrous histiocytoma, malignant melanoma, liposarcoma, fibrosarcoma, eccrine syringofibroadenoma, Merkel cell carcinoma, and keratoacanthoma have all been described. For patients with squamous cell carcinoma (most common type), the degree of differentiation is important prognostically. The risk for metastasis is highly correlated with the degree of differentiation and grade of the tumor. There is early lymph node metastasis in 30% of patients.

Treatment is surgical with wide local excision with pursuit of negative margins. Sentinel lymph node biopsy is frequently pursued. Deeper ulcers involving bone may warrant amputation of the involved extremity.

Marjolin ulcer must be diagnosed and treated (or the diagnosis excluded) before considering any other treatments.

How well did you know this?
1
Not at all
2
3
4
5
Perfectly
8
Q

A 55-year-old man presents with a fungating mass of the right shoulder that has been enlarging for the past 12 years. A photograph is shown. Medical history is unremarkable. The patient reports that he spent his youth working outside. He has a history of smoking. Which of the following is the most significant risk factor for development of this lesion in this patient?

A) CDK4 mutation
B) Previous use of topical steroids
C) Smoking history
D) Sun exposure

A

The correct response is Option D.

The patient presents with a nodular ulcerated basal cell carcinoma to the right shoulder. Basal cell carcinoma is the most common malignancy in the United States due to the increase in sun exposure and tanning salons. The most common risk factor for basal cell carcinomas is sun exposure. Generally, they do not metastasize and are resectable, but do lead to large oncologic resections if left to progress, as is the case in the patient described. Smoking history, although important, does not have a major impact on the risk for basal cell cancer. Marjolin ulcers are a variant of squamous cell carcinoma that results from the chronic inflammatory process that follows burns and is not a risk factor for basal cell carcinoma. Immunosuppression is a risk factor for basal cell carcinomas, but it is not as great as sun exposure and this patient has no history of immunosuppression. Other risk factors for basal cell carcinoma include being male and of older age, but these are not listed as possible choices. A mutation in the CDK4 gene is linked to familial melanoma and would have no relation to the basal cell tumor in this patient. Topical steroid use has no known impact on risk for development of basal cell carcinoma.

How well did you know this?
1
Not at all
2
3
4
5
Perfectly
9
Q

Which of the following subtypes of basal cell carcinoma has the lowest risk for local recurrence?

A) Infiltrating
B) Micronodular
C) Morpheaform
D) Nodular
E) Sclerosing

A

The correct response is Option D.

Basal cell carcinoma (BCC) is the most common human malignancy as well as the most common malignant tumor of the skin. The incidence of BCC is increasing worldwide. The most significant etiological factors are believed to be ultraviolet light exposure and genetic predisposition. Therefore, an aging population and prolonged exposure to sunlight may explain the worldwide increasing incidence.

The great majority of BCCs are successfully treated and will not recur. However, it is important to know the high risk lesions and subtypes to understand when the recurrence rate may be higher. The morpheaform, sclerosing, infiltrating, micronodular, and metatypical subtypes are associated with higher risk for relapse. Anatomic locations on the trunk and limbs are considered low-risk areas, while the forehead, cheek, chin, scalp, and neck are intermediate-risk areas. The nose and perioral areas are high-risk areas. Size greater than 1 cm for head and neck tumors and greater than 2 cm in other body areas also predisposes to a higher recurrence risk.

How well did you know this?
1
Not at all
2
3
4
5
Perfectly
10
Q

A 36-year-old woman, gravida 1, para 1, with a history of cesarean delivery is evaluated for a painless, firm, 6-cm wide subcutaneous mass that is fixed to the anterior abdominal wall with no associated symptoms. The mass was noted after pregnancy. Tumor markers are within normal limits. She has a family history of familial adenomatous polyposis (FAP) syndrome. Which of the following tumors is the most likely diagnosis?

A) Dermatofibrosarcoma protuberans
B) Desmoid
C) Lipoma
D) Lymphoma
E) Neurofibroma

A

The correct response is Option B.

Abdominal wall tumors are rare, accounting for less than 10% of all soft-tissue tumors. Desmoid tumors and soft-tissue sarcomas account for 45% and 40%, respectively, of all abdominal wall tumors. Desmoid tumors, also known as aggressive fibromatosis, are characterized by unpredictable progression or spontaneous regression, but lack the ability to metastasize. The majority of desmoid tumors arise from sporadic mutations in CTNNB1, whereas 10% arise in association with an APC mutation in familial adenomatous polyposis (FAP) syndrome. Abdominal wall desmoid tumors demonstrate an increased prevalence in women of childbearing age.

Dermatofibrosarcoma protuberans (DFSP) is an uncommon, locally aggressive abdominal wall tumor with low metastatic potential. DFSP originates from cutaneous tissues and is limited to superficial structures. The majority of abdominal wall DFSP occur in adults aged 20 to 50 years with similar sex distribution, and tumors are small (<5 cm) with characteristic purple or blue discoloration.

Neurofibromas appear as soft, skin-colored papules or small, subcutaneous nodules. The majority of neurofibromas are localized and arise from sporadic neurofibromin 1 gene (NF1). While the plexiform type is pathognomonic for hereditary neurofibromatoses caused by germline mutations in the neurofibromin 1 gene (NF1) or neurofibromin 2 gene (NF2).

Lipomas are common benign tumors composed of mature adipose cells. Lipomas usually develop on the trunk or proximal limbs as discrete rubbery masses in the subcutaneous tissues that present at any age. Among solitary cutaneous lipomas, 60% display clonal alterations, which are not associated with presentation of multiple lipomas. There is an increased prevalence of solitary lipomas in women, and multiple lipomas occur more frequently in men.

Primary abdominal wall lymphoma is a rare extra-nodal presentation with increased prevalence among male patients. Extra-nodal disease may present at all ages, but more that 75% of patients are over age 50 years. Patients with atypical lymphoma may not present with fever, night sweat, weight loss or anemia. Lactate dehydrogenase tumor marker is observed in extra-nodal lymphoma. Most soft-tissue lymphomas are of B-cell origin.

How well did you know this?
1
Not at all
2
3
4
5
Perfectly
11
Q

A 50-year-old woman presents with a 1.5-cm white plaque on the left cheek that has been slowly growing for the past 3 years. A photograph is shown. Examination of the specimen obtained on biopsy shows perineural invasion and basaloid epitheloid cells displaying ductal lumina. Which of the following is the most likely diagnosis?

A) Keratoacanthoma
B) Malignant melanoma
C) Microcystic adnexal carcinoma
D) Squamous cell carcinoma

A

The correct response is Option C.

Microcystic adnexal carcinoma (MAC) is a slow-growing skin cancer that primarily affects the head and neck and usually presents as a fleshy, plaque-like lesion. MAC is commonly misdiagnosed as basal or squamous cell carcinomas amongst others. MAC commonly has perineural invasion (PNI) and, as its names implies, affects apocrine structures. Although PNI can occur with basal cell carcinoma, squamous cell carcinoma, and melanoma, the latter two are generally much faster growing and this finding is still much less common. For basal cell carcinoma, PNI is typically observed in patients with advanced disease. Keratoacanthoma is a form of well-differentiated squamous cell carcinoma and is generally fast growing. Treatment of MAC is typically carried out with complete circumferential, peripheral, and deep margin assessment, and most often with Mohs micrographic surgery.

How well did you know this?
1
Not at all
2
3
4
5
Perfectly
12
Q

A 67-year-old retired landscaper with Fitzpatrick type I skin has a 5-year history of numerous flat erythematous scaly skin lesions (>20) on his bald scalp. He reports several new skin lesions of similar appearance on routine follow-up. Examination of several specimens obtained on shave biopsy shows cellular atypia above the basement membrane zone with overlying parakeratosis. He has no prior history of skin malignancy. Which of the following is the most appropriate initial treatment?

A) Cryotherapy
B) Systemic pembrolizumab
C) Topical 5-fluorouracil
D) Wide local excision
E) Observation

A

The correct response is Option C.

Actinic keratosis is a pre-malignant skin lesion that occurs on sun-exposed skin. Risk factors for actinic keratosis include advanced age, male sex, cumulative sun exposure, and fair skin type. The risk of progression from actinic keratoses to invasive squamous cell carcinoma is 0.025 to 16% per year, and the calculated lifetime risk for progression across a 10-year follow-up is approximately 6 to 10%.

The goals of the treatment are to eliminate clinical and subclinical actinic keratoses, minimize their risk for malignant transformation to invasive squamous cell carcinoma, and obtain acceptable cosmetic outcomes.

Topical field-directed therapy is the most appropriate initial next step in treatment of multiple, widespread actinic keratoses within an area of chronic sun damage. Topical medications include 5-fluorouracil (5-FU), imiquimod, diclofenac sodium, and ingenol mebutate. 5-Fluorouracil is an FDA-approved topical chemotherapeutic pyrimidine analog to treat multiple undetectable and clinically detectable actinic keratosis, and it is associated with a clearance rate of 84%. The mechanism of action is interference of DNA synthesis by inhibiting thymidylate synthetase. The one to two times per day application for 2 to 4 weeks is well tolerated with local skin reactions: pain, pruritus, burning, erythema, erosion, and inflammation.

Despite ongoing observation with routine skin examinations, the patient presented with additional actinic skin lesions. While observation may offer a nontreatment option, actinic skin lesions may undergo malignant change over time. Therefore, topical field-directed therapy is the best initial treatment option for widespread actinic keratoses.

Cryotherapy is a lesion-directed treatment option for a few individual actinic keratoses. It is a destructive modality often using liquid nitrogen (?195°C or ?319°F) that is better tolerated when less than 15 lesions are present. Clearance rates range from 39 to 99.8% with the use of cryotherapy. Common adverse effects include mild discomfort, scarring, and dyschromia.

Wide local excision is a directed therapy for individual malignant lesions, and it is therefore not appropriate in this patient due to the extensive nature of this benign condition.

Pembrolizumab is a systemic immune checkpoint inhibitor of PD-1 that treats metastatic or unresectable cutaneous melanoma.

How well did you know this?
1
Not at all
2
3
4
5
Perfectly
13
Q

A 35-year-old African American woman presents with multiple draining sinus tracts and nodular abscesses in the bilateral inframammary folds, groins, and axillae. She has failed topical and oral antibiotic therapies. Medical history includes type 2 diabetes mellitus, obesity, and keloid scarring. Which of the following is the most appropriate initial treatment for this patient?

A) Adalimumab therapy
B) Incision and drainage
C) Radiotherapy
D) Skin-tissue-saving excision with electrosurgical peeling (STEEP) procedure
E) Wide excision and skin grafting

A

The correct response is Option A.

This patient has hidradenitis suppurativa characterized by multiple nodules, abscesses, tunnels, and scars most commonly in the axillae, inframammary folds, groin, perigenital, and perineal region. Significant advances in medical therapy have decreased the need for surgical intervention. The disease is classified according to the Hurley classification: stage I as transient nonscarring inflammatory lesions; stage II as separate lesions consisting of recurrent abscesses with tunnel formation and scarring, and single or multiple lesions separated by normal-looking skin; and stage III as coalescent lesions with tunnel formation, scarring and inflammation. This patient is stage II. Mild disease is often treated with topical antibiotics (e.g., clindamycin) and/or resorcinol, while moderate disease may benefit from oral antibiotics (e.g., tetracycline 500 mg twice daily). Refractory disease may benefit from antibody therapy and/or surgical intervention.

Adalimumab (Humira) is a recombinant human igG1 anti-TNF monoclonal antibody that binds the proinflammatory cytokine TNF-alpha. It was approved by the FDA in 2016 for the treatment of moderate to severe disease where patients have required long-term antibiotics or rapid flares upon cessation of antibiotics. Alternative agents include infliximab (Remicade), anakinra (Kineret), ustekinumab (Stelara), dapsone, or acitretin (Soriatane).

Radiotherapy is incorrect. There are no randomized trials comparing radiotherapy with medical or surgical therapy, but it can be effective and well-tolerated for focal areas such as the scalp. The risk for secondary cancer is minimal but not negligible; therefore, radiotherapy is not considered a front-line option.

Surgical incision and drainage is indicated for fluctuant abscesses but not effective for inflamed nodules since there is no collection of fluid to drain.

Skin-tissue-saving excision with electrosurgical peeling (STEEP) or deroofing removes unhealthy tissue with step-wise tangential excisions and preserving normal tissue to heal via secondary intention. There is significant postoperative morbidity and risk for scarring with higher recurrence rates. Given this patient’s history for keloid scarring, this is not the best option.

Wide excision is associated with lower recurrence rates but higher morbidity (e.g., infection, bleeding, contractures). This can be effective in patients with areas of limited disease but should be reserved for patients with severe disease refractory to nonsurgical therapies when large total body surface area is involved. Nonsurgical options should be fully explored in this patient given the keloid history.

How well did you know this?
1
Not at all
2
3
4
5
Perfectly
14
Q

A 70-year-old man presents with a 2-cm keratoacanthoma involving the left ear. Medical history includes melanoma of the left cheek. Which of the following is an indication for Mohs micrographic surgery over conventional excision in this patient?

A) Anatomic location of lesion
B) Diagnosis of keratoacanthoma
C) History of previous melanoma
D) Male biological sex
E) Patient age

A

The correct response is Option A.

Mohs micrographic surgical technique has demonstrated cure rates of 97% for primary squamous cell carcinomas and up to 95% for recurrent squamous carcinomas. In this particular patient, the strongest indication for utilization of the Mohs technique is the involvement of the patient’s ear. Keratoacanthomas are epithelioid neoplasms of sun-exposed skin. Classically, they appear as a solitary papule that rapidly increases in size with a crateriform center. Because they can undergo spontaneous involution, some groups feel keratoacanthomas should be classified as a benign growth. However, there is strong evidence that keratoacanthomas are a distinct variant of squamous cell carcinoma. They have been reported to demonstrate invasion and aggressive behavior, including perineural invasion in up to 2.5 to 14% in the head and neck. Because squamous cell carcinoma involvement of the ear increases the risk for recurrence and metastasis, Mohs micrographic surgery is indicated in this patient. Patient age, patient sex, history of previous melanoma, and the diagnosis of keratoacanthoma are not indications for Mohs excision.

Other indications for Mohs technique include the following:

Recurrent basal cell/squamous cell carcinomas

Locations prone to recurrence—”H-zone” of the face: periorbital, periauricular, temple, upper lip, nose/nasolabial fold, and chin

Tumors involving critical structures such as the eyelid or lip

Functionally important areas such as the genitals, perianal location, hands, and feet

Tumors arising in sites of previous irradiation therapy

Large tumors, >2 cm

Lesions with ill-defined tumor margins

Histologic aggressive subtype (morpheaform, basosquamous, perineural, and invasive/poorly differentiated squamous cell carcinoma)

Tumors arising in immunosuppressed patients, such as transplant recipients or patients with genetic predisposition (basal cell nevus syndrome, xeroderma pigmentosum)

How well did you know this?
1
Not at all
2
3
4
5
Perfectly
15
Q

Which of the following structures contains apocrine sweat glands?

A) Areola
B) Glans penis
C) Labia minora
D) Palms of the hands
E) Vermilion border of the lips

A

The correct response is Option A.

Sweat glands are of three types: eccrine, apocrine, and apoeccrine. Apocrine glands are located in the areola, axilla, labia majora, scrotum, prepuce, periumbilical, and perianal areas. Their ducts deposit sweat into hair follicles. Apocrine glands start functioning at puberty and secrete a fluid that contains several compounds, some of which are broken down by bacteria to generate odor. Their secretions also contain pheromones. Apocrine glands are thought to be involved in the pathogenesis of hidradenitis suppurativa.

Eccrine sweat glands are distributed throughout the body, except the vermilion border of the lip, nail bed, external auditory canal, nipples, labia minora, glans penis, and clitoris. The palms and soles have the highest density of eccrine sweat glands in the body. Their ducts deposit sweat directly onto the skin surface. This sweat is a clear, colorless, and odorless fluid that contains water and electrolytes. Their main function is temperature homeostasis. They also participate in the barrier function of skin by secreting immunoglobulins. Eccrine glands also contain stem cells that play an important role in skin repair and wound epithelialization. Abnormally increased activity of these glands in response to emotional stimuli results in hyperhidrosis.

Apoeccrine glands share histologic features of both apocrine and eccrine glands and present in the axilla and perianal areas. They secrete watery sweat in response to psychological stress. High numbers of these glands are thought be an etiologic factor in patients with axillary hyperhidrosis.

How well did you know this?
1
Not at all
2
3
4
5
Perfectly
16
Q

A 32-year-old woman is referred by her dermatologist for evaluation and treatment of a painful skin lesion on her lateral cheek. Examination shows a 5-mm raised skin lesion with a bluish hue. Excisional biopsy with 1-mm margins is performed. Pathologic analysis shows a well-circumscribed tumor arising within the dermis with diffuse dense basophilic cellular proliferation, eosinophilic hyaline deposits, and a lymphocytic infiltrate. The pathologic diagnosis is spiradenoma. Which of the following is the most appropriate next step in management of this lesion?

A) Chemotherapy targeting lymphocytes
B) Radiation therapy
C) Re-excision with 1-cm margins
D) Sentinel lymph node biopsy
E) Reassurance and observation

A

The correct response is Option E.

The most appropriate next step in management of a completely excised spiradenoma is reassurance and observation. Spiradenomas are well-differentiated, benign, dermal neoplasms. Their origin is controversial, with some believing that they originate from sweat glands and others believing that they originate from hair follicles. Most spiradenomas are seen in patients between 15 to 35 years of age. They usually present as small, solitary, painful nodules that can grow to several centimeters, often with a bluish hue. Spiradenomas usually arise on the head, neck, and trunk. There are various morphological subtypes, and they can occur alongside cylindromas, trichoepitheliomas, and/or trichoblastomas. Patients with Brooke-Spiegler syndrome have multiple spiradenomas, cylindromas, and trichoepitheliomas.

How well did you know this?
1
Not at all
2
3
4
5
Perfectly
17
Q

A 55-year-old woman undergoes biopsy of a suspicious lesion on the dorsal hand. Which of the following is the most common malignant skin tumor of the hand?

A) Basal cell carcinoma
B) Keratoacanthoma
C) Melanoma
D) Merkel cell carcinoma
E) Squamous cell carcinoma

A

The correct response is Option E.

Malignant tumors are uncommon in the hand, and squamous cell cancer is by far the most common malignant tumor. Approximately 75% of malignant skin lesions on the hand are squamous cell cancer and appear as crusty, scaly raised lesions predominantly on the dorsal skin. They will occasionally ulcerate. Basal cell cancer is a common malignant skin tumor but only accounts for about 10% of hand skin cancers. Melanoma is unusual on the hand; it is responsible for about 3% of hand tumors. Merkel cell carcinoma of the hand is rarely found. Keratoacanthoma is a mimicker of squamous cell carcinoma with a much more rapid growth rate and spontaneous remission.

How well did you know this?
1
Not at all
2
3
4
5
Perfectly
18
Q

A 65-year-old man presents with the rapidly growing, painful dorsal hand lesion shown in the photographs. It is not fixed to the underlying structures and the tendon and bone are not involved. He is a kidney transplant recipient and has been on immunosuppression for the past 10 years. Eighteen months ago, he was treated for a previous squamous cell carcinoma on the contralateral hand. Which of the following are the most appropriate surgical margins and depth of excision for this lesion?

A) 1-cm peripheral margins, excision to the deep dermal level
B) 1-cm peripheral margins, excision to the deep subcutaneous level
C) 1-mm peripheral margins, excision to the deep dermal level
D) 1-mm peripheral margins, excision to the deep subcutaneous level

A

The correct response is Option B.

Squamous cell carcinoma that arises in immunocompromised patients tends to behave more aggressively than tumors in immunocompetent patients. Resection with at least 6-mm margins (some recommend up to 10-mm margins), extending into the subcutaneous tissue, is recommended. Cure rates with Mohs surgery and with frozen section margin control are similar to those for wide local excision.

One-mm surgical margins are too narrow for squamous cell carcinoma, and would lead to a high rate of reoperations for positive microscopic margins. Furthermore, the entire dermis must be completely excised, along with at least part of the subcutaneous tissue, as lesions like this are likely to penetrate to that depth.

How well did you know this?
1
Not at all
2
3
4
5
Perfectly
19
Q

A 7-year-old girl is evaluated because of a 1.5-cm, slow-growing, isolated, firm subcutaneous mass on the posterolateral neck that has been present for the past year. Examination of a specimen obtained on excisional biopsy results in a diagnosis of pilomatrixoma. The specimen is most likely to have a mutation of which of the following genes?

A) CTNNB1
B) GLUT1
C) NF1
D) p57

A

The correct response is Option A.

Pilomatrixoma (also known as pilomatricoma or calcifying epithelioma of Malherbe) is a benign, slow-growing skin tumor of the hair follicle. These tumors are most commonly found in children, although they have been increasingly found in patients of all ages. They tend to develop in the head and neck region but can also be found in the trunk and extremities, and they are usually not associated with any other isolated signs and symptoms. Pilomatrixomas can rarely become cancerous via transformation to the malignant pilomatrix carcinoma. Mutations in the CTNNB1 gene have been found in at least 75% of isolated pilomatrixomas. The CTNNB1 gene is needed to regulate cell growth and attachment, and mutation in this gene directly implicates beta-catenin/LEF dysregulation as the major cause of hair matrix cell tumorigenesis in this condition.

The GLUT1 gene mutation is associated with infantile hemangioma, while the NF1 and p57 gene mutations are associated with neurofibromatosis 1 and Beckwith-Wiedemann syndrome, respectively.

How well did you know this?
1
Not at all
2
3
4
5
Perfectly
20
Q

A 53-year-old man presents with the lower eyelid skin lesions shown in the photograph. The lesions have been slowly growing over the past 12 months. Which of the following treatment options is the most appropriate?

A) Excision of the mass and overlying skin
B) Excision with 1-mm tissue margins
C) Excision with 2-mm tissue margins
D) Excision with 4-mm tissue margins and sentinel lymph node biopsy
E) Mohs micrographic surgery

A

The correct response is Option A.

The patient presents with xanthelasmata, which are localized accumulation of lipid deposits on the eyelids. Multiple treatment modalities are available, including chemical peels, cryotherapy, and laser ablation. Traditionally, surgical excision has been used and yields excellent cosmetic outcomes. As the lesions are benign, there is no indication for Mohs micrographic surgery and no margins are required.

How well did you know this?
1
Not at all
2
3
4
5
Perfectly
21
Q

A 68-year-old woman presents with a 3-cm morpheaform basal cell carcinoma (BCCA) involving the left mid cheek. A photograph is shown. Which of the following is an indication for Mohs micrographic surgery over conventional excision?

A) Anatomic location of lesion
B) Diagnosis of BCCA
C) History of previous melanoma
D) Morpheaform subtype
E) Patient age

A

The correct response is Option D.

Mohs micrographic surgical technique has demonstrated cure rates of 99% for primary basal cell carcinomas (BCCA) and up to 95% for recurrent BCCAs. In this particular patient, the strongest indication for utilization of the Mohs technique is the more aggressive morpheaform subtype of BCCA. This patient underwent Mohs excision with adjacent tissue transfer reconstruction. Patient age, history of previous melanoma, and the diagnosis of BCCA (without aggressive features) are not indications for Mohs excision.

Other indications for Mohs technique include the following:

Recurrent basal cell/squamous cell carcinomas;

Locations prone to recurrence—“H-zone” of the face: periorbital, periauricular, temple, upper lip, nose/nasolabial fold, and chin;

Tumors involving critical structures such as the eyelid or lip;

Functionally important areas such as the genitals, perianal location, hands, and feet;

Tumors arising in sites of previous irradiation therapy;

Large tumors (greater than 2 cm);

Lesions with ill-defined tumor margins;

Histologic aggressive subtype (morpheaform, basosquamous, perineural, and invasive/poorly-differentiated squamous cell carcinoma);

Tumors arising in immunosuppressed patients, such as transplant recipients or patients with genetic predisposition (e.g. basal cell nevus syndrome, xeroderma pigmentosum).

How well did you know this?
1
Not at all
2
3
4
5
Perfectly
22
Q

A 6-month-old female infant is referred by the pediatrician for management of a skin lesion on the right parietal scalp that was noticed at birth. The lesion is a 2 × 1-cm yellow plaque that is devoid of hair and has grown in proportion with the child. She is otherwise healthy and is doing well. Which of the following is the most appropriate recommendation for the child’s parents?

A) Biopsy to rule out malignancy
B) CO2 laser therapy prior to puberty
C) Excision due to high risk of malignant transformation
D) MRI to evaluate for brain abnormalities
E) Continued observation because of anesthetic risk

A

The correct response is Option E.

These clinical features are typical of nevus sebaceous. They present as yellow-orange flat plaques, occurring most commonly on the scalp (60%) or face (30%). They are usually present at birth but may appear in the first few years of life. They are hamartomas, arising from the pilosebaceous units of the skin. They occur due to mutations in the RAS pathway. Maternal transmission of genetic material from the human papilloma virus to the fetus has been implicated as a causative factor.

Excision of nevus sebaceous is performed because of the cosmetic concerns and risk of secondary tumors. The most common neoplasia is trichoblastoma, which is a benign tumor, although more than 40 types of secondary tumors have been described. The most common malignant tumor is basal cell carcinoma. Initial studies reported the risk of malignant transformation to be 10%, however, more recent studies indicate that this number is 1%. The risk of malignant transformation increases with age; it is extremely rare in childhood and has not been reported in children younger than 5 years of age. The risk of malignant transformation is, thus, very small and in the absence of any morphologic change in the lesion, biopsy is not indicated.

Nevus sebaceous lesions undergo change in appearance during puberty and become thick and verrucous, presumably due to hormonal influence. Most practitioners thus recommend definitive treatment prior to puberty. Surface ablative therapies like electrodessication, curettage, dermabrasion, photodynamic and CO2 laser have been proposed to improve the appearance of these lesions. However, nevus cells can be left behind in the deeper layers, with the risk of developing secondary tumors and potentially making future detection of neoplastic change more difficult.

The definitive treatment of nevus sebaceous is full thickness skin excision. In December of 2016, the Food and Drug Administration (FDA) issued a warning that “repeated or lengthy use of general anesthetic and sedation drugs during surgeries or procedures in children younger than three years or in pregnant women during their third trimester may affect the development of children’s brains.” The FDA modified the warning in April of 2017, stating “consideration should be given to delaying potentially elective surgery in young children where medically appropriate.”

Most of the data that lead to these warnings came from animal studies that showed learning and behavioral problems after exposure to anesthetics that block N-methyl-D-aspartate (NMDA) and gamma-aminobutyric acid (GABA). Research in humans is not conclusive, with some studies indicating neurotoxicity with multiple exposures, but not with a single exposure. However, the duration of a “brief exposure” has not been well-defined. There are ongoing studies that will hopefully shed further light on the matter. In view of this, it may be prudent to delay elective procedures in children if this will be not detrimental to the child’s health or final outcome.

Numerous syndromes are associated with nevus sebaceous. These mostly involve the central nervous and ocular system, but can also involve other organs. There does not appear to be a correlation between size of skin lesions and risk of nervous system involvement, but large lesions and centrofacial location have been suggested as having higher risk. Small isolated nevus sebaceous lesions in the absence of any other systemic manifestations do not warrant central nervous system imaging or systemic work up. The vast majority of nevus sebaceous are isolated lesions.

How well did you know this?
1
Not at all
2
3
4
5
Perfectly
23
Q

A 32-year-old Caucasian woman presents with multiple (>50) brown lesions on her arms and lower legs. They appear to be in areas of sun exposure. On examination, many of these lesions are well circumscribed, even in color, and less than 5 mm in size. The patient has a family history of melanoma. There are too many lesions to excise. Which of the following findings in one of these lesions would prompt an excisional biopsy?

A) Asymmetry
B) Clearly demarcated borders
C) Dark coloration
D) Waxy surface

A

The correct response is Option A.

In this patient with multiple melanocytic nevi, lesions should be treated with excisional biopsy if there is a high suspicion for melanoma. As there are more than 50 lesions, clearly there are too many to excise. These lesions should be evaluated for asymmetry, border irregularity, variable color, diameter greater than 6 mm, and evolution. Any of these signs in a lesion should lead to an excisional biopsy with a suspicion of melanoma, especially given the patient’s family history.

Lesions with a waxy surface are seborrheic keratoses and commonly found in an elderly population in sun-exposed areas. Dark coloration does not lead to a suspicion of melanoma.

How well did you know this?
1
Not at all
2
3
4
5
Perfectly
24
Q

A 38-year-old African American man presents with multiple purulent tunneling lesions in bilateral axillae and his right groin. Which of the following surgical treatments will result in the lowest likelihood of recurrence?

A) Deroofing
B) Electrosurgical peeling
C) Incision and drainage
D) Skin-tissue-saving excision
E) Wide excision

A

The correct response is Option E.

Hidradenitis suppurativa (HS) is an inflammatory skin disease with a characteristic clinical presentation of recurrent or chronic painful or suppurating lesions in the apocrine gland-bearing regions. HS should be differentiated from infections such as furuncles, carbuncles, and abscesses (due to infectious agents and response to antibiotics), cutaneous Crohn disease (often concurrent with gastrointestinal Crohn, which has “knife-cut” ulcers and no comedones [whiteheads or blackheads]), and acne (distributed on the face and upper truncus, whereas HS predominantly affects intertriginous areas).

Surgery is required to definitively treat the tunnels and scars associated with chronic HS. Although surgery is commonly recommended, the literature supporting surgical treatment is anecdotal, composed mostly of large case series or retrospective study reports. A systematic review by Mehdizadeh et al. concluded that a lower recurrence rate was found in procedures with wide excision (overall, 13%; primary closure, 15%; using flaps, 8%; grafting, 6%) compared with local excision (22%) or deroofing (27%). These operations can be disfiguring, and despite the removal of significant amounts of tissue, do not necessarily protect against disease recurrence.

How well did you know this?
1
Not at all
2
3
4
5
Perfectly
25
Q

A 74-year-old woman presents with biopsy-proven Merkel cell carcinoma involving the left cheek and no palpable lymphadenopathy. The most appropriate treatment involves wide local excision with 2-cm margins and which of the following?

A) Adjuvant radiation and immunotherapy
B) Adjuvant radiation only
C) Cervical lymphadenectomy
D) Sentinel lymph node biopsy and adjuvant radiation
E) No additional intervention is needed

A

The correct response is Option D.

Merkel cell carcinoma is a rare and aggressive neuroendocrine cutaneous malignancy with a propensity for lymph node and distant metastasis and high mortality. It typically presents as an asymptomatic erythematous papule or nodule in the head and neck region. Surgical treatment includes wide local excision of the primary lesion with 1- to 2-cm margins to achieve histologically clear margins. Sentinel lymph node biopsy has become standard of care and helps determine prognosis and may guide additional adjuvant therapy. Negative sentinel node biopsy correlates with longer disease-free survival, especially in early stage disease. Adjuvant radiation has been shown in improve local-regional control and increase survival. Cervical lymphadenectomy would only be indicated in the setting of clinically positive lymph nodes and is insufficient as the sole adjuvant therapy.

How well did you know this?
1
Not at all
2
3
4
5
Perfectly
26
Q

A 48-year-old man presents with an asymptomatic, slowly growing violaceous nodule overlying his right scapula. Dermoscopy shows a delicate pigmented network, vessels, and structureless light brown areas. Biopsy identifies the lesion as a dermatofibrosarcoma protuberans. To minimize recurrence, surgical excision should include which of the following?

A) Bone
B) 6-cm margin
C) Deep fascia
D) Frozen sections
E) Periosteum

A

The correct response is Option C.

Dermatofibrosarcoma protuberans (DFSP) is a rare neoplasm of intermediate malignancy. Taylor first described it in 1890, but Darier is credited with establishing DFSP as a distinct clinicopathological entity in 1924, and finally, Hoffman established the term in 1925.

Most DFSPs occur on the trunk (42%), followed by the upper extremities (23%), lower extremities (18%), then the head and neck (16%). The reason for recurrence is that microscopic projections are not removed adequately or assessed satisfactorily; the wider the margin, the higher the probability the tumor will be removed completely. Using 1-cm margins around the primary tumor leaves residual microscopic tumor in more than 70% of patients, 2-cm margins in 20 to 40% of patients, 3-cm margins in 9 to 15.5% of patients, and 5-cm margins in 5% of patients. Peripheral margins of 5 cm have a close to 0% recurrence rate. Wide resections may also not be practical in patients with tumors located in critical areas like the head and neck, and intraoperative frozen section assessments have not been reliable for determining margin status. Deep margins should always include excision of the deep fascia. The complete resection of the tumor requires excision of the external outer table in the cranium; muscle in trunk and extremities; peritoneum in thin patients with DFSP located on the abdomen, sternum, and clavicles; and vertebral apophysis when located on the thorax.

How well did you know this?
1
Not at all
2
3
4
5
Perfectly
27
Q

Cutaneous squamous cell carcinoma arises from which of the following epidermal layers?

A) Stratum basale
B) Stratum corneum
C) Stratum granulosum
D) Stratum lucidum
E) Stratum spinosum

A

The correct response is Option A.

Squamous cell carcinoma arises from keratinocytes in the stratum basale. The epidermis comprises four layers, from deep to superficial: stratum basale, stratum spinosum, stratum granulosum, and stratum corneum. Glabrous skin has an additional layer between the stratum granulosum and stratum corneum called stratum lucidum. The stratum basale consists of basal cells. These are stem cells that differentiate into keratinocytes. These keratinocytes then migrate upwards. In the stratum spinosum the keratinocytes form intercellular connections via desmosomes. In the stratum granulosum, the keratinocytes have keratohyalin granules. In the stratum corneum, the cells are compact and surrounded by a lipid layer, thus giving the skin its barrier function. Melanocytes and Merkel cells are found in the stratum basale. Langerhans cells are found in the stratum spinosum, stratum granulosum, and the dermis.

How well did you know this?
1
Not at all
2
3
4
5
Perfectly
28
Q

A 43-year-old man presents with a recurrent 6-mm skin lesion on his upper lip. It is characterized by an erythematous papule surrounded by scale and crust. Biopsy shows nests of poorly differentiated cords of spindle cells of keratinocyte origin. Which of the following is the most appropriate treatment?

A) Cryosurgery
B) Electrodessication and curettage
C) Mohs micrographic surgery
D) Radiation therapy
E) Standard surgical excision

A

The correct response is Option C.

The patient’s histology is consistent with squamous cell carcinoma. Mohs micrographic surgery (MMS) has the highest cure rates for both primary and recurrent squamous cell carcinoma. For locally recurrent tumors, the 5-year cure rate for MMS is 90% compared with 76.7% for standard surgical excision. Because MMS is a tissue-sparing technique, smaller surgical margins are taken, and scarring and functional impairment are minimized compared with standard surgical excision and electrodessication and curettage. Tumor removal and reconstruction are usually performed on the same day, using local anesthesia in an office-based setting. Recurrence rates after cryosurgery and radiation therapy are not as favorable as for MMS.

How well did you know this?
1
Not at all
2
3
4
5
Perfectly
29
Q

A 12-year-old boy presents with warts on his fingers. His parents report that the warts have persisted despite application of a variety of topical treatments. Which of the following viruses is the most likely cause of this patient’s condition?

A) H1N1
B) Hepatitis C
C) Herpes simplex
D) Human immunodeficiency
E) Human papilloma

A

The correct response is Option E.

The cause of warts is the human papilloma virus (HPV). There are more than 100 known types of HPV. Types 2 and 4 are the most common cause of warts on the hands, as in this patient. People whose occupations expose their hands to wet environments, such as meat, poultry, and fish handlers and veterinary surgeons have higher risk of developing warts. The virus can be transmitted on shared clothing or public spaces, such as showers, and then gain entry through breaks in the skin. The virus is then able to alter the squamous epithelium to produce a cauliflower-like growth. Warts can be present in single lesions or multiple lesions. They are often painless and are usually not cancerous but can be a source of embarrassment for the patient. Warts are generally self-limited and will resolve spontaneously within months or years. Spontaneous resolution appears to occur in 50% of children within 1 year and 90% within 5 years.

There are many treatments for warts. Home remedies include topical salicylic acid, duct tape, and cold treatments. Clinical treatments have shown topical acids and cryotherapy to be equally effective. Intralesional injections and topical antivirals, as well as immunologic stimulators of interferon, have been tried with some success, including purified candida; measles, mumps, and rubella; and tuberculin (PPD) protein. Chemical ablation with silver nitrate has been shown to resolve almost half of warts a month after a 9-day treatment protocol. Mechanical removal can be performed with direct excision with good success. Pulsed dye and CO2 lasers have been successfully used to ablate warts but can be painful, expensive, and leave scars. Periungal warts can be particularly difficult to treat topically and may require a more invasive treatment method. A patient’s ability to eradicate a wart can be reduced by a compromised immune system. Multiple progressive warts in immunosuppressed patients may need to be biopsied as these warts may transform into squamous cell carcinoma.

Human immunodeficiency virus (HIV), H1N1, hepatitis C, and herpes simplex are all viruses that affect humans but do not cause lesions on the fingers. Herpes simplex virus is the cause of cold sores around the mouth and genital herpes. Hepatitis C is a virus that causes inflammation of the liver. HIV decreases the effectiveness of the immune system. H1N1 is an influenza virus and known cause of the “swine flu” outbreak in 2009.

How well did you know this?
1
Not at all
2
3
4
5
Perfectly
30
Q

A patient undergoes excision of a low-risk basal cell carcinoma lesion that is 1.5 cm in diameter. According to recommended guidelines, which of the following surgical margins is most appropriate to excise?

A) 1 mm
B) 2 mm
C) 4 mm
D) 6 mm
E) 8 mm

A

The correct response is Option C.

Basal cell carcinoma (BCC) is the most common cancer in the United States and it is estimated that BCC occurs in close to 2 million Americans annually. This exceeds the incidence of all other cancers combined. Fortunately, the mortality rate is low, but there is certainly significant morbidity because the incidence of this common malignancy is increasing rapidly. BCC is twice as common as squamous cell carcinoma (SCC), which is the second most common type of skin cancer.

The clinical margins chosen by the panel for low risk tumors are based on the work of Wolf and Zitelli. Many other authors and consensus panels have agreed with their recommendation. Higher risk lesions and clinical signs may make it appropriate to increase or decrease this margin control based upon anatomic and systemic considerations.

How well did you know this?
1
Not at all
2
3
4
5
Perfectly
31
Q

An otherwise healthy 30-year-old woman is diagnosed with dermatofibrosarcoma protuberans (DFSP) of the upper back. A wide excision is performed, and a local flap is used to reconstruct her back. On follow-up evaluation, CT scan shows multiple pulmonary metastases. Which of the following is the most appropriate next step in management?

A) Chemotherapy
B) Hormone therapy
C) Immunotherapy
D) Radiation therapy
E) Surgical excision

A

The correct response is Option A.

The most appropriate next step in management is chemotherapy. Patients with inoperable, recurrent, or metastatic disease may benefit from imatinib which is a tyrosine kinase inhibitor and acts as a molecularly targeted drug. It acts by inhibiting the platelet-derived growth factor receptor tyrosine kinase. Dermatofibrosarcoma protuberans (DFSP) is characterized by chromosomal rearrangements resulting in the production of platelet-derived growth factor B, eventually leading to autocrine growth stimulation of DFSP. Imatinib functions as an inhibitor of platelet-derived growth factor receptors, thus blocking this autocrine stimulation. Therefore, imatinib can be used as an adjuvant therapy for cases in which obtaining sufficient surgical margins is impossible. Neoadjuvant imatinib has also been used for locally advanced primary tumors.

Radiation therapy may improve local control and reduce the risk of recurrence postoperatively in patients with DFSP. There is no described role of it in pulmonary metastases. Resection of the multiple lesions in the lung, and hormonal or immune therapy are not recommended for metastatic DFSP.

How well did you know this?
1
Not at all
2
3
4
5
Perfectly
32
Q

An otherwise healthy 20-year-old woman develops a keloid on her right earlobe after an ear piercing. Excision and radiation therapy are planned. Which of the following is the ideal time after excision for the initiation of radiation therapy?

A) 1 day
B) 7 days
C) 2 weeks
D) 4 weeks
E) 6 weeks

A

The correct response is Option A.

The ideal time to give radiation therapy in this case is on postoperative day one. Earlobes and the helix of the auricle are common sites for keloid formation, usually after trauma or ear piercing, with an incidence of approximately 2.5%. There are numerous adjuvant therapies (eg, radiation therapy), medical therapies, (eg, intralesional steroids, 5-fluorouracil interferon, and topical silicone), and physical approaches (eg, pressure) that can be used in addition to excision of the keloid to help reduce its recurrence. However, the treatment options for such lesions are still controversial, because there are numerous challenges, and no single best treatment or best combination of treatments has been proved to manage these conditions effectively.

Radiation therapy has long been known to be effective in the early phase of wound healing. More specifically, radiation therapy is sensitive to endothelial vascular buds and decreases proliferation of new fibroblasts. As such, radiation therapy after keloid excision should be performed as soon as possible. Usually, better results are reported when radiation therapy is performed within 1 to 3 days after surgery. The most commonly administered doses are between 10 and 15 Gy over a period of 2 or 3 days.

The other options are incorrect as they are outside the ideal time frame.

How well did you know this?
1
Not at all
2
3
4
5
Perfectly
33
Q

A 65-year-old man comes to the office with a 2-cm basal cell carcinoma involving the left nasal ala extending across the nasofacial junction onto the cheek. A photograph is shown. Medical history includes previous melanoma. Which of the following is an indication for Mohs micrographic surgery over conventional excision in this patient?

A) Cancer size
B) Diagnosis of basal cell carcinoma
C) History of previous melanoma
D) Location of the lesion
E) Patient age

A

The correct response is Option D.

Mohs micrographic surgical technique has demonstrated cure rates of 99% for primary basal cell carcinomas and up to 95% for recurrent basal cell carcinomas. In this particular patient, the strongest indication for use of the Mohs technique is the anatomical location. The nose is considered a high-risk location in the classically described “H-zone.” This patient underwent Mohs excision with multi-stage forehead flap reconstruction, as shown in the photographs. Patient age, history of previous melanoma, and tumor size 2 cm or less are not standard indications for Mohs excision. Other indications for Mohs technique include the following: Recurrent basal cell/squamous cell carcinomas, Locations prone to recurrence- “H-zone” of the face: periorbital, periauricular, temple, upper lip, nose/nasolabial fold, and chin, Tumors involving critical structures such as the eyelid or lip, Functionally important areas such as the genitals, perianal location, hands, and feet, Tumors arising in sites of previous irradiation therapy, Large tumors (> 2 cm) Lesions with ill-defined tumor margins, Histologic aggressive subtype (morpheaform, basosquamous, perineural, and invasive/poorly differentiated squamous cell carcinoma) Tumors arising in immunosuppressed patients such as transplant recipients or patients with genetic predisposition (basal cell nevus syndrome, xeroderma pigmentosum)

How well did you know this?
1
Not at all
2
3
4
5
Perfectly
34
Q

A 76-year-old man comes to the office for follow-up after undergoing Mohs resection of a basal cell carcinoma (BCC) of the left central cheek. He smokes a half pack of cigarettes daily and has no intention of quitting. Final margins have been confirmed to be free of residual tumor. The defect is full-thickness skin and measures 2 × 2 cm. Which of the following is the most appropriate treatment option for this patient?

A) Cervicofacial flap
B) Direct primary closure
C) Full-thickness skin grafting
D) Healing by secondary intention
E) Rhomboid transposition flap

A

The correct response is Option B.

Direct primary closure is the best option for this patient. The cheek skin has ample laxity and allows for closure of small- to medium-sized cheek defects. In general, closure should be aligned with the relaxed skin tension lines, and aggressive management of dog ears should be performed. Direct linear closure also allows for simple monitoring for recurrence without any distortion of the anatomy. Secondary intention is better utilized in areas of concavity, like the medial canthus. Full-thickness skin grafting, rhomboid flaps and cervicofacial flaps will have a higher complication rate in an active smoker, while likely providing a lesser aesthetic outcome given the poor orientation of scars.

How well did you know this?
1
Not at all
2
3
4
5
Perfectly
35
Q

An 85-year-old man who takes an anticoagulant medication comes to the office for evaluation of a recurrent 1-cm nodular basal cell carcinoma at his nasal tip that has started to bleed intermittently. Medical history includes placement of a cardiac stent 1 month ago, after myocardial infarction. Which of the following is the most appropriate treatment for this patient?

A) Electrodessication
B) Excision with forehead flap
C) Pembrolizumab therapy
D) Superficial radiation therapy
E) Topical application of 5-fluorouracil

A

The correct response is Option D.

With a 5-year recurrence rate of about 3% for nodular basal cell carcinomas (BCC), superficial radiation therapy has become a viable alternative to Mohs micrographic surgery, which remains the gold standard for treatment of nonmelanomatous skin cancers.

Electrodessication has a high recurrence rate and would subject this anti-coagulated patient to the risk of bleeding after surgery or thrombosis should his anticoagulation be discontinued.

Excision with forehead flap would not be safe for a patient with a recent myocardial infarction.

Topical 5-fluorouracil is not indicated in the management of nodular BCC.

Pembrolizumab therapy is indicated for Stage IV melanoma.

How well did you know this?
1
Not at all
2
3
4
5
Perfectly
36
Q

A 72-year-old man comes to the office for evaluation of a 2-cm growth over the lower half of his nose that has been growing slowly during the past 15 years. A photograph is shown. Medical history includes acne rosacea. Physical examination shows a broad, exophytic mass. Which of the following is the most appropriate management?

A) Application of 5-fluorouracil
B) Direct excision and coverage with a forehead flap
C) Direct excision and coverage with a skin graft
D) Direct excision and healing by secondary intention
E) Observation only

A

The correct response is option D.

Rhinophyma occurs on the far end of progressive acne rosacea involving the sebaceous glandular overgrowth of the skin. It most commonly occurs on the nose but phymas can occur elsewhere on the face.

In early stages, rhinophyma can be treated with isoretinoin and antibiotics; however, in advanced states it needs to be resected. Ablation modalities include dermabrasion, carbon dioxide laser excision, scalpel excision, or a combination of techniques. Since the procedure resects the skin to mid-dermis, skin grafting or other coverage is unnecessary and can lead to unsatisfactory aesthetic results. The best option is excision with secondary epithelialization.

How well did you know this?
1
Not at all
2
3
4
5
Perfectly
37
Q

Which of the following peripheral nerve tumors is most commonly associated with von Recklinghausen disease?

A) Astrocytoma
B) Glioblastoma
C) Neurilemoma
D) Neurofibroma
E) Schwannoma

A

The correct response is Option D.

A neurofibroma is a lesion of the peripheral nervous system, which is derived from Schwann cells, other perineural cell lines, and fibroblasts. Neurofibromas may arise sporadically, or in association with von Recklinghausen disease (neurofibromatosis 1 or NF1). A neurofibroma may arise at any point along a peripheral nerve, and comes in two varieties.

The plexiform neurofibromas are larger tumors that develop inside the body and tend to intimately involve the nerves, blood vessels, and other structures in the body. They can reside deep inside the body or closer to the skin. Plexiform neurofibromas can cause pain, numbness, weakness, and disfigurement. These tumors do have a small chance of becoming cancerous. Plexiform neurofibromas may also be asymptomatic.

Dermal (subcutaneous) neurofibromas are small, nodule-like tumors that grow on or just under the surface of the skin. They can be painful, itchy, disfiguring, or tender when touched, but they have no known potential to become cancerous. Dermal neurofibromas may also be asymptomatic.

Schwannomas are peripheral nerve sheath tumors that can be seen with NF1, but are more commonly associated with neurofibromatosis 2. The major distinction between a schwannoma and a solitary neurofibroma is that a schwannoma can be resected while sparing the underlying nerve, whereas resection of a neurofibroma requires the sacrifice of the underlying nerve. A neurilemoma is another name for a schwannoma.

Astrocytomas and glioblastomas are tumors of the central nervous system. Astrocytomas and optic gliomas can be seen in association with NF1.

How well did you know this?
1
Not at all
2
3
4
5
Perfectly
38
Q

A 60-year-old man presents with a 6-mm lesion of the forehead. The patient states that it has enlarged over a period of 2 years. Examination of a biopsy specimen shows squamous cell carcinoma (adenoid subtype) with a 1.5-mm depth of involvement. Which of the following is the most likely risk factor for recurrence of this patient’s lesion after surgical excision?

A) Anatomic location
B) Depth of involvement
C) Growth rate
D) Histologic subtype
E) Size

A

The correct response is Option D.

A number of risk factors for recurrence have been identified for squamous cell carcinomas.

Histologic subtype is one such factor. Adenoid, adenosquamous, and desmoplastic subtypes are considered high risk for recurrence.

Anatomic location is another such factor and is typically considered in combination with the size of the lesion. High-risk areas include the “mask” areas of the face (eyelids, eyebrows, periorbital, nose, lips, chin, temple, ear), as well as genitalia, hands, and feet. Lesions greater than 6 mm in these areas indicate a high risk for recurrence. Mid-risk areas include the cheeks, forehead, scalp, and neck, with lesions greater than 10 mm indicating a high risk for recurrence. Low-risk areas include the trunk and extremities, with lesions greater than 20 mm indicating a high risk for recurrence. Based on this, this patient with a 6-mm forehead lesion would not be considered at high risk for recurrence.

Rapid growth is a risk factor for recurrence. However, this patient’s lesion grew gradually over a period of 2 years and not rapidly.

Depth of involvement greater than 2 mm indicates a high risk for recurrence.

Other risk factors for recurrence include poorly defined borders, immunosuppression, prior irradiation, site of inflammatory process, neurologic symptoms, moderate/poor differentiation, and perineural/vascular involvement.

How well did you know this?
1
Not at all
2
3
4
5
Perfectly
39
Q

A 13-year-old girl is brought for evaluation because of a 4-month history of severe pain of the tip of the right index finger. There is no history of trauma. On examination, the finger appears normal with no visible swelling or discoloration. The pain is exacerbated by local pressure when the patient writes and during her weekly swimming lessons. MRI (T2-weighted) image is shown. Which of the following is the most likely diagnosis?

A) Digital fibroma
B) Giant cell tumor
C) Glomus tumor
D) Neuroma
E) Venous malformation

A

The correct response is Option C.

This lesion is a glomus tumor. Glomus tumors are benign hamartomas originating from the glomus body, a structure comprised of vascular and neural elements that is responsible for thermoregulation in the skin. These often inconspicuous tumors present with pain, point tenderness, and sensitivity to cold. X-ray studies may show cortical erosion of the bone adjacent to the lesion, and ultrasonography can provide confirmation. MRI is the most accurate imaging modality and the tumor appears as a bright, discrete mass on T2-weighted images.

Digital fibroma is a cutaneous fibroblastic proliferation and would be visible. It rarely causes pain and would not enhance on T2-weighted MRI imaging. Neuroma can cause focal pain as described in the vignette, but the enhancing focal lesion on the T2-weighted MRI effectively rules out solid masses such as neuroma or giant cell tumor (which is common but rarely causes pain or cold intolerance). Venous malformation would enhance on T2-weighted MRI, but would typically present with swelling and would not be as well circumscribed as the lesion shown here.

How well did you know this?
1
Not at all
2
3
4
5
Perfectly
40
Q

Which of the following is the most appropriate surgical treatment recommendation for a 4-cm round sebaceous nevus of the scalp in a child?

A) Excision, skin grafting, and delayed tissue expansion
B) Serial excision without tissue expansion
C) Serial monitoring and selective excision
D) Tissue expansion and excision
E) Tissue expansion, excision, and rotational flaps

A

The correct response is Option C.

Historically, there has been a 10 to 15% reported malignant degeneration in nevus sebaceous in children and the recommendation had been for all lesions to be removed before puberty. More recent studies have shown malignant transformation of these lesions to be less than 1%. Therefore, the decrease in malignant change prompted a more conservative approach to nevus sebaceous based on serial monitoring and excision of lesions that are a source of irritation, difficult to monitor clinically, or aesthetically displeasing. Tissue expansion is an option in patients with giant nevus sebaceous and usually requires one to two rounds of expansion and serial excisions. However, this is not a first line treatment for a small 4-cm lesion. Although serial excision is an alternative reconstructive option, it is recommended only for larger lesions that can be excised in three stages or less. If that is not possible, then tissue expansion is the preferred procedure. Reconstruction with rotational flaps should be planned in consideration of natural facial aesthetic units and an effort should be made to maintain natural brow and hairline position and symmetry and avoid creating tension in the perioral, periorbital, and periauricular regions.

How well did you know this?
1
Not at all
2
3
4
5
Perfectly
41
Q

A 56-year-old woman presents with a 1-cm primary superficial basal cell carcinoma on the left volar mid-forearm. Medical history includes renal transplantation, carcinoma of the right breast managed with lumpectomy and radiation, and treatment for a gunshot wound to the left forearm. The basal cell carcinoma is located within the previous traumatic scar. Which of the following clinical features is an indication for Mohs micrographic surgery in this patient?

A) Basal cell carcinoma arising in traumatic scar
B) History of radiation
C) Immunocompromised status of patient
D) Size of basal cell carcinoma
E) Superficial basal cell carcinoma

A

The correct response is Option A.

The clinical feature in this particular patient that fulfills the criteria to get Mohs micrographic surgery is that the basal cell carcinoma is arising in a traumatic scar. There are many clear indications for Mohs micrographic surgery for basal cell carcinoma: certain size, histology, and anatomic location, all recurrent basal cell carcinomas, and the occurrence of basal cell carcinoma in irradiated skin, traumatic scars, areas with osteomyelitis/chronic ulceration/inflammation, and/or patients with genetic syndromes. Almost all primary basal cell carcinomas in the H and M zones, regardless of pathology (i.e. superficial, nodular, or aggressive), size, or health status of the patient, are candidates for Mohs micrographic surgery. In the L zone, most basal cell carcinomas are also candidates for Mohs micrographic surgery (except all superficial subtypes [irrespective of health of patient], or those less than 1 cm size in immunocompromised patients or nodular subtypes).

Area H: “Mask areas” of face (central face, eyelids [including inner/outer canthi], eyebrows, nose, lips [cutaneous/mucosal/vermilion], chin, ear and periauricular skin /sulci, temple), genitalia (including perineal and perianal), hands, feet, nail units, ankles, and nipples/areola.

Area M: Cheeks, forehead, scalp, neck, jawline, and pretibial surface.

Area L: Trunk and extremities (excluding pretibial surface, hands, feet, nail units and ankles).

In this clinical case, the patient has a basal cell carcinoma that has a favorable pathology (i.e. superficial subtype) in the L zone, not an indication for Mohs micrographic surgery, irrespective of tumor size. Additionally, with a 1-cm tumor of this pathology subtype, her immunocompromised state is not an indication for Mohs micrographic surgery, either. The radiation was remote from the area she developed her basal cell carcinoma, so it is not an indication, either.

How well did you know this?
1
Not at all
2
3
4
5
Perfectly
42
Q

A 50-year-old woman is evaluated for multiple firm, nodular, pink-colored lesions of the scalp, ranging in size from 2 to 4 mm. Examination of a specimen obtained on biopsy shows benign cylindroma. Which of the following is the most appropriate management of these lesions?

A) Cryotherapy
B) Electrodessication and curettage
C) Imiquimod therapy
D) Radiotherapy
E) Surgical excision

A

The correct response is Option E.

Cylindromas are benign adnexal tumors showing an eccrine and an apocrine differentiation. They are found most commonly on the scalp and face, and are more common in women. Solitary cylindromas are generally sporadic in nature. Multiple cylindromas can be seen in patients with Brooke-Spiegler syndrome as an autosomal dominant trait with variable penetrance.

Cylindromas may undergo malignant transformation, and therefore surgical excision is typically recommended, with close postoperative follow-up given high recurrence rates.

Cryotherapy, electrodessication and curettage, and imiquimod are not treatments for cylindromas.

Radiotherapy has been used to treat malignant cylindromas (also known as cylindrocarcinoma or adenoid cystic carcinoma), but not benign cylindromas.

How well did you know this?
1
Not at all
2
3
4
5
Perfectly
43
Q

An obese 47-year-old man comes to the office for pain and drainage in the perineal region. A photograph is shown. Medical history includes hypertension, hypercholesterolemia, and diabetes. Which of the following is the most important factor in reducing the risk of recurrence after surgery?

A) Adjuvant radiation
B) Antibiotic therapy
C) Extent of resection
D) Intralesional corticosteroid injection
E) Method of closure

A

The correct response is Option C.

This patient has hidradenitis suppurative (HS). There are various surgical treatments available to these patients with varying risk of recurrence. Surgical options include incision and drainage, surgical deroofing, local excision, and radical resection of all involved tissue. Reconstructive and closure techniques include secondary healing, primary closure, skin grafting and locoregional pedicled flaps. Numerous studies have looked at the outcomes with various closure techniques and they show that risk of recurrence is likely influenced more by the extent of disease rather than the method of definitive closure. Because of the recurrent nature of this disease, surgery has been considered as the only effective curative therapy for HS. Inadvertent compromise in the margin of resection may diminish the probability of successful healing. Radical resection of all hair-bearing skin with a 1- to 2-cm clear margin of normal tissue is the gold standard and the most important factor in reducing risk of recurrence. Recurrence rates tend to be higher after excision in regions where functional and aesthetic outcomes take priority, and often limit the extent of resection and compromise the ability to obtain clear margins (ie, axilla, perineum, breast). Historically, low-voltage radiation was used as a treatment modality for HS and it is thought to cause complete follicular destruction. Current studies on radiotherapy as treatment for HS are very limited because of concern regarding malignancies arising in radiation fields. Radiation should be reserved for individuals with severely recalcitrant disease and used with extreme caution in younger individuals. Bacterial burden may also diminish the probability of successful wound healing. Topical and systemic antibiotics are still the mainstay treatment for mild HS. Despite their widespread use, few studies have shown their efficacy. In addition, antibiotics do offer relief by reducing the burden of abscesses and pustules in some individuals, but recurrence in these people is frequent. Standard practice of managing acute flares with intra-lesional steroid injections lacks clinical evidence. It has been shown in a series of patients to reduce erythema, edema, size and pain, but no effect on recurrence of disease has been shown.

How well did you know this?
1
Not at all
2
3
4
5
Perfectly
44
Q

A 20-year-old man comes to the office for definitive treatment of a punch biopsy–proven dermatofibrosarcoma protuberans (DFSP) of the scalp. Which of the following is the most appropriate treatment plan?

A) Cryoablation
B) Radiation
C) Referral for Mohs micrographic surgery
D) Resection with 0.5-cm margin
E) Resection with 1-cm margin

A

The correct response is Option C.

Dermatofibrosarcoma protuberans (DFSP) is relatively uncommon, soft-tissue-only tumor that is locally aggressive. Since it can extend along connective tissues along the deep layers, margins required for DFSP need to be no less than 2 cm for an acceptable non-recurrence rate, and even then it is quite high. Predicted rate of recurrence is anywhere from 11 to 20% with a 3-cm margin.

Mohs micrographic surgery has demonstrated to have a much lower recurrence rate, with multiple studies demonstrating less than 10%. Subsequently, Mohs micrographic surgery is the best initial treatment plan for complete resection of DFSP.

Radiation therapy is contraindicated for DFSP. Cryoablation is employed for precancerous skin lesions.

How well did you know this?
1
Not at all
2
3
4
5
Perfectly
45
Q

A 7-year-old boy is evaluated because of new nodular lesions on his skin. Patient history includes a jaw cyst, pits in the hands and feet, and a treated meduloblastoma. Biopsy is planned. Which of the following is the most likely diagnosis?

A) Basal cell carcinoma
B) Melanoma
C) Merkel cell carcinoma
D) Sebaceous adenocarcinoma
E) Squamous cell carcinoma

A

The correct response is Option A.

The patient has basal cell nevus (Gorlin) syndrome. It is an autosomal dominant genetic condition affecting 1 in 56,000. Males and females are equally affected. It is caused by a mutation in the PTCH1 gene. Clinical characteristics include multiple basal cell carcinomas, odontogenic cysts of the mandible, facial dysmorphism, and skeletal abnormalities of the vertebrae, skull, and ribs. 5 to 10% of patients will develop medulloblastoma. The other malignancies are not associated with Gorlin syndrome.

How well did you know this?
1
Not at all
2
3
4
5
Perfectly
46
Q

A 50-year-old woman with a history of scleroderma is evaluated because of a 1.5-cm lesion on her right cheek. Patient history includes basal cell carcinoma excision at the same site 3 years ago. A punch biopsy shows basal cell carcinoma (micronodular subtype). Which of the following is the most appropriate indication for Mohs micrographic surgery in this patient?

A) Histologic subtype
B) History of scleroderma
C) Location of lesion
D) Recurrence of lesion
E) Size of lesion

A

The correct response is Option D.

Mohs micrographic surgery is a surgical technique in which tumor excision and microscopic examination of tissue margins are performed by the same surgeon. Use of a beveled excision and careful mapping of the peripheral and deep margins of horizontal frozen sections allow for comprehensive examination of all the borders of the excised tissue, resulting in excellent cure rates. In addition to the high cure rate, Mohs surgery is a tissue-sparing procedure that is an important advantage in cosmetically and functionally sensitive areas and contrasts with traditional approaches in which a set margin of excision is performed.

Indications for Mohs surgery include recurrent basal cell carcinomas (BCC) and squamous cell carcinomas (SCC), locations prone to recurrence (“H-zone” of the face: inner canthus, nasolabial fold, nose, periorbital, temple, upper lip and periauricular regions, retroauricular, and chin), at/near critical structures (e.g., eye, lip), large tumors (>2 cm), ill-defined tumor margins, aggressive histology (BCC - morpheaform infiltrative, basosquamous, perineural; SCC - poorly differentiated, invasive, perineural), and special hosts (immunosuppressed, basal cell nevus syndrome, xeroderma pigmentosum). Therefore, in this patient, the primary indication for Mohs surgery would be the recurrent nature of her BCC.

How well did you know this?
1
Not at all
2
3
4
5
Perfectly
47
Q

A 25-year-old obese man is evaluated because of new onset of inflamed nodules involving the bilateral axillae. A diagnosis of hidradenitis suppurativa is made. Which of the following is the most appropriate initial medication for treatment of this patient’s condition?

A) Botulinum toxin type A
B) Clindamycin
C) Cyclosporine
D) Etanercept
E) Prednisone

A

The correct response is Option B.

Hidradenitis suppurativa is a chronic inflammatory skin disease. Also known as acne inversa, it is characterized by recurrent nodules and abscesses, typically of apocrine gland–bearing skin.

This patient has mild hidradenitis, with an initial presentation of a few abscesses without sinus tracts or cicatrization/scarring.

Clindamycin applied topically is often used as a first-line therapy for mild hidradenitis. In a randomized, placebo-controlled trial, patients treated with twice-daily topical application of 1% clindamycin solution were found to have significantly less disease burden, and the treatment was well tolerated with few side effects.

Although there have been reports of the use of botulinum toxin in the treatment of hidradenitis, its role and efficacy in this setting are currently unclear.

Etanercept is a TNF-alpha inhibitor. Although some TNF-alpha inhibitors, particularly infliximab, have shown efficacy in patients with moderate-to-severe hidradenitis, data are conflicting with regard to Etanercept.

Prednisone is occasionally used to calm the inflammatory process in severe hidradenitis. However, it does not prevent formation of new lesions and is rarely used for long-term therapy in patients with hidradenitis because of possible adverse effects.

A few case reports have described improvement with cyclosporine in refractory cases of hidradenitis. However, it is typically not used for initial medical treatment of hidradenitis, and duration of use is often limited by adverse effects.

How well did you know this?
1
Not at all
2
3
4
5
Perfectly
48
Q

A 12-year-old girl develops a 12-mm nodule on her right cheek that grows slowly over 2 months. It is firm to the touch, mildly tender, and slightly bluish. There is no redness, ulceration, or visible punctum. Which of the following is the most likely diagnosis?

A) Hemangioma
B) Keratinous cyst
C) Pilomatricoma
D) Sebaceous nevus
E) Spitz nevus

A

The correct response is Option C.

Pilomatricoma (also known as pilomatrixoma or calcifying epithelioma of Malherbe) is a common, benign calcifying tumor of the hair appendages that mostly occur under the age of 20. Most occur in the head and neck, but the extremities and trunk are also affected. Surgical excision is the treatment of choice. Malignancy is very rare. Intraoperative findings show a calcific, friable mass adherent to the undersurface of the skin. Unlike a keratinous or sebaceous cyst, there is no discrete capsule or punctum (plugged pore). Recurrence is reported in the 1 to 2% range. This benign growth is related to a somatic (non-inherited) gene mutation CTNNB-1, that is involved in cell replication of the hair matrix.

Sebaceous nevus presents as a waxy textured skin patch, often present at birth. The scalp is a common site, although it may present elsewhere. Lesions are slow-growing and benign, but over the course of one’s lifetime, they have up to a 50% transformation rate to basal cell carcinoma, with squamous cell carcinoma less likely.

Hemangiomas are cutaneous blood vessel proliferations that are bright red or purple in color and are typically present at birth. While they are also common in childhood, they are more superficial in location and have a very different appearance versus pilomatricomas, which are subepithelial.

Spitz nevi are melanocytic lesions that can occur in children and adults. They can mimic melanoma though they are benign spindle cell lesions. Malignant transformation is not common, though atypical variants exist, therefore excision is recommended. They appear as dark brown or black macules on the skin.

How well did you know this?
1
Not at all
2
3
4
5
Perfectly
49
Q

A 62-year-old man is evaluated because of a new skin lesion in his back. Excision of the lesion with administration of a local anesthetic agent is planned. Medical history shows hypertension and coronary artery disease treated with coronary balloon angioplasty 10 years ago. He takes 81 mg of aspirin daily. Preoperatively, which of the following is the most appropriate aspirin regimen for this patient?

A) Maintaining current dosage
B) Withholding for 1 day
C) Withholding for 2 days
D) Withholding for 7 days
E) Withholding for 14 days

A

The correct response is Option A.

For this patient with increased risk of cardiovascular events, the safest approach is not to withhold aspirin prior to dermatologic surgery.

Aspirin inhibits platelet aggregation by irreversibly binding to the cell’s cyclooxygenase enzyme stores, blocking the production of thromboxane. Its use at lower doses in long-term primary and secondary prevention of cerebrovascular and cardiovascular thrombotic events is well established.

Withholding of daily aspirin in patients with known cardiovascular disease can lead to a platelet rebound phenomenon featuring increased thromboxane production, decreased fibrinolysis, and a prothrombotic state. Discontinuation of oral antiaggregants has been found to be an independent predictor of both death and major ischemic events.

Several studies revealed no significant increase in the risk of bleeding complications after cutaneous surgery in patients who continued taking aspirin perioperatively.

How well did you know this?
1
Not at all
2
3
4
5
Perfectly
50
Q

Which of the following patients’ skin lesions is most suggestive of malignancy on the basis of its clinical features?

A) A 14-year-old boy with a 5-mm, round, brown macule present since birth on the distal aspect of the thigh
B) A 22-year-old woman with a 6-mm friable, pedunculated papule present for 6 weeks on the cheek
C) A 55-year-old man with a 5-mm pink, pearly papule present for 6 months on the lateral nasal sidewall
D) A 70-year-old man with a 1.5-cm waxy, yellow and brown, scaly plaque present for several years on the left temple

A

The correct response is Option C.

Once an individual’s personal and family histories are considered, the history and appearance of a suspicious skin lesion will provide important information and provide clues about a diagnosis.

The “ABCD” (Asymmetry, Border irregularity, Color variegation, Diameter >6 mm) criteria are important for assessing potential malignancy of pigmented lesions. A flat, unchanging pigmented lesion in a young patient is unlikely to be malignant.

A pyogenic granuloma classically presents after local trauma and is characterized by a friable papule that bleeds easily.

Basal cell carcinoma and squamous cell carcinoma are the most common cutaneous skin malignancies. They may present as discrete, slowly growing lesions that have a history of bleeding and ulceration. Basal cell malignancies have a characteristic appearance of round, oval nodules with a shiny, pearly appearance and overlying telangiectasias.

Seborrheic keratoses are common benign skin lesions found in old age. They are a proliferation of immature keratinocytes and have a characteristic appearance of being well circumscribed, scaly, and have a “stuck on” appearance. While unsightly, these lesions have no malignant potential.

How well did you know this?
1
Not at all
2
3
4
5
Perfectly
51
Q

A 75-year-old woman is evaluated because of a new skin lesion on the right upper eyelid. Examination of the specimen obtained on biopsy shows a 1-cm Merkel cell carcinoma. In addition to regional node sampling, which of the following is the most appropriate excision and adjuvant management in this patient?

A) 1-cm margins and chemotherapy
B) 2-cm margins and chemotherapy
C) 1-cm margins and postoperative radiation therapy
D) 2-cm margins and postoperative radiation therapy
E) 5-mm margins and postoperative radiation therapy

A

The correct response is Option C.

Merkel cell carcinoma, an aggressive neuroendocrine tumor, is most likely. It presents in older, immunocompromised women in sun-exposed areas. About 80% of Merkel cell carcinomas are secondary to polyomavirus infection. Treatment of the primary tumor should be wide local excision or Mohs micrographic surgery. For wide local excision of tumors smaller than 2 cm, the recommended surgical margin should be 1 cm. As there is a high rate of occult nodal metastasis, and nodal status is associated with mortality rates, biopsy of the sentinel node is recommended for all cases regardless of primary tumor size. Merkel cell carcinoma is a radiosensitive tumor, and postoperative adjuvant radiation therapy has been shown to decrease local recurrence. Chemotherapy is only currently indicated for palliation and distant metastasis.

How well did you know this?
1
Not at all
2
3
4
5
Perfectly
52
Q

A 58-year-old Caucasian farmer comes to the office because of several pink scaly macules on his cheeks and nose. Biopsy of one of the lesions shows pleomorphic keratinocytes within the basal layer of the epidermis and hyperkeratosis, consistent with actinic keratosis. If left untreated, which of the following is the likelihood that these lesions will become malignant?

A) 0%
B) 10%
C) 30%
D) 50%
E) 70%

A

The correct response is Option B.

The patient has actinic keratoses (AKs). AKs are common in in people with significant sun exposure and are a response to ultraviolet radiation. The likelihood of malignant transformation to squamous cell carcinoma (SCC) is approximately 10%.

There are various treatment modalities for AKs, including cryotherapy, 5-flurouracil (5-FU), photodynamic therapy, superficial glycolic peels, and imiquimod. Cryotherapy with liquid nitrogen is commonly used for isolated lesions, while the other therapies are more commonly used for diffuse disease.

How well did you know this?
1
Not at all
2
3
4
5
Perfectly
53
Q

A 34-year-old woman comes to the office because of a 6 × 7-cm subcutaneous mass below the left scapula. Biopsy confirms dermatofibrosarcoma protuberans. To minimize recurrence yet maximize the chances of primary closure, which of the following is the most appropriate margin when planning wide local excision?

A) 5 mm
B) 10 mm
C) 20 mm
D) 40 mm
E) 50 mm

A

The correct response is Option C.

Several recent studies have confirmed that a surgical margin of 15 to 20 mm is associated with high rates of recurrence-free survival and primary closure when wide local excision is performed. Marginal excision is associated with higher rates of recurrence, and larger wide local excisions (>20 mm) are associated with similar recurrence-free survival but a much higher need for reconstructive surgery. Mohs micrographic surgery has shown promise, with higher initial clearance rates using smaller margins, but the question specifically addressed surgical margins when planning wide local excision.

How well did you know this?
1
Not at all
2
3
4
5
Perfectly
54
Q

A 62-year-old woman with biopsy-proven basal cell carcinoma of left mid cheek presents for consultation for surgical treatment. On physical examination, the lesion is 0.6 cm in diameter and has indistinct borders. Which of the following criteria is the most likely indication for Mohs micrographic surgery in this patient?

A) Anatomical location of the cancer
B) Diagnosis of basal cell carcinoma
C) Indistinct borders
D) Patient age
E) Size of the carcinoma

A

The correct response is Option C.

In the case presented above, the strongest indication for Mohs micrographic surgery comes from the anatomic finding of indistinct borders. Other findings are not necessarily indications for Mohs. Other proven indications are recurrent cancer, high risk zones of the face, morpheaform basal cell cancer, or evidence of neurovascular involvement. By some studies, larger sized basal cell cancers (>2 cm) may be indications.

By definition, Mohs micrographic surgery is a technique for treatment of complex or ill-defined skin cancers with examination of 100% of tissue. One physician performs the procedure, acting in two distinct capacities: surgeon (excising the cancer) and pathologist (reading the slides). Usually, the final pathologic clearance is given on the same day as the resection. This is in comparison with a wide local excision technique, which is performed by two different physicians: a surgeon who removes the cancer and a pathologist who reads it separately. In the latter scenario, although a “wet” read can be done on the same day, the final pathologic evaluation has to await the permanent preparation of slides and a final read. Also, using the routine pathologic evaluation, only 2% of the margins are looked at (using the common “bread loafing” technique), compared with the 100% of tissues evaluated using the Mohs technique.

The vast majority of Mohs resections is done with one or two excisions. Although at times there is a need for multiple excisions, the need for excisions is driven by the positive margins detected pathologically. In Mohs technique, the amount of healthy tissue taken to obtain “clear” margins is the minimum needed to do so, thus preserving the healthy tissue, which can be critically useful in the ensuing reconstruction. These margins are definitively assessed on that same day of surgery, so once the patient leaves the Mohs surgeon’s office, he or she knows the cancer has been completely removed. This is why the margin control is superior in Mohs technique compared with other surgical techniques.

How well did you know this?
1
Not at all
2
3
4
5
Perfectly
55
Q

A 75-year-old man comes to the office because of a 5-year history of a pruritic lesion on the right groin that has been enlarging gradually in size. A photograph is shown. Examination of a specimen obtained on biopsy shows Paget disease. Which of the following is the most appropriate next step in management?

A) Oral miltefosine
B) Topical hydrocortisone
C) Topical miconazole
D) Wide excision
E) Observation

A

The correct response is Option D.

This patient described has extramammary Paget disease and the treatment is wide excision. Paget disease of the breast also presents with eczematous skin changes and is associated with breast cancer. Extramammary Paget disease, however, is an intraepithelial carcinoma that commonly involves the vulvar, perianal, perineal, scrotal, and penile regions. It presents as well-defined, moist, erythematous plaques associated with pruritis. Histopathologic examination shows epidermal acanthosis and elongated rete ridges. Paget cells are large intraepidermal cells with a large nucleus and abundant pale cytoplasm. There is a 7 to 40% rate of associated malignancy. Wide excision is the standard of care, and recent reports have shown that Mohs micrographic surgery can improve evaluation of resection margins.

Observation is not appropriate for extramammary Paget disease given the potential risk for malignancy. Topical treatment with steroids or antifungals is also not appropriate. Oral miltefosine is a treatment for leishmaniasis, an infectious disease that can involve the skin, mucous membranes, and internal organs. Although cutaneous leishmaniasis can present with ulcerating lesions or a dense dermal infiltrate, the histology is predominantly histiocytes, lymphocytes, and plasma cells.

How well did you know this?
1
Not at all
2
3
4
5
Perfectly
56
Q

An otherwise healthy 65-year-old man comes to the clinic because of a 3-cm ulcerated lesion of the scalp. There are no palpable regional lymph nodes. Chest x-ray study shows no abnormalities. A punch biopsy is performed and a diagnosis of well-differentiated squamous cell carcinoma is made. Wide local excision of the lesion is planned. Which of the following is the recommended minimum surgical margin in this patient?

A) 1 mm
B) 2 mm
C) 4 mm
D) 6 mm
E) 12 mm

A

The correct response is Option D.

The most appropriate surgical margin recommended for the clinical scenario described is 6 to 10 mm.

Cutaneous squamous cell carcinoma (cSCC) is the second most common skin cancer after basal cell carcinomas. They are broadly categorized into low- and high-risk lesions depending on size, location, depth of invasion, recurrence, and patient factors such as immunosuppression. A 4-mm margin of healthy tissue is recommended for lower-risk lesions. This category includes well-differentiated tumors smaller than 2 cm in diameter that do not occur on the scalp, ears, eyelids, lips, or nose, and do not involve subcutaneous fat. Therefore, simple excision is most valuable in the treatment of small primary squamous cell carcinomas on the trunk, extremities, or neck, where tissue sparing is less essential. The recurrence rate after the excision of low-risk lesions ranges from 5 to 8%.

A 6-mm margin of healthy tissue is recommended for lesions that are larger than 2 cm, invasive to fat, or in high-risk locations (i.e., central face, ears, scalp, genitalia, hands, feet). Given the cosmetic and functional impact of these wider margins, tumors in this latter category are often removed via Mohs micrographic surgery to achieve high cure rates while sparing healthy tissue. The depth of an excision should always include a portion of the subcutaneous fat.

Cure rates following simple excision of well-defined T1 lesions may be as high as 95 to 99%. The generally accepted 5-year cure rate for primary tumors treated with standard excision is 92%; this rate drops to 77% for recurrent cSCC. No large randomized studies have addressed the issue of appropriate margin size in cSCC, as has been done for melanoma. The recommendations for margin size should be taken only as rough guidelines, with the understanding that large, aggressive lesions frequently have substantial extension beyond the apparent superficial boundary. Therefore, a surgeon’s experience and judgment when planning surgical margins is paramount to the successful treatment of cSCC.

How well did you know this?
1
Not at all
2
3
4
5
Perfectly
57
Q

A 24-year-old woman is evaluated because of a slow-growing subcutaneous mass of the scalp. An excisional biopsy is performed, and pathologic examination shows keratin and its breakdown products. Which of the following is the most likely origin of this lesion?

A) Adipocyte
B) Capillary
C) Hair follicle
D) Mechanoreceptor
E) Sebaceous gland

A

The correct response is Option C.

Pilar cysts, also known as trichilemmal cysts, originate from the outer root sheath of the hair shaft. They present as firm, slow-growing subcutaneous nodules, and may be difficult to differentiate clinically from epidermoid cysts. They are commonly found on the scalp where they are the most common cutaneous cyst. They are lined by stratified squamous epithelium, which undergoes keratinization. In some cases, these lesions can demonstrate aggressive biologic behavior (proliferating trichilemmal tumors, malignant proliferating trichilemmal tumors), in which case they should be completely excised.

Lesions that originate from adipocytes include lipomas and angiolipomas.

Lesions of vascular origin include cherry angiomas and pyogenic granulomas.

58
Q

A 45-year-old woman is evaluated because of a 1-year history of skin abnormalities. Physical examination shows areas of thickened skin involving the forearms and hands. Telangiectasias are seen on the face and oral cavity. A review of systems discloses symptoms of heartburn and dysphagia. Which of the following is the most likely diagnosis?

A) Amyloidosis
B) Dermatomyositis
C) Hypothyroidism
D) Scleroderma
E) Systemic lupus erythematosus

A

The correct response is Option D.

The scleroderma spectrum of disorders includes localized scleroderma and systemic sclerosis, both of which are characterized by thickened sclerotic skin. Localized scleroderma involves only the skin, whereas systemic sclerosis is associated with extracutaneous involvement and is subcategorized into limited and diffuse forms. Limited cutaneous systemic sclerosis is restricted to the hands, distal forearm, face, and neck. Limited scleroderma is sometimes referred to as CREST syndrome, which is an acronym for the following features: calcinosis, Raynaud syndrome, esophageal dysmotility, sclerodactyly, and telangiectasia. Diffuse cutaneous systemic sclerosis includes truncal involvement (chest, abdomen, upper arms, shoulders).

In most patients with systemic sclerosis, there is gastrointestinal involvement. Symptoms are present in more than half of patients and most commonly are related to gastroesophageal reflux disease, resulting in symptoms such as heartburn and dysphagia. Other findings in systemic sclerosis include: diarrhea, mucocutaneous telangiectasia of the face/lips/oral cavity/hands, dyspnea on exertion/interstitial pulmonary disease, and digital infarctions/pitting.

Amyloid infiltration of the skin may produce thickening and stiffness. Telangiectasias are not a feature of amyloidosis. Gastrointestinal involvement with amyloid typically does not cause esophageal reflux or heartburn.

Dermatomyositis is an idiopathic inflammatory myopathy characterized by skin and muscle abnormalities. Cutaneous involvement manifests in the form of distinct rashes, such as of the upper eyelids (heliotrope rash), hands (Gottron sign), and chest and shoulders (shawl sign). However, thickened skin of the trunk and limbs, and telangiectasias are not features of dermatomyositis.

Hypothyroidism can result in cutaneous and dermal edema (myxedema). Other findings include fatigue, cold intolerance, weight gain, constipation, dry skin, myalgia, and menstrual irregularities. It is not associated with telangiectasias.

Systemic lupus erythematosus (SLE) is a chronic inflammatory disease that can affect the skin, joints, kidneys, lungs, nervous system, serous membranes, and/or other organs of the body. Mucocutaneous changes include butterfly rash, erythematous plaques (discoid lesions), and ulcers. SLE is not associated with telangiectasias.

59
Q

A 55-year-old woman comes to the office because of a 3-year history of a raised, dark brown lesion on her back. The lesion is not painful and does not bleed. A photograph is shown. An excisional biopsy is performed. Pathologic examination shows proliferation of cells from the basal layer of the epidermis with cystic inclusions. The lesion exhibits hyperkeratosis, acanthosis, and papillomatosis. Which of the following is the most likely diagnosis?

A) Basal cell carcinoma
B) Malignant melanoma
C) Nevus sebaceous
D) Seborrheic keratosis
E) Verrucous carcinoma

A

The correct response is Option D.

The lesion is a seborrheic keratosis, also known as verruca senilis or pigmented papilloma. Seborrheic keratoses are common benign lesions that may begin to appear during the fifth decade of life. They arise from the basal layer of the epidermis and are composed of well-differentiated basal cells. They often contain cystic “inclusions” of keratinous material called “horn cysts.” Lesions exhibit hyperkeratosis (thickening of the stratum corneum), acanthosis (diffuse epidermal hyperplasia and thickening of the skin), and papillomatosis (skin surface elevation). The growth and depth of pigmentation vary directly with exposure to sunlight. If left untreated, they will enlarge gradually and increase in thickness. Seborrheic keratoses typically do not involute spontaneously. They may appear on the head, neck, and trunk after age 50 years and are often distinctly marked and have a waxy, stuck-on appearance. The surface is soft and oily to the touch. Sizes can range from 1 mm to 5 cm. No treatment is necessary, but these lesions are cosmetically unappealing, and for that reason alone, patients may request to have them removed. It is extremely rare for cutaneous malignancies to develop within seborrheic keratoses.

A verrucous carcinoma is a variant of squamous cell carcinoma. As such, it requires wide local excision with negative margins for treatment.

Nevus sebaceous (Jadassohn nevus) may be described as cerebriform, nodular, or verrucous. It is hairless and can appear on the scalp, face, or neck. When present at birth, it persists throughout life and tends to become more verrucous and nodular during the growth phase associated with puberty. Over time, these lesions are associated with a risk of basal cell carcinoma, which occurs in approximately 15 to 20%. Keratoacanthoma and squamous cell carcinoma may also develop, although with much less frequency than basal cell carcinoma. Because of the risk of malignant transformation, complete excision is generally recommended.

Melanoma results from malignant transformation of the melanocyte; the pigment-producing cell of the body can occur anywhere melanocytes are present, including skin, eye, and the mucous membranes of the upper digestive tract, sinuses, anus, and vagina. The incidence of cutaneous melanoma in the United States has increased steadily over the past 50 years and is now 15 per 100,000. Worldwide, the incidence of melanoma is generally reflective of variation in genetic, phenotypic, and ultraviolet (UV) exposure risk factors. Major risk factors include exposure to UV radiation and genetic predisposition. The exposure risk primarily involves intermittent, damaging exposure to the sun such that history of a severe sunburn (blistering or pain for more than 2 days), even in youth, confers an approximately twofold increase in risk. Patients who have fair skin, blue eyes, red hair, and are prone to freckling are at increased risk for melanoma. Mutations in two genes are associated with hereditary melanoma predisposition.

60
Q

A 60-year-old woman comes to the office for evaluation of a firm, violaceous, 3-cm lesion of the left cheek. An incisional biopsy shows a Merkel cell tumor. The residual margins are positive. Which of the following is the most appropriate management?

A) Chemotherapy
B) Laser ablation
C) Mohs micrographic surgery
D) Radiation therapy and chemotherapy
E) Wide local excision and radiation therapy

A

The correct response is Option E.

Merkel cell tumor is an unusual and highly aggressive skin cancer. More than 50% of Merkel cell tumors occur in the head and neck region. Risk factors for Merkel cell tumors are exposure to sun and ultraviolet light, and immunosuppression. There is a recent association with Merkel cell polyomavirus.

Merkel cell generally presents as a firm, painless nodule (up to 2 cm in diameter) or as a mass (>2 cm in diameter). Although classically described as red in color, it may be flesh-colored or blue. It often enlarges rapidly.

The standard of management is surgical excision combined with radiation therapy. Radiation therapy decreases local recurrence rates. Node-negative patients with no distant metastasis treated with surgery and radiation have 5-year survival rates of approximately 90%. Mohs micrographic surgery and wide local excision are both accepted modalities of surgical resection. It is well known that surgery alone is insufficient to cure or control Merkel cell tumors. Consideration should be given to evaluation of the lymph nodes. Sentinel node biopsy is a common modality.

61
Q

A 14-year-old girl with Fitzpatrick Type V skin comes to the office for evaluation of a nevus on the right side of the face. Physical examination shows a macular, bluish grey, irregular area of hyperpigmentation involving the right infrapalpebral region, nasolabial fold, and zygomatic region. Pigmentation of the right sclera is noted. Which of the following is the most appropriate treatment for this lesion?

A) Camouflage therapy
B) Cryotherapy
C) Dermabrasion
D) Mohs micrographic excision
E) Q-switched ruby laser

A

The correct response is Option E.

This patient has the acquired form of nevus of Ota, also known as nevus fuscoceruleus ophthalmomaxillaris or oculodermal melanocytosis, a dermal melanocytic hamartoma that demonstrates bluish hyperpigmentation along the ophthalmic and maxillary divisions of the trigeminal nerve. The failure of complete embryonic migration of melanocytes from the neural crest to the epidermis results in dermal nesting with the resultant dermal melanin causing the Tyndall effect. This disorder primarily affects darker-pigmented individuals and is more prevalent in females. It has a bimodal age incidence, with a peak at 1 year of age and a second around puberty. The lesion tends to become increasingly prominent with age, puberty, and postmenopausal state. Most patients have no family history. Ophthalmologic examination is recommended because of a reported 10% association with ipsilateral glaucoma. Malignant degeneration to melanoma occurs in approximately 4% of reported cases and is more frequent in lighter-skinned individuals. Diagnosis is mainly clinical with confirmatory biopsy indicated when the diagnosis is uncertain or in rapidly expanding or nodular lesions suggestive of malignancy.

The most effective treatment option is laser therapy, particularly with a Q-switched laser with ruby (694 nm), alexandrite (755 nm), or neodymium: yttrium-aluminum-garnet (1064 nm). The wavelength, pulse duration, and energy densities inherent in the Q-switched laser provide the desired parameters for melanin photothermolysis. Dyspigmentation is a possible complication, although it is mostly transient.

Before the advent of laser therapy, treatment options were suboptimal. Makeup or camouflage therapy offered only temporary improvement. Dermabrasion followed by cryotherapy had the potential for dermal scarring and atrophy and was ineffective for those lesions with deep dermal melanocytes. Surgical excision options were also associated with scarring. Mohs micrographic excision has not been described for excision of these lesions.

62
Q

An 87-year-old Caucasian man comes to the office with multiple 5- to 6-mm lesions on the forehead. He has a long history of sun exposure. Physical examination shows the lesions are erythematous, rough, and scaly. Which of the following is the most appropriate treatment?

A) Dexamethasone
B) Docosanol
C) Imiquimod
D) Isotretinoin
E) Observation with 1-month follow up

A

The correct response is Option C.

This patient’s lesions are most consistent with actinic keratoses. Actinic keratoses are most commonly seen in fair-skinned individuals in areas that have had long-term sun exposure. They are the most common skin lesions to demonstrate malignant potential and may progress to squamous cell carcinoma.

Given the propensity of actinic keratoses to malignant transformation, treatment is generally recommended over observation. For multiple lesions, topical agents are generally effective and well tolerated. Imiquimod is thought to exert its effects by inducing a local immune response as well as apoptotic pathways. Other effective treatments include photodynamic therapy, cryotherapy, 5-fluorouracil, and diclofenac gel.

Dexamethasone is a corticosteroid typically used for inflammatory or autoimmune skin conditions. Isotretinoin is used to treat cystic acne. Docosanol is an antiviral medication used for herpes simplex.

63
Q

A 55-year-old woman is evaluated for a biopsy-proven squamous cell carcinoma of the right preauricular area measuring 2.1 cm in diameter. She is otherwise healthy. Which of the following is the most appropriate next step in management?

A) Electrodessication of the lesion
B) Excision of the lesion with frozen sections
C) Excision with a 2-mm margin
D) Excision with a 4-mm margin
E) Topical application of 5% fluorouracil

A

The correct response is Option B.

Successful local treatment of squamous cell carcinoma of the skin depends significantly on whether the tumors are at high or low risk for the complications of recurrence and metastasis. The external ear, lips, nose, and scalp appear to be high-risk locations for squamous cell carcinoma of the skin.

Squamous cell carcinomas of the skin larger than 2 cm are twice as likely to recur locally and three times as likely to metastasize than tumors that are less than 2 cm in diameter.

Frozen intraoperative examination of specimen edges can be used to judge thoroughness of excision before closure. Frozen sections of margins are recommended for high-risk squamous cell carcinoma and basal cell carcinoma in high-risk areas, lesions more than 2 cm, and any morpheaform basal cell carcinoma.

Electrodessication has excellent cure rates in small, low-risk squamous cell carcinoma of the skin.

Topical application of 5% fluorouracil has a role in the treatment of diffuse actinic keratoses of the face.

Surgical excision is subdivided into excision with standard margins, excision with frozen-section margin evaluation, and Mohs micrographic surgery. For low-risk non-melanoma skin cancers extending into the dermis only, excision with standard margins (4 mm for basal cell carcinoma) is the usual treatment. Adequate margins of 4 mm for low-risk squamous cell carcinoma and 6 mm for high-risk squamous cell carcinoma have been demonstrated by direct tumor extension from the clinical margin but are not necessarily an estimate of cure rate.

Squamous cell carcinomas are slower to invade deeper tissue than are cutaneous malignant melanomas.

64
Q

A 55-year-old woman with a BMI of 32 kg/m2 comes to the office with advanced hidradenitis suppurativa of the groin, lower abdomen, and upper thigh. Which of the following treatments is most likely to have the greatest likelihood of success in this patient?

A) Antibiotics and excision
B) Antibiotics and percutaneous drainage
C) Clindamycin irrigation
D) Intralesional injection of a corticosteroid
E) Sclerotherapy

A

The correct response is Option A.

Percutaneous drainage of hidradenitis suppurativa pustule and fistula tracts, although a plausible short-term fix to address the immediate symptoms, does little to ensure long-term resolution of this very difficult clinical entity with marked impact on quality of life. Addition of topical antibiotic washes or oral antibiotics to percutaneous drainage had no significant effect on long-term recurrence rates.

Hidradenitis suppurativa is a recurrent inflammatory disease of the apocrine glands. It initially develops from follicular occlusion with subsequent abscess, inflammation, fistulas, sinus tracts, and scarring. The sites most commonly affected are the intertrigonal regions such as the axilla, groin, and genital/anal region; although, it can also affect the breasts, hips, and thighs. Women are affected three times as often as men.

Initial treatment involves local wound care and antibiotic therapy. For advanced disease, this may be followed by excision of the area of high-density apocrine glands with minimal undermining and direct closure at the site of the hidradenitis wound. At the site of inadequate resection of an area of infected glands, or if there is a recurrence, radical resection yields the best long-term result. Skin grafting and fasciocutaneous and musculocutaneous flaps have been described to cover the excisional defect. The musculocutaneous flap has been reported to be a valid option for managing infected lesions because of the abundant blood supply. Delayed secondary wound closure, with or without vacuum-assisted closure or skin substitutes, has also shown plausible outcomes.

Sclerotherapy has no role in treatment of hidradenitis suppurativa.

65
Q

A 54-year-old man comes to the office because of a lesion on his back that has enlarged gradually for the past 2 years. He says it occasionally expresses white material when he squeezes it. Physical examination shows a mobile, firm, 2 × 1.5-cm nodule with a central opening and white plug on the mid back. No tenderness is noted on palpation. Which of the following is the most appropriate initial step in management?

A) Excision
B) Incision and drainage
C) Punch biopsy
D) Wide local resection

A

The correct response is Option A.

The patient described has an epidermal inclusion cyst or sebaceous cyst. Cysts can occur anywhere on the body, including the face, back, and chest. These cysts are benign and occur as a result of proliferation of epidermal cells within the dermis. They are usually well circumscribed by a cyst wall made of stratified squamous epithelium. They communicate with the surface through a small opening, which may contain a keratinous plug or blackhead. Epidermal inclusion cysts grow slowly and occasionally can become inflamed and infected. Manual squeezing of the cysts may produce white keratinous material, which is often foul-smelling. Treatment is excision of the cyst in its entirety with care to leave no epithelial remnants in the wound that could develop into a recurrent cyst.

Wide local resection is performed for malignant skin lesions with appropriate negative margins that can be evaluated by frozen-section pathology.

66
Q

A 67-year-old Caucasian man comes to the office because of a 3-month history of a lesion on his forehead that he says has enlarged gradually and sometimes bleeds. Examination shows a nontender, soft, flat, purple lesion on the anterior hairline that is 4 cm in its largest dimension. Results of punch biopsy are consistent with angiosarcoma. Which of the following is the most appropriate treatment?

A) High-dose chemotherapy
B) Radiation therapy
C) Vascular embolization
D) Wide local excision
E) Observation Only

A

The correct response is Option D.

The patient has cutaneous angiosarcoma of the scalp that requires wide local excision with reconstruction.

Angiosarcoma is a rare (2% of all soft-tissue sarcomas) but highly aggressive tumor that is most commonly found in the face and scalp in older Caucasian men. Fifty percent of all cutaneous angiosarcomas are found in the head and neck, and they are also commonly found in the breast and extremities, particularly in patients with a history of lymphedema or radiation therapy. It appears initially as a purple plaque which is often confused with a bruise or cellulitis, which can delay diagnosis. It is frequently multifocal, and local recurrences are common, so wide local excision is recommended, frequently combined with radiation therapy. Preoperative serial punch biopsies of the surrounding tissue can help in determining appropriate resection margins. The tumor can microscopically infiltrate normal tissues for some distance away from the obvious tumor, so wide local excision is necessary.

Observation or embolization is not indicated for an aggressive cancer like angiosarcoma. Although radiation is frequently used postoperatively, its role as a neoadjuvant therapy is not well established. There is no current standard of care for chemotherapy in angiosarcoma patients, and most trials of chemotherapy have shown no survival benefit. There may be a role for taxanes in the treatment in metastatic angiosarcoma.

67
Q

A 39-year-old woman is evaluated because of a 6-month history of a growth on the face. A biopsy is planned. Which of the following findings on pathology can be safely treated with observation only in this patient?

A) Cylindroma
B) Eccrine poroma
C) Nevus sebaceous
D) Trichoepithelioma

A

The correct response is Option D.

Trichoepitheliomas are neoplasms of follicular differentiation. Trichoepithelioma usually presents as multiple, yellowish-pink, translucent papules distributed symmetrically on the cheeks, eyelids, and nasolabial area. The lesions are more frequently seen in women. Lesions are benign but can be confused with basal cell carcinomas clinically and histologically. As they are benign, no further measures should be taken. However, in cases of desmoplastic trichoepithelioma, complete excision or Mohs surgery may be needed to clearly differentiate this entity from a carcinoma.

Eccrine poroma occurs as a solitary lesion usually on the sole of the foot or the palm of the hand in persons older than 40 years. It may also occur on the chest, the neck, or other locations. Eccrine poromas are seen as firm papules less than 2 cm in size. Lesions may occasionally be pedunculated and have a normal or erythematous color and a firm consistency. In rare instances, malignant eccrine poroma or porocarcinoma develops either spontaneously or from long-standing benign eccrine poroma. Treatment is surgical excision.

Verrucous nevus consists of closely set verrucous papules that may coalesce to form well-demarcated plaques. They may be skin colored, brown, or gray-brown. A linear configuration is common, especially for lesions on the limb. Such lesions may appear to follow skin tension lines. On histologic evaluation, there is hyperkeratosis, acanthosis, and papillomatosis. The histologic appearance is essentially that of a benign papilloma. Excision is the most reliable treatment. This may not be practical or advisable if the lesion is extensive or at sites not amenable to simple surgery. The excision should extend to the deep dermis; otherwise, the lesion may recur. Alternative treatments have included laser cryotherapy and electrodesiccation dermabrasion.

Cylindroma presents as either solitary or multiple lesions. Nodules may also be present on the face and rarely on the extremities. The lesion appears in adulthood. The surface is smooth and may be telangiectatic. Cylindromas are usually benign, but malignant changes have been reported. For solitary lesions, treatment is by excision or electrosurgery. For small cylindromas, the carbon dioxide laser may be used. Multiple cylindromas usually require extensive plastic surgery that may be obviated by progressively excising a group of nodules in multiple procedures.

Nevus sebaceous is a distinctive growth most commonly found on the scalp, followed by the forehead and retroauricular region. A nevus of epithelial and nonepithelial skin components, nevus sebaceous sustains age-related modifications in morphologic appearance. The nevus occurs singly and is asymptomatic. Two thirds are present at birth; the remaining third develop in infancy or early childhood. Male and female infants are equally affected. The three-stage evolution of the nevoid condition (newborn, puberty, and adult) parallels the natural histologic differentiation of normal sebaceous glands. In approximately 20% of patients, a third phase of evolution involves the development of secondary neoplasia in the mass of the nevus. A number of benign and malignant “nevoid tumors” may occur, the most common of which is the basal cell epithelioma. The malignant degenerations are relatively low grade; only a few instances of metastasis have been reported. Surgical excision of a nevus sebaceous is recommended because of the high potential for development of basal cell carcinoma and other tumors. The lesion should preferably be excised before puberty because it may enlarge, and the risk of malignant transformation increases after puberty.

68
Q

A 47-year-old man comes to the office because of an asymptomatic lesion of the anterior abdominal wall that has been enlarging gradually for the past 10 years. The lesion has accelerated in growth during the past several months and recently ulcerated. Examination shows a 6-cm, raised, indurated, and irregularly shaped violaceous plaque consisting of firm, irregular nodules. Examination of a specimen obtained on incisional biopsy shows a soft-tissue malignancy arising from mesenchymal cells in the dermis. Which of the following is the most appropriate treatment?

A) Wide local incision and molecular targeted therapy
B) Wide local excision and sentinel node biopsy
C) Wide local excision, molecular targeted therapy, and radiation therapy
D) Wide local excision only
E) Wide local excision, sentinel node biopsy, and chemotherapy

A

The correct response is Option D.

The diagnosis of the described lesion is dermatofibrosarcoma protuberans (DFSP). It accounts for less than 0.1% of all malignant neoplasms and approximately 1% of all soft-tissue sarcomas, but is the most common type of cutaneous sarcoma. It is a malignant mesenchymal tumor that arises in the dermis and is characterized by latency in its initial detection, slow infiltrative growth, and local recurrence if not adequately treated. Distant metastasis is rare and generally occurs as a late sequela after repeated local recurrences. DFSP is most commonly found on the trunk followed by the proximal extremities, and rarely in the head and neck. These tumors have irregular shapes, frequent finger-like extensions, and an infiltrating growth pattern extending beyond clinical margins that result in incomplete removal and a propensity for local recurrence. Treatment primarily consists of wide surgical excision to include margins of 2 to 3 cm beyond the clinical tumor border if possible. Mohs micrographic surgery has been used with good outcomes in aesthetically sensitive areas such as the head and neck where tissue sparing is important. Reconstruction with tissue rearrangement or flaps should be performed after negative margins are confirmed. Most recurrences occur within 3 years of the primary excision, and close follow-up is indicated.

Conventional chemotherapy is rarely used. Radiation therapy is used as an adjunct to surgery for close or positive margins in areas where adequate wide resection alone may result in major cosmetic or functional deficits. Molecular targeted therapy such as imatinib mesylate (Gleevec) is indicated for unresectable, recurrent, or metastatic DFSP. Sentinel node biopsy is not indicated in the treatment for DFSP.

69
Q

A 47-year-old man with hypertension and ulcerative colitis comes to the office because of a painful lesion of the right lower leg. He says that he dropped a typewriter on his leg 1 year ago and noticed a small wound that slowly began to grow larger despite treatment with bandages and topical antibacterial ointment. Examination of a specimen obtained on incisional biopsy shows neutrophilic dermatosis. Cultures grow Staphylococcus aureus. Which of the following is the most likely diagnosis?

A) Leishmaniasis
B) Marjolin ulcer
C) Necrotizing fasciitis
D) Pyoderma gangrenosum
E) Scleroderma

A

The correct response is Option D.

The patient has pyoderma gangrenosum, which is often associated with ulcerative colitis, Crohn disease, or rheumatoid arthritis. The lesion may begin as a small erythematous plaque or have purplish discoloration, and it commonly occurs on the lower extremities after minor trauma. The lesions may rapidly become painful and appear as a necrotizing ulcer. The exact cause of pyoderma gangrenosum is not entirely understood. It is believed the disease is immune-related, such that initial treatments may be primarily medical, including systemic steroids or immunosuppression. Generally speaking, surgery is reserved as a last resort as it is believed that further surgery may exacerbate the condition. Furthermore, recurrent, ulcerative lesions after surgery are not uncommon.

Marjolin ulcer is typically a squamous cell carcinoma arising from a long-standing wound, chronically inflamed tissue, or previous burn scar. The latency period may range from 5 to 30 years from the time of initial injury before developing into a Marjolin ulcer. Treatment generally involves radical resection, as Marjolin ulcers tend to be aggressive.

Leishmaniasis is a disease caused by a protozoan parasite (Leishmania), which is typically transmitted by the bite of a sand fly. In the area of the bite, patients develop ulcerations of the skin. Diagnosis can be made by obtaining a history of an insect bite, a history of travel and exposure to where sand flies reside (Central America, South America, West Asia, or the Middle East), and by obtaining scrapings from the ulcers and looking for the organisms under the microscope.

Scleroderma is a chronic autoimmune disease that is characterized by fibrosis of the skin. The underlying cause is not entirely understood. Patients may present with skin ulcers. Scleroderma may be categorized as systemic or limited. Patients with limited disease may present with Raynaud phenomenon as an early sign. The hands and fingers may be affected with distal tip ulceration and thin, taut skin over joints. Patients with systemic disease may have additional manifestations including gastrointestinal, pulmonary, renal, and cardiac fibrosis.

Necrotizing fasciitis is a rapidly spreading soft-tissue infection involving the skin and subcutaneous tissues. It typically travels along fascial planes and can result in tissue necrosis, sepsis, and death. It can be mono- or polymicrobial, with typical offending pathogens including Group A Streptococcus, Staphylococcus aureus, Bacteroides fragilis, and Clostridium. Treatment includes high-dose intravenous antibiotic therapy and timely surgical debridement. Patients may require repeat debridement frequently in the early stages of treatment to limit progression of the disease.

70
Q

An otherwise healthy 17-year-old boy is brought to the office because of a 3-month history of a bleeding, 8 × 4-mm, red lesion of the left temple that has enlarged rapidly. A photograph is shown. Which of the following is the most likely diagnosis?

A ) Congenital hemangioma
B ) Infantile hemangioma
C ) Kaposiform hemangioendothelioma
D ) Pyogenic granuloma
E ) Venous malformation

A

The correct response is Option D.

The most likely diagnosis is pyogenic granuloma. Pyogenic granuloma usually appears in childhood as a rapidly growing, red lesion. Lesions are commonly small (less than 1 cm) and complicated by bleeding. Definitive treatment is excision.

Congenital hemangioma is fully grown at birth. There are two types: rapidly involuting congenital hemangioma (RICH) and noninvoluting congenital hemangioma (NICH). RICH rapidly involutes postnatally and is usually fully regressed by age 12 months. Treatment is rarely necessary. NICH does not undergo involution; it remains the same size over the course of the patient’s lifetime. NICH is rarely problematic, but may be resected if it causes a significant deformity.

Infantile hemangioma is usually noted 2 weeks after birth and enlarges rapidly over the first few months of life; bleeding is rare. By age 1 year, the tumor begins to regress. Problematic lesions are treated with intralesional corticosteroid, oral prednisolone, or resection.

Kaposiform hemangioendothelioma (KHE) is usually present at birth and does not increase in size. It is often large, superficial, and diffuse. It typically involves the trunk and extremities. Patients commonly have Kasabach-Merritt phenomenon (thrombocytopenia, bruising, bleeding). First-line treatment is vincristine.

Venous malformation is a congenital lesion that is present at birth and enlarges slowly over time. It is typically blue, and bleeding is uncommon. Treatment involves either sclerotherapy or resection.

71
Q

A 17-year-old girl presents to the clinic to have the lesion in the photograph removed. She has had it since birth. She does not like its appearance and has noted it has changed in size and shape during the last two years. Histopathologic examination of the specimen is most likely to show which of the following?

A) Amelanotic melanoma
B) Basal cell carcinoma
C) Nevus sebaceous
D) Squamous cell carcinoma
E) Trichoblastoma

A

The correct response is Option C.

The photograph shows a sebaceous nevus of Jadassohn, and on pathology, a benign organoid nevus is the most likely finding. Given the patient’s age, malignant transformation is very rare (around 2.5%). Until recently, the most common transformation has been thought to be basal cell carcinoma (1.1%). However, it is now thought that many of these were trichoblastomas. A melanoma present since childhood does not fit the presentation of this patient.

72
Q

A 56-year-old man comes to the office because of a lesion on the tip of his nose. He has a 10-year history of coronary artery disease, skin cancers, excisions of the face, and poorly controlled type 2 diabetes mellitus. He has smoked one pack of cigarettes daily for the past 30 years. Medications include 81 mg of aspirin daily. Physical examination shows a 1-cm nodule with irregular borders. Examination of a specimen obtained on biopsy shows morpheaform basal cell carcinoma. Which of the following is the most appropriate next step in management?

A ) Cryotherapy
B ) External beam radiation therapy
C ) Intralesional injection of 5-fluorouracil
D ) Mohs micrographic surgery
E ) Wide local excision

A

The correct response is Option D.

Morpheaform basal cell carcinoma is notorious for having “finger-like” extensions that are not always apparent on gross visual examination and can only be appreciated histologically. The morpheaform subtype of basal cell carcinomas tends to have a higher recurrence rate when simply excised. Mohs micrographic surgery is a technique that uses tangential excisions and frozen section pathology, which more accurately assess the margins of excision. Mohs micrographic surgery may also help minimize the extent of resection. Mohs micrographic surgery is particularly useful in morpheaform basal cell carcinoma, where obtaining negative surgical margins is critical to minimizing the risk of recurrence.

Cryotherapy and external beam radiation therapy are recognized treatments for basal cell carcinomas only when surgical excision is not feasible. In the patient described with no absolute contraindications to surgery, surgical management would be considered the mainstay of therapy.

5-Fluorouracil is an antineoplastic agent and can be used topically for basal cell carcinomas when surgical therapy is not an option. Intralesional injection for this purpose is infrequently performed and would be considered an off-label use of the product. It has a low cure rate.

Wide local excision in the area of the tip of the nose would be inappropriate because this would create a larger defect than may otherwise be required to obtain clear margins. Gross margins of 2 to 3 mm are generally sufficient for non–morpheaform-type basal cell carcinomas.

73
Q

A 2-year-old boy is brought to the emergency department because of a 2-day history of lethargy, fever, and a reddish purple rash on his arms and legs. Temperature is 103.0°F (39.4°C). Physical examination shows a petechial rash on the upper and lower extremities and trunk. Broad-spectrum antibiotics are initiated. After 6 hours, the rash begins to hemorrhage and blister and his digits become ischemic. Which of the following is the most appropriate management?

A ) Activated protein C
B ) Amputation of the ischemic digits
C ) Avoidance of inotropic support
D ) Debridement of skin
E ) Thrombolytics

A

The correct response is Option A.

Purpura fulminans is a rapidly evolving autoimmune syndrome of septic shock and hemorrhagic bullae that can result in massive desquamation and is frequently fatal. Management includes prompt recognition of the infection (which is usually due to Neisseria meningitidis), initiation of broad-spectrum antibiotics, mechanical ventilation, and aggressive fluid resuscitation with inotropic support. Patients develop disseminated intravascular coagulopathy and appear to benefit from replacement of activated protein C. A recently published, multicenter retrospective review of 70 patients documented an amputation rate of 90% and suggested the need for early fasciotomy to improve limb salvage. It is very difficult to determine tissue viability during the resuscitation period; therefore, amputation, debridement, and coverage are delayed until demarcation has occurred. Thrombolytics are not used in this situation because the patient has a hemorrhagic disorder.

74
Q

A 75-year-old man has a biopsy-proven basal cell carcinoma of the eyelid that measures 5 mm in diameter. After a discussion about the patient’s options of resection, he declines Mohs micrographic surgery. Which of the following is the smallest clinical margin beyond the edge of the tumor that will yield at least a 95% chance of cure?

A ) 1 mm
B ) 3 mm
C ) 5 mm
D ) 7 mm

A

The correct response is Option B.

Many studies have had various observations as to the least margin required to cure a basal cell carcinoma. Almost all are retrospective, and conclusions have ranged greatly. Most articles recommend a 4-mm margin. However, in aesthetically sensitive areas like the eyelid, knowing the minimum margin of resection is crucial. Of course, Mohs micrographic surgery is an alternative, but it is not always feasible because of cost, availability, and the difficulty in coordinating two physicians’ schedules. A recent meta-analysis of the literature concluded that for basal cell carcinomas 2 cm or less, a 3-mm margin is sufficient to achieve at least a 95% chance of cure. A surgical margin difference of 1 mm can mean the difference in flap requirements, aesthetic outcome, and in some cases, functionality of the affected area.

75
Q

A 67-year-old man comes to the office with biopsy-proven Merkel cell carcinoma of the forehead. In addition to wide resection, which of the following is the optimal treatment?

A) Administration of interferon
B) Injection of 5-fluorouracil
C) Neoadjuvant chemotherapy
D) Radiation therapy

A

The correct response is Option D.

Merkel cell carcinoma is a rare tumor that usually consists of smooth, painless, indurated, solitary dermal nodules approximately 2 to 4 mm in size. It occurs more frequently in patients older than age 65 years. Merkel cell carcinoma appears most often at sun-exposed sites on white skin; 50% occur on the head and neck, and 40% on the trunk. Merkel cell carcinoma is an aggressive tumor; metastases to regional lymph nodes are noted on initial diagnosis in 12 to 15% of patients. Regional metastasis eventually occurs in one half to two thirds of patients. Local recurrence following primary excision develops in 24 to 44% of patients. Time from diagnosis of the primary tumor to clinically apparent regional nodal metastases is approximately 7 to 8 months. Distant metastases occur ultimately in one third of patients; in order of frequency, metastases occur in the lymph, liver, bone, brain, lung, and skin. The mean time from diagnosis to systemic involvement is 18 months, with death occurring 6 months later. The 5-year survival rate has been reported as 30 to 64%. Two thirds or more of patients with local or regionally recurrent disease ultimately die. Surgical excision is the treatment of choice for primary tumors. The prevailing opinion regarding Merkel cell cancers is that they should be excised with margins similar to those for melanoma.

Sentinel lymph node biopsy is used in clinically node-negative patients with Merkel cell carcinoma.

Radiation should be considered for all patients with Merkel cell carcinoma, as it is radiosensitive. Injection of 5-fluorouracil and administration of interferon have never been shown to be effective in Merkel cell treatment.

In Merkel cell carcinoma, tumor width dictates surgical excision, not tumor depth of invasion (as in melanoma).

Mohs micrographic surgery has been advocated for resection of Merkel cell carcinoma, not only to obtain clear margins, but also to preserve the most amount of tissue.

76
Q

A 42-year-old woman comes to the office because of a 1-year history of a mass in the upper abdominal wall that has enlarged gradually. Examination of a specimen obtained on excision biopsy shows a desmoid tumor. Which of the following is the most appropriate next step in management?

A) Cryoablation
B) Enucleation
C) Excision with a 1-mm margin
D) Excision with a 1-cm margin
E) Observation

A

The correct response is Option D.

Desmoid tumors are relatively rare, technically benign fibrous tumors that may arise in the musculoaponeurotic abdominal wall. These lesions exhibit local invasion and a high rate of recurrence. Wide local excision is regarded as the most effective treatment for these lesions, and intraoperative frozen section is often helpful. Observation will likely result in recurrence, and cryoablation has not been reported as a treatment option for desmoid tumors. Enucleation is not appropriate because the tumor will recur. Excision with a 1-mm margin is not appropriate because wide margins are necessary.

77
Q

A 79-year-old man has a rapidly growing lesion on the left side of his forehead. Physical examination shows a 2-cm, raised, fungating lesion of the left temple with intact facial nerve function and no lymphadenopathy. Examination of a specimen obtained on biopsy is suspicious for squamous cell carcinoma. Four weeks later, the lesion has disappeared, leaving a small circular scar. Excision is performed, and pathologic study shows no evidence of malignancy. Which of the following is the most likely diagnosis?

A) Amelanotic melanoma
B) Cutaneous horn
C) Keratoacanthoma
D) Merkel cell carcinoma
E) Squamous cell carcinoma

A

The correct response is Option C.

The most likely diagnosis is keratoacanthoma, a low-grade malignancy that resembles squamous cell carcinoma both clinically and pathologically. The most common natural course of the disease is that of rapid growth followed by spontaneous regression over several months, which is not seen in squamous cell carcinoma. Keratoacanthoma can progress to squamous cell carcinoma with metastasis. Amelanotic melanoma is an uncommon (less than 5%) form of melanoma characterized by nonpigmented lesions that appear pink or tan and can mimic basal cell or squamous cell carcinoma. It can also occur in the context of cutaneous metastatic melanoma, when cells lack the differentiation required to synthesize melanin. It does not usually ulcerate or regress. A cutaneous horn is a conical projection of hyperkeratosis overlying a hyperproliferative skin lesion such as a seborrheic keratosis or actinic keratosis. Less commonly, it can form from a squamous cell carcinoma or other skin cancer. Ulceration and regression are not characteristic. Merkel cell carcinoma is a rare and aggressive form of skin cancer that shows neuroendocrine features and is prone to metastasis if left untreated.

78
Q

A 65-year-old woman is evaluated because of multiple ulcerative, nonhealing wounds on the left shoulder 8 years after undergoing left modified radical mastectomy and subsequent radiation therapy to the chest wall. She has a 5-year history of chronic lymphedema. Which of the following is the most appropriate next step?

A) Brachytherapy
B) Hyperbaric oxygen therapy
C) Incisional biopsy
D) Isolated limb perfusion with chemotherapy
E) Skin resection only with 5-mm margins

A

The correct response is Option C.

Stewart-Treves syndrome is an aggressive but rare upper extremity lymphangiosarcoma that occurs in postmastectomy patients. Although plastic surgeons typically do not manage these patients, plastic surgeons should be familiar with their diagnostic workup. These patients may be referred to a plastic surgeon for management of a chronic wound, which can be confused with radionecrosis. Initial tissue diagnosis, via incisional biopsy, is the critical first step in management.

Despite a historical 5-year survival of less than 1%, cure rates may be improving, caused in part by refinement of the resection techniques and adjuvant therapies used to treat the original breast cancer. Therapeutic options for lymphangiosarcoma are not standardized but include radical ablative surgery and isolated limb perfusion with tumor necrosis factor and melphalan.

Hyperbaric oxygen may accelerate growth and is contraindicated. Excision of skin only may be indicated, but this approach would require margins greater than or equal to 1 cm.

79
Q

A 40-year-old man comes to the office because of a 12-year history of recurrent painful nodules in his groin, buttocks, and perineum. Physical examination shows deep subcutaneous abscesses. Some have ruptured and formed multiple discharging sinus tracts. In addition to meticulous hygiene, which of the following is the most appropriate management?

A) Antiandrogen therapy
B) Anti-tumor necrosis factor-a therapy
C) Excision
D) Radiotherapy
E) Regular chlorhexidine baths

A

The correct response is Option C.

The patient has hidradenitis suppurativa. The disease presents with tender subcutaneous nodules beginning around puberty. The nodules may spontaneously rupture or coalesce, forming deep, painful dermal abscesses. Eventually, fibrosis and the formation of extensive sinus tracts result. The location of the lesions may lead to social embarrassment.

Due to the multiple interconnected sinus tracts and abscesses throughout an entire region, the patient described has such a debilitating disease that only surgery can adequately address his symptoms. Wide excision of all affected tissue and the underlying sinus tracts is the most effective treatment. It is also advisable to stage the process, preferably with the use of allograft.

For patients with abscesses but no cicatrization or sinuses, hygienic measures and antibiotics are an appropriate first-line therapy. The disease primarily involves the follicular epithelium, which is colonized secondarily and infected by bacteria. Clindamycin and tetracycline have shown benefit in clinical trials. Smaller studies using dapsone and minocycline have shown useful short-term benefit.

Several trials using antiandrogen therapy have been conducted. In a randomized trial comparing ethinyl estradiol 50 mg/cyproterone acetate 50 mg to ethinyl estradiol 50 mg/norgestrel 500 mg in 24 women, both regimens produced improvement in disease activity. The tumor necrosis factor-a inhibitors infliximab and etanercept have also produced favorable outcomes.

Limited lesions can be injected with corticosteroids, and flares can be addressed with short courses of oral or intramuscular corticosteroids. Patients with one or more widely separated, recurrent abscesses with sinus tract formation and scars or patients who have failed the first-line therapies may need more aggressive treatment than those with early stage lesions. Treatments that carry more risk may be worth trying depending on the severity of the patient’s disease course. Long-term immunosuppressive therapy or surgical therapies, such as limited excisions or the laying open of sinus tracts, may be helpful.

Radiation therapy for hidradenitis suppurativa was used extensively in the past. Techniques and responses have varied widely, but poor tissue healing was noted.

80
Q

A 58-year-old man comes to the office because of a 3-month history of multiple light-red, scaly lesions of the scalp and forehead. Physical examination shows extensive sun damage to the face and scalp, including multiple flat lesions measuring between 3 and 10 mm in diameter. Examination of a specimen obtained on shave biopsy shows actinic keratosis without invasive malignancy. In addition to daily application of sunscreen and wearing protective garments, which of the following is the most appropriate management?

A) Application of topical 5-fluorouracil
B) Excision of the lesions with 2-mm margins
C) Repeat biopsy in 6 months
D) Shave excision of the lesions
E) Observation only

A

The correct response is Option A.

Actinic keratosis is a common, premalignant lesion that is a direct result of sun damage. These lesions typically occur in fair-skinned patients who have an extensive history of solar injury. Lesions are flat or slightly raised, red, and scaly. They can be isolated or diffuse. Over time, they can progress to squamous cell carcinoma or other precancer lesions, such as Bowen disease (in situ squamous cell), cutaneous horns, and keratoacanthomas.

All patients must be counseled on sun protection measures, such as daily sunscreen application and wearing protective garments. Observation alone may be applicable for very small lesions or in certain infirm or elderly patients, but it is not appropriate in most circumstances because of the potential for malignancy. The rate of transformation is not clear but has been reported to be 1 to 25% per year per lesion.

Ideal treatment involves topical destructive measures, such as application of 5-fluorouracil cream. Typically, it is applied twice daily for 2 to 4 weeks. Other common treatments include imiquimod cream (Aldara), cryotherapy with liquid nitrogen, and photodynamic therapy with 5-aminolevulinic acid (Levulan). These topical treatments are preferable to excision of the superficial lesions described because they can typically treat multiple and diffuse lesions without extensive scarring. Local redness and irritation does occur and resolves over a period of weeks. Subclinical lesions can also be treated, preventing growth of new lesions. Excision or repeat biopsy should be reserved for isolated lesions that are refractory to less invasive measures. Chemical peels and laser resurfacing have also been described as treatment options.

81
Q

A 64-year-old Hispanic woman comes for evaluation of a biopsy-proven basal cell carcinoma on her right cheek. She says that the lesion has been present for 8 months and has grown rapidly during the past 3 months. A photograph is shown. Which of the following is the most appropriate treatment?

A ) Cryotherapy

B ) Electrodesiccation and curettage

C ) Mohs micrographic surgery

D ) Radiation therapy

E ) Wide local excision

A

The correct response is Option C.

The patient described has a basal cell carcinoma (BCC), which is the most common type of cancer worldwide affecting approximately 1 million people per year. These types of cancers were once more common in people over 40 years of age, but they now are often diagnosed in younger people (85% of lifetime sun exposure occurs before age 18).

BCC starts in the epidermis and grows slowly and painlessly. It most often appears on areas of skin that are regularly exposed to sunlight or other ultraviolet radiation, such as the face, scalp, ears, chest, back, and legs.

These tumors can have several different forms. The most common appearance of BCC is that of a small dome-shaped bump that has a pearly white color. Telangiectasias may be seen on the surface. BCC can also appear as a pimple-like growth that heals, only to come back again and again. A common sign of BCC is a sore that bleeds and heals up, only to recur again; oozing or crusting spots within the sore are also common.

While there are many ways to treat BCC, Mohs micrographic surgery is the most appropriate option in the scenario described, as the lesion demonstrated has indistinct borders, is located in both a high-risk and cosmetically sensitive area, and is growing rapidly. Wide local excision would be indicated for BCC with discrete, distinct borders. Radiation therapy is not a primary option for BCC. Cryotherapy and electrodesiccation and curettage have lower cure rates and higher recurrence rates compared with Mohs micrographic surgery.

Generally accepted indications for Mohs micrographic surgery include the following:

  1. Recurrent or incompletely excised BCC or squamous cell carcinoma (SCC)
  2. Primary BCC or SCC with indistinct borders
  3. Lesions located in high-risk areas (ie, eyelids, nose, ear, nasolabial folds, upper lip, vermillion border, columella, periorbital, temples, preauricular and postauricular areas, scalp)
  4. Cosmetically and functionally important areas, including genital, anal, perianal, hand, foot, and nail units
  5. Tumors with aggressive clinical behavior (ie, rapidly growing, greater than 2 cm in diameter)
  6. Tumors with an aggressive histologic subtype (ie, morpheaform BCC) and/or those with perivascular invasion
  7. SCCs ranging from undifferentiated to poorly differentiated, and SCCs that are adenoid (acantholytic), adenosquamous, desmoplastic, infiltrative, perineural, periadnexal, or perivascular
  8. Tumors arising in sites of previous radiation therapy
  9. Tumors arising in immunosuppressed patients
  10. Basal cell nevus syndrome patients

Mohs micrographic surgery has the highest 5-year cure rates for primary and recurrent BCC.

Five-year recurrence rates for primary BCC:

  1. Mohs micrographic surgery €”1%
  2. Surgical excision €”10.1%
  3. Curettage and desiccation €”7.7%
  4. Radiation therapy €”8.7%
  5. Cryotherapy €”7.5%

Five-year recurrence rates for recurrent BCC:

  1. Mohs micrographic surgery €”5.6%
  2. Surgical excision €”17.4%
  3. Curettage and desiccation €”40%
  4. Radiation therapy €”9.8%
  5. Cryotherapy €”13% (< a 5-year period)
82
Q

A 44-year-old woman has a biopsy-proven basal cell carcinoma of the cheek. An indication for resection using conventional excision, over Mohs micrographic surgery, includes which of the following?

A ) Diameter of 1.2 cm

B ) Location next to the alar rim

C ) Morpheaform subtype

D ) Preexisting burn scar surrounding the lesion

E ) Recurrent tumor

A

The correct response is Option A.

Indications for Mohs micrographic resection of a basal cell carcinoma include morpheaform or other aggressive subtypes, recurrence of a tumor, a previous scar, or locations where the preservation of tissue is critical for final reconstruction.

A basal cell carcinoma of the cheek, measuring 1.2 cm in diameter, would require a several millimeter margin circumferentially. It could be resected and closed without difficulty using a variety of tissue rearrangements, such as a rhomboid flap or a slide-swing flap.

83
Q

A 22-year-old woman comes for evaluation of a lesion on her right anterior thigh (shown). She says that the lesion has been present for a number of years and that she frequently nicks it when shaving. She has no other similar lesions, and no family history of similar lesions. Physical examination shows a single, firm lesion. The nodule contracts with lateral compression. No other abnormalities are noted. Which of the following is the most appropriate next step in management?

A ) Cryotherapy

B ) Excisional biopsy

C ) Mohs micrographic surgery

D ) Shave biopsy

E ) Observation

A

The correct response is Option B.

The lesion is a dermatofibroma, which is a benign nodule derived from mesodermal and dermal cells. They can be found anywhere on the body but most commonly appear on the anterior surface of the lower legs. Studies have not definitely determined whether these are true neoplasms or whether they are fibrous reactions to minor trauma, insect bites, viral infections, ruptured cysts, or folliculitis. Dermatofibromas are asymptomatic, firm, raised papules, plaques, or nodules that can vary in size from 3 to 10 mm in diameter. Coloration ranges from brown to purple, red, yellow, and pink. They are diagnosed based on physical examination and commonly display a Fitzpatrick sign, which is the dimpling or retraction of the lesion beneath the skin with lateral compression.

Underlying autoimmune disorders, such as systemic lupus erythematosus, may be suspected if multiple dermatofibromas (ie, > 15) are found on a patient.

Treatment is generally performed for cosmetic reasons or histologic diagnosis in case there is any question about the clinical diagnosis. Excisional biopsy is the procedure of choice to ensure clear histology, as well as complete removal of the lesion, as it exists in the dermal plane; however, the drawback to this technique is scar formation. Cryosurgery is an option for a less invasive treatment, but it will not completely destroy the lesion. Mohs micrographic surgery is not indicated for this benign lesion. Observation is not advised in this particular case, as the patient described notes a €œsignificant nuisance € that motivated her to seek treatment. Shave biopsy would not completely eliminate the lesion, as it frequently does not penetrate deep enough into the dermis to clear the pathology.

84
Q

A 38-year-old woman has severe hidradenitis suppurativa of the groin and axillae. Which of the following dermal appendages are located in these areas and implicated in the disease process?

A ) Apocrine glands

B ) Eccrine glands

C ) Glomus bodies

D ) Hair follicles

E ) Sebaceous glands

A

The correct response is Option A.

Apocrine glands are uniquely located in the axillae, groin, and perineum, and they secrete a viscid, milky fluid that becomes malodorous with bacterial colonization. Occlusion of the glands causes inflammation and subcutaneous abscess formation with pain, drainage, and foul odor. This crippling disease can be medically managed with chronic suppressive topical and systemic antibiotics but often requires intermittent incision and drainage, or even surgical resection.

Eccrine glands are found throughout the skin, secreting a thin, clear, hypotonic fluid ( €œsweat €). Glomus bodies are located in tissues exposed to the cold, such as the fingertips and ears. They form a thickening in the arterial wall before naturally occurring thermoregulatory arteriovenous shunts, and they are thought to control the flow through these shunts. Although some recent data suggest that hair follicle occlusion and folliculitis lead to apocrine gland occlusion and subsequent hidradenitis suppurativa, hair follicles are located all over the body and do not form the abscesses responsible for the clinical disease. Sebaceous glands secrete sebum, an oily substance that lubricates hair follicles and surrounding skin. They are found throughout the skin, except for the palms and soles. They are found in abundance in the face and scalp.

85
Q

A 72-year-old man is being evaluated because of a 3-month history of lesions on the nasal dorsum and cheek (shown). He is a poor surgical candidate and is treated with imiquimod (Aldara). Which of the following is the most likely mechanism of action of this treatment?

A ) Inhibition of the cyclooxygenase pathway

B ) Inhibition of DNA synthesis

C ) Modification of gene transcription

D ) Modulation of cell differentiation

E ) Stimulation of proinflammatory cytokine production

A

The correct response is Option E.

Imiquimod (Aldara) is a new immune response enhancer that stimulates host cytokine production and induces apoptosis of tumor cells. It has been used to treat actinic keratoses, viral warts, and nonmelanoma skin malignancy. Topical 5-fluorouracil is a topical chemotherapeutic agent that directly inhibits DNA synthesis. Retinoids prevent new skin cancer development by regulating cell differentiation. Topical diclofenac is an anti-inflammatory drug that inhibits the cyclooxygenase pathway and has been found useful in the treatment of actinic keratoses. Interferons control cell differentiation by modification of gene transcription and have been used in combination with retinoids for advanced squamous cell cancers.

86
Q

An 86-year-old woman comes to the office because of an 18-month history of the isolated, painful lesion shown. History includes CREST (calcinosis cutis, Raynaud phenomenon, esophageal dysfunction, sclerodactyly, and telangiectasia) syndrome. Which of the following is the most appropriate initial management?

A ) Debridement

B ) Hyperbaric oxygen therapy

C ) Injection of corticosteroids

D ) Measurement of serum calcium level

E ) Measurement of serum uric acid level

A

The correct response is Option A.

CREST syndrome is a member of the heterogeneous sclerodermas. Calcinosis cutis is the pathologic calcification of soft tissues and skin.

The calcific deposits, when symptomatic, can be tender and painful. They can ulcerate, drain a white chalky substance, and become secondarily infected. In scleroderma, calcific deposits are found predominantly in the extremities and around joints and bony prominences. Deposits are typically found in the flexor surfaces of the hands and the extensor surfaces of the forearms and knees. The deposits rest in the dermis but can also be found in deeper periarticular tissues.

Serum calcium, phosphorus, and alkaline phosphatase levels typically are normal; serum measurement does not contribute to management.

CREST is not associated with gout, so measurement of serum uric acid is noncontributory. Autografting prior to wound debridement and removal of calcific deposits is not recommended. Hyperbaric oxygen therapy may follow debridement.

No consistently reliable pharmacologic treatment, including injection of corticosteroids, has been shown to prevent or eliminate calcinosis. However, surgical excision of localized, painful large deposits can relieve symptoms; recurrence is rare. If calcinosis is diffuse, recurrence is more common. Successful palliation and significant remission of calcinosis using a carbon dioxide laser for debridement was shown in two case reports with a total of seven patients.

Calcinosis cutis is distinct from calciphylaxis, which is a poorly understood and highly morbid syndrome of vascular calcification and skin necrosis typically seen in 1 to 4% of patients with end-stage renal disease. Wound debridement in this setting is controversial and may cause exacerbation of skin necrosis.

87
Q

Which of the following is the most important management for patients with xeroderma pigmentosum?

A ) Cryotherapy

B ) Fluorouraci

l C ) Isotretinoin

D ) Minimization of sun exposure

E ) Skin grafting

A

The correct response is Option D.

Xeroderma pigmentosum (XP) is an autosomal recessive disorder demonstrating defective DNA repair. The ability to repair damage caused by ultraviolet (UV) light is deficient. In severe cases, it is necessary to avoid sunlight completely. The most common defect in XP is a genetic defect whereby nucleotide excision repair (NER) enzymes are mutated, leading to a reduction in or elimination of NER. Unrepaired damage can lead to mutations, altering the information of the DNA in individual cells. If mutations affect important genes, like tumor suppressor genes or proto-oncogenes, then this disorder may lead to cancer. Patients with XP exhibit elevated risk of developing cancer. Multiple basaliomas and other skin malignancies, such as squamous cell carcinoma, occur as early as 8 years of age.

Patients with XP are usually diagnosed at 1 to 2 years of age. The sunburn usually occurs during a child €™s first sun exposure. An unusually severe sunburn that may last for several weeks even after a short sun exposure course is common. Other symptoms include development of many freckles at an early age, irregular dark spots on the skin, thin skin, excessive dryness of skin, rough-surfaced growths (solar keratoses), and skin cancers; eyes that are painfully sensitive to the sun and may easily become irritated, bloodshot, and clouded; blistering or freckling on minimum sun exposure; premature aging of skin, lips, eyes, mouth, and tongue; skin crusting; spidery blood vessels; scaly skin; and oozing raw skin surface.

The most important management is minimizing sun exposure. The number of keratoses can be reduced with isotretinoin (though there are significant side effects). Existing keratoses can be treated using cryotherapy or fluorouracil. Skin cancers are detected at approximately 8 years of age. Frequent skin cancer surgical excisions may be necessary for local control.

88
Q

A 52-year-old man is scheduled to undergo a midline laparotomy for colon cancer. History includes cutis laxa. Examination shows hypoelastic skin that does not spring back when stretched. This patient is at increased risk for which of the following complications?

A ) Anastomotic dehiscence

B ) Hypertrophic scarring

C ) Thrombosis

D ) Ventral hernia

E ) Wound dehiscence

A

The correct response is Option D.

Cutis laxa patients are at a higher risk for ventral hernia formation.

Cutis laxa is a rare condition with hypoelasticity of the skin from a defect in elastin fibers. The skin does not spring back in place immediately upon being stretched, as compared to Ehlers-Danlos syndrome, where the skin springs back without wrinkling. These conditions can be congenital (autosomal dominant, recessive, X-linked recessive) and acquired (drug ingestion, paraneoplastic, postinflammatory). There is an increased risk of hernia formation. Scar formation and healing appear normal; however, procedures to reduce skin redundancy, such as rhytidectomy and blepharoplasty, may require serial procedures as progression of skin laxity continues.

Ehlers-Danlos syndrome (cutis hyperelastica) is characterized by skin hyperextensibility, joint laxity, and tissue friability. It is one of the most frequently inherited collagen disorders and is frequently underdiagnosed. Patients can dehisce their wounds or anastomoses at 1 to 2 weeks. Wound failure is common. Ventral hernias are common and may require closure techniques, such as components separation. Scar formation is wide and extremely thin, and it has been described as papyraceous and cigarette-paper.

Homocystinuria is associated with an increased risk of thrombosis.

89
Q

For each patient with dermatologic symptoms, select the most likely diagnosis (A €“E).

A ) Bazex syndrome

B ) Erythroplasia of Queyrat

C ) Nevoid basal cell carcinoma syndrome

D ) Squamous cell cancer

E ) Xeroderma pigmentosum

A 5-year-old girl is brought to the office because of erythema, inflammation, and bullae formation on the face and neck area one day after spending a sunny day in the park with her family.

A 16-year-old boy comes to the office because of a three-year history of dome-shaped, tan papules on the face, neck, and trunk; erythematous pits in the palm of his hand; and swelling and pain in the molar and premolar areas.

A

The correct response for Item 12 is Option E and for Item 13 is Option C.

Xeroderma pigmentosum is an autosomally recessive disease with incidence of 1:1,000,000 in the United States and 1:40,000 in Japan (incidence in men and women is the same). The usual age of presentation is the first few years of life. Physical findings include acute sun sensitivity with sunburn-like reaction (erythema, inflammation, and bullae formation), pigmented macules, telangiectasias in photodistribution, dry, scaly, and atrophic skin, and multifold increased risk for BCC, squamous cell carcinoma, and malignant melanoma. Treatment includes sun avoidance, sunscreen, dermatologic cancer screening every three months, surgery, cryotherapy, topical treatment with 5-FU and imiquimod.

Nevoid basal cell carcinoma (Gorlin) syndrome is an autosomal dominant disease with a 1/60,000 incidence (incidence in men and women is the same). The usual age of presentation is birth or childhood. Physical findings include dome-shaped tan papules on face, neck, and trunk (basal cell carcinoma [BCC]); erythematous pits in the palm of the hand or plantar aspect of the feet; and swelling and pain in molar and premolar areas (odontogenic keratocysts), colobomas, hypertelorism, and fibrosarcomas. Differential diagnoses include Bazex syndrome, melanocytic nevi, and xeroderma pigmentosum. Diagnosis is usually made by clinical findings and supported by biopsy and skeletal surgery. Treatment includes surgical excision, application of topical 5-fluorouracil (5-FU) and imiquimod, and frequent cutaneous examinations. Patients should avoid radiotherapy, radiographs, and sun as these can induce the formation of new BCCs.

Bazex syndrome is another autosomally dominant inherited disorder. It is characterized by multiple BCCs of the face, follicular atrophoderma of the extremities, localized or generalized hypohidrosis, and hypotrichosis. Physical findings are classically unlike those of Gorlin syndrome and usually do not include pits in the hands and feet and molar pain.

Erythroplasia of Queyrat arises from the squamous epithelial cells of the glans penis. It is synonymous with Bowen disease of the glans penis, is seen mostly in uncircumcised men, and represents an in situ form of squamous cell carcinoma. Progression to invasive carcinoma may occur after a variable period of time.

90
Q

A 60-year-old man is referred to the office by his primary care physician for consultation regarding deformity of the nose caused by rhinophyma. Physical examination shows enlargement of the nasal tip, pitting and scarring of the skin, and several irregular, nodular growths. Which of the following conditions is the most likely cause of these findings?

A ) Acne vulgaris

B ) Facial dermatitis

C ) Hidradenitis suppurativa

D ) Rosacea

A

The correct response is Option D.

Rhinophyma is a condition involving the nose in which the nasal skin is erythematous with telangiectasias and is sometimes purple in color. In severe cases, the skin can have pits, fissures, and scarring. Sebum and bacteria can result in chronically infected skin and often an unpleasant odor. The nasal tip is preferentially enlarged, and as the nasal skin hypertrophies, the aesthetics units of the nose are distorted and obliterated. Patients may suffer from secondary nasal airway obstruction. Tumorous growths can develop late, and a severe aesthetic deformity can result. Rhinophyma is believed to be the fourth stage of evolving rosacea. It is the phase that develops after acne rosacea. Patients have a predisposition to increased facial vascularity that can result in pre-rosacea or frequent facial flushing, vascular rosacea with erythrosis and telangiectasias, followed by inflammatory rosacea or acne rosacea, and ultimately rhinophyma. Although facial flushing caused by vasoactive substances such as caffeine or alcohol may exacerbate the condition, a direct cause and effect relationship has not been found and rhinophyma is believed to be more likely a severe form of acne rosacea. Isotretinoin (Accutane) is used in the treatment of acne rosacea and is not a direct cause of rhinophyma. Similarly, ultraviolet radiation is not known to be a direct cause of rhinophyma.

Acne vulgaris is a common, chronic inflammation of the pilosebaceous units of the face and trunk.

Facial dermatitis usually occurs in the perioral region, mainly in young women. It results in discrete erythematous micropapules that can become confluent, forming inflammatory plaques. Although all conditions listed are disorders of sebaceous and apocrine glands, only rosacea is believed to lead to rhinophyma.

Hidradenitis suppurativa is a chronic, suppurative, cicatricial disease of apocrine gland €“bearing skin of the axillae and urogenital region.

91
Q

A 4-year-old boy is brought to the office because of a lump near his right eyebrow. The lesion has been enlarging gradually since it was first noticed by his parents when he was 3 months of age. On physical examination, a 2-cm, firm, mobile mass is palpated at the lateral aspect of the right eyebrow. The lesion has distinct margins and there are no other related symptoms. Which of the following is the most appropriate initial step in management?

A ) Angiography

B ) CT

C ) Needle aspiration

D ) Surgical excision

E ) Observation only

A

The correct response is Option D.

The most likely diagnosis in the patient described is a dermoid cyst of the orbital region. Most orbital dermoids manifest as solitary masses with distinct palpable margins and without intracranial extension. These lesions are present in the lateral aspect of the upper orbit and usually do not have any related symptomatology. As a result, these patients do not require preoperative evaluation to exclude the possibility of intracranial involvement. Simple excision of the cyst is appropriate at the time of diagnosis.

Rarely, a dermoid cyst may present with indistinct margins. During palpation, the lesion may appear to decrease in size and proptosis may occur. These rare patients may have extension of the dermoid cyst through the lateral orbital wall. In these cases, further evaluation with CT should occur to determine the extent of intraorbital involvement and determine whether more complex surgery is needed to reconstruct the defect. Failure to do this may result in an incomplete resection of the dermoid, possibly leading to inflammation, abscess, or sinus formation.

Observation is not recommended for management of dermoid cysts. These cysts should be appropriately managed with either surgical excision or further preoperative evaluation at the time of diagnosis. Needle aspiration has not been shown to be effective for preoperative evaluation of a dermoid cyst, nor has angiography. Dermoid cysts can occur bilaterally.

In contrast, midline dermoid cysts are much more likely to demonstrate intracranial involvement, and, therefore, immediate surgical resection is not recommended. A CT of the head, including the base of the skull, is needed to rule out intracranial extension prior to removal of the lesion and to rule out other types of lesions such as glioma, encephalocele, angiofibroma, and others. Splaying of the nasal bones on CT can be seen without intracranial extension. However, patients with intracranial involvement will also have defects in the foramen cecum and crista galli and significant dural attachment. These findings indicate the need for craniotomy for adequate exposure and resection of the lesion.

92
Q

A 65-year-old man comes to the office because he has a painful lesion on the left ear that has been present for the past eight months. Tenderness is noted on palpation of the lesion. A photograph of the ear is shown. Which of the following is the most likely diagnosis?

(A) Actinic keratosis

(B) Chondrodermatitis nodularis helicis

(C) Keratoacanthoma

(D) Seborrheic keratosis

(E) Tricholemmoma

A

The correct response is Option B.

Chondrodermatitis nodularis helicis is a chronic, inflammatory, painful, nodular lesion located primarily on the helix or antihelix of the ear. These lesions often produce exquisite tenderness that interferes with sleep. Histologically, the underlying cartilage demonstrates focal degenerative change and surrounding perichondritis. The cause of this inflammatory process is unknown, but long €‘term trauma or sun damage may play a role. Surgical excision of the affected cartilage is recommended if conservative therapy with steroid injections is unsuccessful.

This lesion could be actinic keratosis; however, actinic keratosis usually is asymptomatic except for possible pruritus. It is characterized by a discrete, flat, or slightly elevated lesion with surrounding erythema. These lesions are often multiple and appear scaly due to hyperkeratosis and parakeratosis. Twenty to 25% of lesions progress to squamous cell carcinoma. Treatment includes curettage, electrodesiccation, liquid nitrogen, or application of 5 €‘fluorouracil.

A keratoacanthoma is a round, smooth, pink nodule encircling a large keratinous plug. It enlarges rapidly during a period of weeks and can resolve spontaneously. Histologically, it can be difficult to distinguish from squamous cell carcinoma. Although it is classified as benign and self €‘involuting, the treatment of choice is excision. Seborrheic keratosis is a sharply circumscribed, waxy papillomatous lesion with a friable hyperkeratotic surface and a €œstuck on € appearance. It is an asymptomatic benign lesion and excision is elective. Tricholemmoma is a benign tumor often found in the scalp and has a clinical appearance similar to an epidermal inclusion cyst. Treatment is simple excision.

93
Q

An otherwise healthy 65-year-old woman comes to the office because she noticed a well-circumscribed 2-cm dark brown lesion with a waxy surface on her back two years ago. On excision of the lesion, which of the following is the most likely pathologic diagnosis?

(A) Actinic keratosis

(B) Congenital nevus

(C) Cutaneous horn

(D) Keratoacanthoma

(E) Seborrheic keratosis

A

The correct response is Option E.

Seborrheic keratosis is commonly excised to prevent potential confusion with malignant melanoma. These lesions are often seen in large numbers on middle €‘aged and older patients. They are sharply circumscribed, waxy, and friable with a €œstuck on € appearance. Pigmentation ranges from tan to deep black. Treatment options include shave excision, electrodesiccation, freezing with liquid nitrogen, or simple excision. These lesions do not undergo malignant degeneration and are not easily confused with squamous cell carcinoma.

Actinic keratoses are premalignant lesions. These dry, scaly, rough patches range in color from yellow to dark brown and can progress to squamous cell carcinoma. A congenital nevus is seen in childhood and does not develop in later life. A cutaneous horn has significant keratotic growth and does not have a waxy appearance. Keratoacanthoses are rapidly growing lesions and do not have the appearance described.

94
Q

Which of the following treatments for basal cell carcinoma has the lowest recurrence rate?

(A) Cryotherapy

(B) Electrodesiccation and curettage

(C) 5-Fluorouracil

(D) Mohs surgery

(E) Radiation therapy

A

The correct response is Option D.

Mohs surgery yields the most favorable recurrence rate of 3.4% to 7.9%, establishing it as the treatment of choice for recurrent skin tumors. For lesions in cosmetically or functionally important areas such as the nose, eyes, and lips, Mohs surgery is an excellent treatment choice because of the tissue €‘sparing advantage.

Recurrent tumors may have ill €‘defined margins due to fibrosis from previous treatments. Other treatment modalities, including cryotherapy, electrodesiccation and curettage, radiation therapy, and surgical excision may have subclinical extension when they recur. Recurrence rates of previously treated tumors is 12% with cryotherapy, 40% with electrodesiccation and curettage, 10% with radiation therapy, and 18% with excision. 5-Fluorouracil is used for treating multifocal lesions.

95
Q

A 25-year-old woman comes to the office because she has had a raised raw lesion on the dominant right forearm for the past three years since undergoing a cardiac transplantation. The area has slowly increased in size during the past year. She says she has taken oral and topical antibacterial and antifungal agents irregularly for the past year. She currently takes prednisone. Physical examination shows a 3-cm-diameter mass with a raw surface on the dorsal aspect of the right mid forearm. Which of the following is the most appropriate next step in management?

(A) Acticoat dressing

(B) Biopsy for culture and pathology

(C) Contrast and noncontrast MRI

(D) Excision of the lesion followed by skin grafting

(E) Reduction in immunosuppression

A

The correct response is Option B.

Solid organ transplantation continues to be a successful treatment of organ failure (heart, lung, liver, kidney, and bowel). Solid organ transplant patients are referred for management of various hand and upper extremity problems. Organ transplant recipients are predisposed to diabetes mellitus, obesity, gout, osteoporosis, and atherosclerosis. Transplant combination therapy increases the risk of diabetes mellitus. Risk of gout is also increased. Prolonged corticosteroid use increases the risk for fracture, tendon and ligament rupture, and avascular necrosis. Transplant patients develop acute or chronic ischemia of their upper extremities because of the progression of atherosclerosis. Malignant skin tumors, malignant solid tumors, and lymphoproliferative disorders develop with greater frequency in immunosuppressed patients. Cyclosporine immunosuppression appears to drive the time course of tumor development earlier. Moreover, fungal and bacterial infections such as mucormycosis are more common in immunosuppressed patients. In the scenario described, the most useful intervention is to perform a biopsy of a mass specimen for infection (bacterial, fungal, and atypical pathogen) and histopathology. With this information, the treatment can be cause-specific.

Acticoat is a layered silver ion dressing. When activated with water, Acticoat provides a rapid and sustained release of silver ions within the dressing and to the wound bed for three or seven days. Acticoat is bactericidal against antibiotic multiresistant clinical isolates, specifically against methicillin €‘resistant Staphylococcus aureus and vancomycin €‘resistant enterococcus. Imaging with contrast and noncontrast MRI may demonstrate involved tissue type and volume but does not direct treatment as to cause. Wide excision and skin graft coverage may be required, but the cause must be established prior to treatment. Reduction in immunosuppression may lessen the volume of the mass but risk transplant rejection.

96
Q

A 12-year-old girl is brought to the office by her parents for consultation regarding a lesion on the posterior aspect of the scalp (shown). The lesion has been present since birth and has grown proportionately with the child. Which of the following is the most likely diagnosis?

(A) Compound melanocytic nevus

(B) Keratoacanthoma

(C) Nevus of Ito

(D) Nevus of Ota

(E) Nevus sebaceus of Jadassohn

A

The correct response is Option A.

The lesion shown represents a compound melanocytic nevus, which is a benign neoplasm composed mostly of melanocytes. Congenital melanocytic nevi, such as this one, are believed to represent an anomaly in embryogenesis and, as such, could be considered a malformation or a hamartoma. Melanocytic nevi are common in light- or fair-skinned patients and are relatively uncommon lesions in dark €‘skinned individuals. There is no sexual predilection, and melanocytic nevi are most common in children and young adults. Excisional biopsy is the treatment of choice.

Keratoacanthomas are round, firm, pinkish red or flesh-colored papules that rapidly progress to dome €‘shaped nodules with a central crateriform ulceration or a horn €‘like keratin plug. It is a relatively common low €‘grade malignancy that originates in the pilosebaceous glands and closely and pathologically resembles squamous cell carcinoma.

A nevus of Ota is a blue-gray lesion occurring on the face in the distribution of the trigeminal nerve (particularly the ophthalmic and maxillary branches €”V1 and V2).

A nevus of Ito is similar to the nevus of Ota but follows the distribution of the lateral brachial cutaneous and supraclavicular nerves.

A nevus sebaceus of Jadassohn is a yellow €‘orange, slightly elevated plaque that usually occurs on the face and scalp. It has the potential for malignant transformation to a basal cell carcinoma.

97
Q

A 45-year-old woman comes to the office for consultation regarding multiple actinic keratoses on the face. Physical examination shows fair-skinned complexion and small scaly patches on the face and ears. Which of the following interventions will result in the most desirable long-term aesthetic appearance in this patient?

(A) Cryosurgery

(B) Electrodesiccation and curettage

(C) Excision of the lesions with 2-mm margins

(D) Microdermabrasion

(E) Topical application of 0.5% fluorouracil cream

A

The correct response is Option E.

Several formulations and concentrations of topical fluorouracil have received U.S. Food & Drug Administration (FDA) approval for the treatment of keratotic lesions. Fluorouracil 5% and 0.5% creams have demonstrated, in clinical trials, a marked ability to eradicate keratotic lesions. The cosmetic result is often far better than surgical excision. Irritation at the application site, erythema, and burning are common side effects of both formulations, but comparative data suggest that the fluorouracil 0.5% cream is more cost €‘effective and may be safer, more tolerable, and as effective as fluorouracil 5% cream. Actinic keratosis is a common sun €‘induced skin disease. Recent molecular studies indicate an association between actinic keratosis and squamous cell carcinoma. Although 60% of squamous cell carcinoma cases begin as actinic keratosis, the risk of progression to squamous cell carcinoma is minimal. Other methods of treatment such as cryosurgery, topical trichloracetic acid, and curettage should be used with caution because hypopigmentation with scarring may result. Microdermabrasion is not an acceptable treatment for actinic keratosis.

98
Q

200A 17-year-old girl comes to the office because she has had lesions on the upper arms and in the axillary folds for the past eight years. Physical examination shows clusters of white, vesicular lesions ranging from 2 to 5 mm in diameter. The lesions are obliterated with gentle pressure but refill when pressure is removed. Diffuse swelling is palpable in the underlying subcutaneous tissue. Which of the following is the most likely diagnosis?

(A) Blue rubber bleb nevus syndrome

(B) Lymphangioma circumscriptum

(C) Maffucci syndrome

(D) Osler-Weber-Rendu syndrome

(E) Sebaceous nevus

A

The correct response is Option B.

Lymphangioma circumscriptum consists of clusters of small to moderate €‘sized, clear to whitish €‘appearing vesicular lesions. These lesions can be obliterated with gentle pressure, but they refill with the lymph when the pressure is removed due to the presence of communications with larger underlying cisterns of lymphatic malformation. Clear vesicles may be present on apparently normal skin or may top small papules. Although the classic lesion can occur anywhere on the body, it is particularly common over proximal parts of limbs and adjacent parts of the limb girdle including the upper arm, axillary, pectoral, and scapular regions. Adequate treatment of lymphangioma circumscriptum usually involves excision of the involved skin as well as the deeper lymphatic components.

Blue rubber bleb nevi usually present as raised blue to purple rubbery cutaneous lesions that are easily compressible. This is a rare morphogenetic disorder consisting of malformed vascular channels within the skin and bowel. When blood is expressed from the vesicles, the deflated blister becomes a wrinkled sac. When pressure is released, the deflated blisters refill with blood. The lesions sometime present at birth, but more frequently appear throughout adolescence. Subtotal excisions of tender or painful skin lesions can be of symptomatic relief.

Maffucci syndrome classically presents as blood €‘like, spongy, papular, or pedunculated vascular malformations in association with enchondromas. Visceral vascular malformations may also be present, and the involved bones are usually shortened and deformed. Approximately 20% of the enchondromas can degenerate into chondrosarcoma, which is usually manifest by increasing size and pain of a particular lesion.

Osler-Weber-Rendu syndrome presents as malformed ectatic vessels in the skin, mucous membranes, and viscera appearing after puberty and multiplying with advancing age. Hemorrhage from the lesions presents as epistaxis, hematemesis, hematuria, or melena.

Sebaceous nevi usually develop as firm, plaque-like, waxy-appearing yellowish lesions developing on the scalp. These are potentially premalignant lesions. Generally, these lesions should be excised, because there is an approximately 5% incidence of malignant degeneration, particularly during adolescence. Hair growth in these lesions is sparse to absent.

99
Q

A 23-year €‘old woman with neurofibromatosis comes to the office for consultation regarding lesions on the face and trunk that have appeared during the past 10 years. She is dissatisfied with the cosmetic appearance of a lesion on her nose and is concerned about a lesion on the left hip that has been enlarging and is painful on application of pressure. Physical examination shows multiple small, flesh-colored masses on the face and trunk ranging from 5 to 35 mm in diameter. Punch biopsy shows neurofibromas. Which of the following is the most appropriate management of the lesions on the patient €™s nose and hip?

(A) Injection of a corticosteroid

(B) Nd:YAG laser resurfacing

(C) Radiation therapy

(D) Surgical excision

(E) Observation only

A

The correct response is Option D.

Neurofibromatosis is an autosomal-dominant disorder that can affect the skin, soft tissues, bone, and central nervous system. The treatment of isolated soft tissue and skin nodules is primarily surgical. Lesions are composed of Schwann cells, fibroblasts, and mast cells. Malignant transformation may occur in 3% to 15% of patients over their lifetime. Any lesions that grow rapidly or cause pain should be excised or biopsied to exclude neurofibrosarcoma. Observation is, therefore, not appropriate in this case. Laser resurfacing is not useful for lesions that originate in the subcutaneous tissues. Injection of corticosteroids and radiation have no role in treatment. In healthy patients, lesions may also be removed for cosmetic reasons.

This condition is known as von Recklinghausen €™s neurofibromatosis. The disease can range from mild, as in this case, to severe and devastating deformity. Café €‘au €‘lait macules may develop in early childhood. Other physical signs may include hamartomas of the iris (Lisch nodules) and axillary freckling. Central nervous system lesions, such as acoustic neuromas and meningiomas, may occur. Many patients have learning disabilities, and up to 10% may have mental retardation. Pheochromocytomas are also more common in this patient population.

Multiple variants of the disease exist with neurofibromatosis type 1 (NF-1) primarily affecting the peripheral nervous system and neurofibromatosis type 2 (NF-2) affecting the central nervous system. The incidence is approximately 1 in 3000 live births for NF-1 and 1 in 50,000-120,000 live births for NF-2. It occurs with equal frequency across all races and among men and women. Genetic mutations have been identified in the production of a protein, neurofibromin, which acts as a tumor suppressor.

100
Q

Which of the following benign lesions is most common in individuals with Fitzpatrick type VI skin?
(A) Chondrodermatitis nodularis
(B) Dermatosis papulosa nigra
(C) Melasma
(D) Seborrheic keratosis

A

The correct response is Option B.

Dermatosis papulosa nigra is a variant of seborrheic keratosis and is primarily seen in African Americans and some Asians. These lesions typically begin in adolescence and are most commonly noted in the cheek area. Dermatosis papulosa nigra can also present as multiple lesions. Shave excision, curettage, and cryotherapy are acceptable treatment plans, but hypopigmentation can result.

Melasma is a common hyperpigmentation of the face and neck area that is commonly associated with pregnancy or estrogen use. Seborrheic keratoses are proliferations of normal-appearing epithelial cells commonly seen on the trunk and neck area. Actinic keratoses are dark raised lesions seen in sun-exposed areas. Varying degrees of cellular atypia can be seen and malignant degeneration can be seen in approximately 10% of lesions. Chondrodermatitis nodularis chronica helicis is usually seen in men older than 40 years and is a nodular or ulcerative lesion located on the helix, antihelix, or antitragus. All of these lesions are more commonly seen in light-skinned patients than in patients with darkly pigmented skin.

101
Q

A 65-year-old Caucasian woman comes to the office for consultation regarding 5-mm actinic keratosis. Which of the following percentages best represents the risk of malignant degeneration in this patient?
(A) 0%
(B) 25%
(C) 50%
(D) 75%
(E) 90%

A

The correct response is Option B.

Actinic keratosis is a common premalignant lesion due to the cumulative effects of sun exposure. It has up to a 25% chance of progressing to a squamous cell carcinoma. In general, curettage and electrodesiccation are the mainstays of treatment. Wide local excision is not advocated in the treatment of this premalignant lesion.

102
Q

A 45-year-old woman comes to the office for consultation regarding lesions characteristic of actinic keratosis on the face. Worsening of the lesions most likely indicates the presence of which of the following disorders?
(A) Basal cell carcinoma
(B) Keratoacanthoma
(C) Malignant melanoma
(D) Seborrheic keratosis
(E) Squamous cell carcinoma

A

The correct response is Option E.

Actinic (solar) keratoses are the most common premalignant skin lesions and are usually seen in older, light-complexioned individuals. They are the result of cumulative exposure to ultraviolet light, and 20% to 25% of these lesions progress to invasive squamous cell carcinoma.

Malignant melanoma is not associated with actinic keratosis. It arises from the pigmented melanocytes in the dermis.

Keratoacanthomas have a characteristic rapid growth phase and may involute and disappear without treatment. However, these lesions may undergo malignant transformation; therefore, surgical excision is frequently recommended.

Basal cell carcinomas arise from the epithelium basal layer and occur most often at sites with high concentrations of pilosebaceous follicles. Unlike squamous cell carcinoma, it does not arise from malignant changes occurring in preexisting mature epithelial structures.

Seborrheic keratoses are benign lesions that do not have a risk of malignant transformation.

103
Q

A 57-year-old woman comes to the office for management of a gradually enlarging lesion on the nose (shown). Which of the following is the most appropriate management?
(A) Radiation therapy
(B) Dermabrasion
(C) Carbon dioxide laser ablation
(D) Tangential excision
(E) Full-thickness excision and skin grafting

A

The correct response is Option D.

In this patient with rhinophyma, radiation therapy would not be helpful in reducing the size of the lesion or in preventing growth.

Considering the high incidence of skin cancer with rhinophyma (3% to 10%) and the increasing size of this patient’s nose, any therapy involving ablation (such as cryotherapy or laser destruction) without tissue sampling would be contraindicated. The presence of skin cancer in the specimen should prompt referral to a Mohs’ surgeon for definitive reexcision.

Although dermabrasion alone is indicated for mild to moderate cases of rhinophyma, this patient would benefit most from debulking via tangential excision, dermabrasion for final contouring, and carbon dioxide laser therapy for hemostasis only. Postoperative wound care would include application of petrolatum gauze dressing (Xeroform) and topical bacitracin for up to one week, indefinite use of sunscreen, and topical Retin-A beginning one month after wound closure.

Full-thickness excision with grafting is reserved for severe, recurrent cases of rhinophyma.

In terms of pharmacologic therapy, isotretinoin (Accutane) is indicated for rosacea and early rhinophyma, but patients must wait one year after discontinuation of this drug before undergoing surgery. Isotretinoin destroys sebaceous glands and would prevent reepithelialization after tangential excision, dermabrasion, or laser resurfacing.

The photograph below shows the patient after undergoing a combination of direct and tangential excision.

104
Q

A 2-year-old girl is brought to the office for consultation because she has had the lesion shown for the past two months. Physical examination shows tenderness and a fragile eschar that bleeds profusely when touched. Which of the following is the most likely diagnosis?
(A) Basal cell nevus syndrome
(B) Mucocele
(C) Nevus sebaceus of Jadassohn
(D) Pyogenic granuloma
(E) Spitz nevus

A

The correct response is Option D.

The facial lesion in this child is a pyogenic granuloma. These lesions are common in children and young adults. Usually rapidly developing, they are prone to bleeding and local irritation. Treatment consists of excision and cauterization of the base. Nevus sebaceus of Jadassohn is present at birth and presents as salmon-colored and waxy. Nevus sebaceus is treated with complete excision, as malignant potential transformation to basal cell cancer is possible. Spitz nevus lesions are usually smooth and range in color from pink to brown; histology is similar to malignant melanoma. Appropriate treatment is excision; a spitz nevus also has capability of malignant transformation. Basal cell nevus syndrome appears as multifocal basal cell cancer in teens and young adults, with additional findings of jaw cysts and palmar skin pits.

105
Q

A 44-year-old woman has had a persistent nodular lesion measuring 5 mm in diameter on the anterior scalp for the past year. Pathologic evaluation of a specimen obtained on excisional biopsy shows dermatofibrosarcoma protuberans. Surgical resection with a 1-cm border and scalp rotation is used for closure. Which of the following is the most likely long-term outcome?

(A) Brain metastasis
(B) Local recurrence
(C) Pulmonary metastasis
(D) Regional nodal metastasis

A

The correct response is Option B.

Dermatofibrosarcoma protuberans (DFSP) is a low- to intermediate-grade sarcoma that typically is aggressive and has a local recurrence rate of up to 60%. Tumor cells tend to invade local tissue with tentacle-like projections in skin that appears normal clinically. The head and neck are involved in 14% of affected patients, and these tumors account for 1.4% of all head and neck sarcomas. In the head and neck region, the scalp and supraclavicular fossa are most often involved. Recurrence rates are higher for head and neck lesions (in up to 75% of affected patients) than rates in other sites, possibly because the extent of surgical resection is limited. Regional and distant metastases are uncommon, occurring in less than 4% of affected patients.

Treatment of DFSP includes wide surgical resection with margins of 3 cm. Many authors suggest that Mohs’ surgery is associated with a decreased rate of recurrence in DFSP. The effectiveness of adjuvant radiation has not been clearly substantiated.

106
Q

A 70-year-old man has had a painful, ulcerated nodule on the helical rim of the left ear (shown) for the past two years. The lesion was cauterized once by a dermatologist, but it never resolved completely. Punch biopsy shows ulcerated epidermis over an area of degenerated dermal collagen, lymphocytic infiltrate, and fibrous thickening of the perichondrium. Which of the following is the most likely diagnosis?

(A) Chondrodermatitis nodularis helicis
(B) Grade II pressure ulcer
(C) Proliferative actinic keratosis
(D) Suppurative chondritis
(E) Ulcerated basal cell carcinoma

A

The correct response is Option A.

The physical findings and biopsy results suggest a diagnosis of chondrodermatitis nodularis helicis chronica (CNHC). This type of perforating collagenosis affects individuals over age 50 and, based on appearance alone, is commonly confused with sun-related neoplasms of the skin. It may develop from the minor trauma of sleeping on the affected ear. Because it is very painful, the patient typically avoids pressure or friction on the affected area. Most CNHC lesions are 4 to 6 mm in diameter; they rarely exceed 1 cm. They may occur as two lesions close to each other. The primary pathologic finding may be degenerated dermal collagen trying to extrude through the epidermal surface, which is perpetuated by a chronic lymphocytic infiltrate.

The perichondrium and underlying cartilage may show degenerative changes, including fibrosis and patchy hyalinization of the cartilage. Slitlike spaces that contain fibrin occur in the midst of the degenerated collagen and at the dermoepidermal junction. CNHC is best treated with topical or intralesional steroid therapy because cauterization is associated with a recurrence rate of approximately 20%.

A diagnosis of grade II pressure ulcer is unlikely because such a pressure ulcer should heal within a few weeks in an otherwise healthy, mobile patient. Proliferative actinic keratosis also is an unlikely diagnosis. This premalignant condition of the skin is due to damage from ultraviolet light and is a precursor to squamous cell carcinoma. Actinic keratosis usually appears as pinkish, scaly, macular lesions. Hypertrophic or proliferative actinic keratosis may appear more verrucous and may already be a focus of squamous cell carcinoma. Actinic keratosis affects the epidermis, not the dermis, and may cause acanthosis, hyperkeratosis, and parakeratosis. It usually responds to curettage and cautery.

Suppurative chondritis is an unlikely diagnosis because it is a florid condition that presents with a painful, swollen, red ear that drains pus. The pinna takes a sprung-out position, and polymorphonuclear cells predominate in the inflammatory infiltrate. Ulcerated basal cell carcinoma is not an appropriate diagnosis for this patient. Basal cell carcinoma can present as a noduloulcerative lesion, which typically is pruritic and painless. Histologically, it includes pallisading nests of basal cells that infiltrate the dermis. The dermal collagen usually is normal but may show changes caused by sun damage, such as solar elastosis.

107
Q

A 30-year-old woman with melasma comes to the office for consultation regarding treatment options. Examination shows Fitzpatrick type V skin. Which of the following treatments is most appropriate in this patient?

(A) Erbium laser
(B) Phenol/croton oil
(C) 1% Tretinoin
(D) 35% Trichloroacetic acid

A

The correct response is Option C.
Melasma may be difficult to treat in dark-skinned (Fitzpatrick type IV or V) patients due to side effects, most commonly hypopigmentation. For years, the gold standard for treating dark-skinned patients with melasma was alpha-hydroxy acid peel, such as with 70% glycolic acid. Recent data have shown a 1% tretinoin peel, weekly or biweekly, to be equally as effective as 70% glycolic acid for treating melasma in these patients.

Erbium laser treatment is not appropriate because it may cause hypopigmentation in dark-skinned patients. Neither phenol oil nor croton oil is indicated in dark-skinned patients because they may produce hypopigmentation. Also, 35% trichloroacetic acid (TCA) is not recommended in dark-skinned patients for similar reasons. However, 10% or 15% TCA may be used with fewer adverse effects in these patients.

108
Q

Keratoacanthoma is a subtype of which of the following tumors?

(A) Basal cell carcinoma
(B) Malignant melanoma
(C) Merkel cell tumor
(D) Sebaceous carcinoma
(E) Squamous cell carcinoma

A

The correct response is Option E.
Keratoacanthoma is a subtype of squamous cell carcinoma. It is well differentiated in most cases, but can exhibit a rapid growth phase with extensive tissue destruction. This is particularly problematic in the central area of the face, where aggressive perineural, subcutaneous, and intravenous invasion occurs. A rapid growth phase may occur over a period of a few weeks and can be followed by a dormant period. Complete spontaneous resolution can occur in the area without recurrence, although this resolution can leave a sizable area of destruction. These lesions are usually solitary and occur in sun-exposed areas in both men and women older than age 40 years. The hallmark physical feature of keratoacanthoma is a central destructive crater with a distinct fleshy rim. A central plug of keratin can also be associated.

Sebaceous carcinomas are uncommon skin malignancies that arise most frequently on the eyelids. These tumors contain sebaceous cells and are typically organized into circumscribed lobules. Periorbital primary lesions can act aggressively with high rates of early metastasis. Merkel cell tumor is a neuroendocrine carcinoma that is biologically aggressive and usually involves the head and neck area. Although basal cell carcinoma and malignant melanoma are not generally associated with keratoacanthoma, they can be seen in combination with keratoacanthoma because all of these tumors arise in sun-exposed areas.

109
Q

In the eyelid, which of the following malignant skin lesions most commonly originates in the meibomian gland?

(A) Basal cell carcinoma
(B) Malignant melanoma
(C) Merkel cell carcinoma
(D) Sebaceous carcinoma
(E) Squamous cell carcinoma

A

The correct response is Option D.

Sebaceous carcinomas arise from the adnexal epithelium of sebaceous glands and can be divided into ocular and extraocular subtypes. The ocular subtype is more common and typically arises from the meibomian gland in the upper or lower eyelid. Metastasis is common. Extraocular sebaceous carcinomas arise primarily in hair-bearing areas.

Basal cell carcinomas are the most common skin tumors of the eyelids. Malignant melanomas arise from melanocytes in the eyelids and other areas. Merkel cell carcinomas are rapidly metastasizing lesions that occur in the ocular area and arise from neuroendocrine cells in the epidermis. Squamous cell carcinomas arise from the malpighian layer of the skin.

110
Q

Which of the following pigmented nevi is most difficult to distinguish clinically and histologically from malignant melanoma?

(A) Blue nevus
(B) Mongolian spot
(C) Nevus of Ota
(D) Nevus spilus
(E) Spitz nevus

A

The correct response is Option E.

Spitz nevus is uncommon but typically seen on the head and neck region in children. This lesion can also occur in adults, especially in the extremities. It can be very difficult to differentiate clinically and histologically from malignant melanoma. Typically occurring in fair-skinned individuals, incidence of Spitz nevus is equal among men and women. Approximately 70% of cases develop during the first two decades of life. The histologic distinction between Spitz nevus and malignant melanoma is equivocal in approximately 8% of cases.

A malignant melanoma is an asymmetric lesion with an irregular border and an uneven or varied color. It can be larger than 6 mm in diameter and commonly affects sun-exposed areas of the skin, but it also may occur on mucosal surfaces, especially in the eyes. It results from cancerous growth of melanocytes. It may be readily differentiated clinically or histologically from the blue nevus, Mongolian spot, nevus of Ota, and nevus spilus. A blue nevus is a small intradermal papular lesion composed of dermal melanocytes. It rarely undergoes malignant changes. A Mongolian spot and nevus of Ota are dermal melanoses. These brown, flat patches appear in different areas. A Mongolian spot is present at birth and located in the sacral area. A nevus of Ota occurs on the face in the distribution of the trigeminal (V) nerve. A nevus spilus is a café-au-lait spot (light brown patch) with numerous darker speckles within it. This lesion usually appears later in life.

111
Q

Which of the following is the most effective management of herpetic whitlow?

(A) Application of a dry dressing
(B) Debridement and skin grafting
(C) Incision and drainage of blisters
(D) Oral administration of a second-generation cephalosporin
(E) Topical administration of acyclovir

A

The correct response is Option A.

Herpetic whitlow is a herpesvirus infection of the digits that commonly affects dentists and healthcare workers. The condition is characterized by severe pain and the presence of small vesicles containing clear fluid; it usually resolves within three to four weeks.
The most appropriate management is symptomatic application of dry dressings.

Because herpetic whitlow resolves without wounding, debridement and skin grafting are not needed. An incision and drainage procedure is contraindicated because it can spread the infection and cause secondary bacterial infections. Because the infection is of viral origin, cephalosporin antibiotics are of no use. Topical acyclovir has minimal benefit.

112
Q

In a 56-year-old man who has a lesion on the tip of the nose, histopathologic examination of a punch biopsy specimen shows basal cell carcinoma. Mohs’ micrographic resection is recommended if this patient has which of the following types of basal cell carcinoma?

(A) Morpheaform
(B) Nodular
(C) Pigmented
(D) Ulcerated

A

The correct response is Option A.

Mohs’ micrographic resection involves serial excision of the visible tumor in layers of increasing depth followed by immediate microscopic examination of the excised tissue. A map of the specimen is created and divided into sections; the presence of tumor in a certain section necessitates further excision in that section only. Serial excision continues until there is no longer any microscopic evidence of tumor. Cure rates for this technique, when used for treatment of basal cell carcinoma, have been reported to range from 98.2% to 99.9%, compared with 94% to 95% for standard surgical excision.

Mohs’ micrographic resection is appropriate for removal of basal cell carcinomas, especially recurrent lesions and those lesions that are located in anatomically sensitive sites, such as the eyelid margin, medial canthus, and nasal ala. This technique is particularly effective for the morpheaform variation of basal cell carcinoma, also know as sclerosing basal cell carcinoma, because of the difficulties encountered when attempting to visually determine the tumor margins, which are often not well demarcated. In addition, because morpheaform basal cell carcinoma is more prone to recur, pathologic evaluation of the margins is indicated prior to reconstruction.

Because nodulated, pigmented, and ulcerated basal cells typically have distinct margins and are amenable to primary excision, Mohs’ micrographic resection is usually unnecessary.

113
Q

Which of the following medications is most appropriate to prevent and to treat heterotopic ossification?

(A) Calcitonin
(B) Etidronate
(C) Mithramycin
(D) Phosphate

A

The correct response is Option B.

Heterotopic ossification is an abnormal proliferation of bone that occurs in patients who have sustained burns, spinal cord injuries, and closed head trauma. This condition becomes apparent initially on radiographs one to four months after injury. It may occur in up to 40% of patients who sustain spinal cord injuries and is clinically significant in one-half of those patients.

Etidronate (Didronel) and pamidronate (Aredia) are most appropriate for prevention or treatment of heterotopic ossification. These agents act primarily on bone to inhibit the formation, growth, and dissolution of hydroxyapatite crystals and do not impair fracture healing.

Calcitonin, mithramycin, and phosphate decrease serum calcium levels and do not directly affect heterotopic ossification.

114
Q

A 55-year-old man has a lesion on the right forearm that has enlarged over the past six weeks. A photograph is shown above. Which of the following is the most likely diagnosis?

(A) Cylindroma
(B) Dermatofibroma
(C) Keratoacanthoma
(D) Seborrheic keratosis
(E) Syringoma

A

The correct response is Option C.

The most likely diagnosis is keratoacanthoma, a common cutaneous neoplasm that typically occurs in men older than age 50 years. Keratoacanthomas grow rapidly over several weeks and are believed to regress spontaneously if left untreated. They are characterized by an umbilicated center with a keratin plug. Because they are difficult to distinguish from squamous cell carcinoma and may indeed be linked, excision for histopathologic confirmation is recommended.

Cylindromas are round, firm, fleshy tumors of the scalp that are rarely solitary.

Dermatofibroma is a fibrous, papular lesion characteristically found on the lower extremities in young adults.

Seborrheic keratosis has a waxy, greasy, or pressed-on appearance and generally occurs on the face and trunk of older persons.

Syringoma is a flesh-colored or yellow papule that typically develops in females during adolescence or early adulthood. These lesions may be multiple and often occur only on the lower eyelids.

115
Q

A 51-year-old woman has had multiple lesions around the nostrils for the past two years. She says that the lesions first appeared similar to pimples and have enlarged. A photograph is shown above. The lesions have not resolved with intralesional injections of corticosteroids. Findings on nasal smear and tissue cultures are negative. Histologic examination of a biopsy specimen of a lesion shows noncaseating epithelioid granulomata.

Which of the following is the most likely diagnosis?

(A) Cutaneous sarcoidosis
(B) Keloids
(C) Neurofibromatosis
(D) Rhinosporidiosis
(E) Wegener’s granulomatosis

A

The correct response is Option A.

In this 51-year-woman who has the findings shown in the photograph, the most likely diagnosis is cutaneous sarcoidosis. This condition occurs in 10 Caucasians and 35 African Americans per 100,000 of population. Although sarcoidosis typically affects the lungs and intrathoracic lymph nodes, cutaneous symptoms, such as maculopapular eruptions, sarcoidal plaques, lupus pernio lesions, and subcutaneous and cutaneous nodules, occur in 25% of patients. Histologic examination of a biopsy specimen of affected skin shows noncaseating granulomata and aggregates of epithelioid cells and giant cells containing Schaumann (asteroid bodies) within the dermis.

Appropriate treatment of cutaneous sarcoidosis is intralesional administration of a corticosteroid or oral administration of hydroxychloroquine sulfate or methotrexate. Oral corticosteroids are reserved for patients with severe cutaneous disfigurement or symptomatic involvement of internal organs. Surgical options are limited because of the systemic nature of this condition.

In patients with keloids, histologic examination shows extensive fibrous tissue with scattered collagen fibers but without a definite architecture or a granulomatous pattern.

Neurofibromatosis is characterized histologically by swirls of fibroblastic cells, collagen, and nerve fibers. Granulomas are not present.

Rhinosporidiosis is a fungal granulomatous condition of the nose caused by Rhinosporidium seeberi bacteria. Affected patients have pedunculated masses with reddish-purple discoloration. Although findings on culture are usually negative, microscopic examination of nasal smears typically shows sporangia.

Wegener’s granulomatosis is a systemic disease that initially causes localized perforation of the septum or multiple intranasal ulcerations. Histologic examination shows findings consistent with necrotizing granulomatous vasculitis.

116
Q

A 46-year-old woman has had painful purpura-like lesions on the breasts, abdomen, and lower extremities for the past three weeks. She currently undergoes dialysis for end-stage renal disease. Physical examination shows several dry, leathery, full-thickness areas of skin necrosis. A photograph is shown above. Calciphylaxis is diagnosed.

Laboratory studies are most likely to show an increased serum level of which of the following?

(A) Calcium
(B) Glucose
(C) Parathyroid hormone
(D) Protein C
(E) Thyroid-stimulating hormone

A

The correct response is Option C.

Calciphylaxis, also known as uremic gangrene syndrome, is a rare complication of end-stage renal disease. This condition is characterized by painful, ischemic, violaceous skin lesions on the extremities and sometimes the trunk that exhibit a livido reticularis pattern. Full-thickness skin necrosis ensues, leading first to secondary infection, then to sepsis, and frequently to death; the mortality rate in patients with calciphylaxis has been reported to be as high as 60%. Histologic examination of affected skin shows fat necrosis, calcification of the subcutaneous tissues, and microcalcifications in the intima and media of small arteries. Luminal narrowing of the vessels with intimal hyperplasia is common, and complete occlusion also occurs.

Calciphylaxis is a result of the metabolic imbalance in calcium and phosphate homeostasis that is frequently observed in patients with renal failure. Secondary hyperparathyroidism causes an abnormal increase in calcium X phosphate product, which can lead to soft-tissue calcification. In these patients, serum levels of parathyroid hormone are most likely to be increased. Hyperphosphatemia is also characteristic.

The lesions that occur in patients with calciphylaxis rarely heal spontaneously and frequently become infected. Therefore, various management options should be considered, including debridement of the lesions, topical administration of antimicrobial agents, and frequent dressing changes, followed by skin grafting, or direct excision and closure of the lesions. However, in this patient, the dry, leathery composition of the affected skin suggests that infection has not yet developed. Subtotal thyroidectomy is also advocated for treatment in some patients.

Although hypercalcemia may be present, serum calcium levels are normal in most patients with this condition.

Many patients with end-stage renal disease also have diabetes mellitus, but hyperglycemia is not typically associated with calciphylaxis.

A deficiency of, and not an increase in, serum protein C levels has been proposed to be a contributing factor to tissue death in these patients because it causes thrombosis in small vessels.

Increased serum levels of thyroid-stimulating hormone occur in patients with hypothyroidism.

117
Q

Which of the following pigmented nevi is classified as a hamartoma?

(A) Acral nevus
(B) Becker nevus
(C) Dysplastic nevus
(D) Halo nevus
(E) Spitz nevus

A

The correct response is Option B.

A Becker nevus is a common benign cutaneous hamartoma with epidermal and/or dermal elements. These lesions can occur at birth but often develop during the second and third decades of life, and males are affected more frequently. Becker nevi are typically brown patches or plaques on the upper trunk. Hypertrichosis is commonly associated, and an underlying smooth muscle hamartoma is present. The term “nevus” is actually a misnomer because histologic examination of this lesion shows no nevus cells.

Acral nevi are typically junctional or compound and are located on the palmar or plantar surfaces.

A dysplastic nevus is referred to by several terms, including Clark’s nevus, atypical nevus, or atypical mole. This lesion is a precursor to malignant melanoma.

A halo nevus, or Sutton’s nevus, is a central melanocytic nevus surrounded by a rim or halo of hypopigmented skin. This loss of pigmentation typically corresponds with spontaneous regression of the central nevus and is thought to occur secondary to a cell-mediated and humoral immune response to nevus antigens.

A Spitz nevus is a benign proliferation of melanocytes on the face, trunk, or extremities. This lesion develops in children and is characterized by a rapid, irregular growth phase. Because it is difficult to differentiate from malignant melanoma, excision for histopathologic confirmation is recommended.

118
Q

A 60-year-old farmer with multiple actinic keratoses undergoes topical therapy with administration of 5% 5-fluorouracil cream for three weeks. Six months later, a 3-mm ulcer develops in the right infraorbital region and enlarges to 1 cm over three months.

Which of the following is the most likely diagnosis?

(A) Basal cell carcinoma
(B) Bowen’s disease
(C) Keratoacanthoma
(D) Seborrheic keratosis
(E) Squamous cell carcinoma

A

The correct response is Option E.

This 60-year-old farmer with a 1-cm ulcerated lesion in the right infraorbital region most likely has a squamous cell carcinoma. Actinic keratoses occur in older, fair-skinned persons and are thought to represent the cumulative effects of exposure to ultraviolet light. Treatment of actinic keratoses (typically with administration of 5-fluorouracil) is recommended because of the potential for malignancy; it has been estimated that 20% of patients with actinic keratoses will eventually develop cutaneous squamous cell carcinoma. However, this type of carcinoma rarely metastasizes.

Basal cell carcinomas can arise in this region and appear similar to but are less common than squamous cell carcinoma. The rapid growth (enlarging to 1 cm over a period of three months) would not be consistent with a basal cell carcinoma.

Bowen’s disease, or intraepithelial squamous cell carcinoma, occurs commonly in both sun-exposed and non-sun-exposed areas in older persons. These are typically solitary, sharply defined lesions that have red discoloration and
are covered with plaque. Potential causes of Bowen’s disease include exposure to ultraviolet light or arsenic, viral infections, chronic trauma, and heredity.

Keratoacanthomas grow rapidly over several weeks and regress spontaneously over four to six months if left untreated. Because they are have been shown to progress to invasive or metastatic carcinoma in rare instances, excision for histopathologic confirmation is recommended. They are initially firm, round, flesh-colored or reddish solitary papules, then progress to dome-shaped nodules with a smooth, shiny surface and an umbilicated center with a keratin plug. The face, neck, and dorsal aspect of the arms are affected most commonly; lesions on the trunk are rare.

Seborrheic keratoses are common lesions that occur on the trunk and upper and lower extremities in middle-aged and older persons. They are sharply circumscribed and have a waxy, greasy, or pressed-on appearance with a friable hyperkeratotic surface. Pigmentation varies from mild to deep black. This is not a premalignant condition.

119
Q

A 59-year-old man is undergoing follow-up evaluation one year after excision of a recurrent basal cell carcinoma involving the skin of the cheek. Current examination shows a slight bulge at the site of excision that is firm to palpation and is covered by a well-healed scar.

Which of the following is the most appropriate initial management?

(A) Observation
(B) Antibiotic therapy
(C) Massage therapy
(D) Incision and drainage
(E) Incisional biopsy

A

The correct response is Option E.

The recurrence rate following initial treatment of a basal cell carcinoma smaller than 2 cm is 10%. This rate increases to 25% following treatment of recurrent tumors and can be as high as 82% if tumor excision is incomplete. Recurrence rate is highest for tumors of the periorbital, periauricular, and perinasal regions. Morpheaform basal cell carcinoma, which is characterized by indistinct margins, is also prone to recurrence. Any patient who has had one recurrent tumor should be considered at risk for a second recurrent tumor.

Because of the risk for recurrence, any patient who has had a basal cell carcinoma (and especially those with recurrent carcinomas) should undergo frequent follow-up examinations. A biopsy should be performed if a new abnormality is detected at the site of previous excision; therefore, this patient who has a firm bulge on the cheek at the site of a previously excised basal cell carcinoma should initially undergo incisional biopsy. Any treatment should be deferred until the diagnosis is made. In this patient, a recurrent tumor will most likely be ruled out following histologic examination of the biopsy specimen. A fluid collection may be associated with the adenoid cystic variety of basal cell carcinoma. If recurrent basal cell carcinoma is excluded, the most appropriate management would be needle aspiration of the fluid and open drainage and packing of the wound or placement of a drain. If a recurrent carcinoma is found, treatment would involve surgery, radiation therapy, or a combination of both.

Antibiotics should be administered only to patients with proven infection. Massage therapy is useful for management of contracted scar tissue but is not indicated as initial management of a patient in whom a recurrent tumor may be likely.

120
Q

Which of the following lesions has been shown to result from sun exposure?

(A) Dermatofibroma
(B) Ephelides
(C) Lentigines
(D) Seborrheic keratoses
(E) Xanthelasma

A

The correct response is Option C.

Lentigines are benign pigmented macules that result from increased activity of epidermal melanocytes; these lesions do not fade in the absence of sun exposure. The two primary types of lentigines are simple, which can occur in any area and usually develop during childhood, and solar, also known as senile, which occur in sun-exposed areas such as the face, dorsal aspect of the hands and forearms, and upper trunk.

Dermatofibromas are common benign, fibrous, papular lesions smaller than 1 cm that develop on the trunk and upper or lower extremities in early and middle adulthood. These lesions may be induced by minor trauma and are more common in women than in men. It is important to differentiate them from dermatofibrosarcoma protuberans, which is a malignant tumor. Histologic examination of a biopsy specimen of a dermatofibroma will show localized proliferations of fibrohistiocytes within the dermis.

Ephelides are common pigmented freckles that are not related to sun exposure. They contain a normal quantity of melanocytes, but the amount of melanin within the epidermal cells is increased.

Seborrheic keratoses are benign keratinocytic tumors seen in patients older than age 30 years. They exhibit a classic waxy, greasy, or pressed-on appearance and may have a generalized distribution.

Xanthelasma involves multiple, soft, yellow-orange plaques that occur around the eyes as a result of deposition of lipid-laden macrophages. Approximately 30% of patients with xanthelasma have a disorder of lipid metabolism.

121
Q

A 45-year-old man has had a painful lesion on the left ear for the past six months. The patient says that he typically sleeps on his left side. Physical examination shows a nodule on the antitragus. Histologic examination of a shave biopsy specimen shows no evidence of malignancy.

Which of the following is the most likely diagnosis?

(A) Acrochordon
(B) Chondrodermatitis nodularis helicis
(C) Dermatofibroma
(D) Sebaceous hyperplasia
(E) Xanthelasma

A

The correct response is Option B.

The most likely diagnosis is chondrodermatitis nodularis helicis, also known as chondrodermatitis nodularis chronica helicis, a benign lesion of the ear that usually occurs in men older than age 40 years. This is a painful erythematous nodule that typically involves the helix or antihelix and often occurs on the side that the patient prefers to sleep. Although intralesional injection of corticosteroids may be curative, shave excision or direct excision is usually required to exclude the possibility of cutaneous malignancy.

Acrochordons are simple skin tags that occur more frequently with age.

A dermatofibroma is a common benign, fibrous, papular lesion usually seen on the trunk and upper or lower extremities in early and middle adulthood. This lesion is more common in women than in men.

Patients with sebaceous hyperplasia have small, yellowish lesions that usually occur on or around the face.

Xanthelasma involves multiple soft yellow-orange plaques, typically in the preorbital area, that result from deposition of lipid-laden macrophages. Approximately 30% of affected patients have a disorder of lipid metabolism.

122
Q

In a 70-year-old man with rhinophyma, which of the following is the most appropriate management?

(A) Oral administration of isotretinoin
(B) Topical administration of 5-fluorouracil
(C) Tangential excision and healing by second intention
(D) Staged excision and coverage with a full-thickness skin graft
(E) Excision and coverage with a forehead flap

A

The correct response is Option C.

In this 70-year-old man who has rhinophyma, the most appropriate management is tangential excision of the affected skin, followed by healing by second intention. Rhinophyma, or sebaceous hyperplasia of the nasal skin, is a sequela of acne rosacea that typically occurs in men older than 60 years. It has not been linked to alcohol abuse. The skin lesions are not considered premalignant, and occult basal cell carcinoma is rarely associated. Tangential excision by shaving, dermabrasion, or laser therapy is recommended for management of rhinophyma; the wound will heal by second intention.

Oral retinoids, such as isotretinoin, have not been shown to be effective in improving symptoms associated with rhinophyma. Topical 5-fluorouracil is appropriate for treatment of premalignant tumors and superficial skin malignancies. Skin grafting or flap closure would be unnecessary because the dermal appendages are preserved following tangential excision.

123
Q

=A 65-year-old man undergoes surgical excision of a morpheaform basal cell carcinoma with a diameter of 1.5 cm from the right cheek. Pathologic examination of tissue obtained from the lesion shows one positive margin. Which of the following is the most appropriate management?

(A) Observation
(B) Topical application of 5-fluorouracil
(C) Radiation therapy
(D) Cryosurgery of the involved margin
(E) Repeat excision

A

The correct response is Option E.

Because untreated positive margins have been shown to result in clinical recurrence in 16% to 42% of patients, the involved margins should be immediately re-excised under frozen section or Mohs’ micrographic chemosurgery. Morpheaform basal cell carcinoma is an aggressive subtype with indistinct margins. As a result, it may be difficult for patients to observe changes in the affected area, and rapid growth may result in compromise of vital structures.

Topical application of 5-fluorouracil, radiation therapy, and cryosurgery do not contribute to the treatment of positive margins.

124
Q

A patient with which of the following disorders is a candidate for surgical rejuvenation of the aging face?

(A) Cutis laxa
(B) Ehlers-Danlos syndrome
(C) Elastoderma
(D) Progeria
(E) Werner’s syndrome

A

The correct response is Option A.

Of the disorders listed above, only patients who have cutis laxa should be considered for elective cosmetic surgery. Cutis laxa is a disorder in which patients have a defect of elastic tissues, more specifically a nonfunctioning elastase inhibitor or degenerative elastic fibers, resulting in the appearance of coarsely textured, drooping skin over the entire body. The condition is often diagnosed during the neonatal period or in early childhood; associated conditions include congenital heart disease, emphysema, and pneumothorax. Affected patients are prone to developing aneurysms and hernias. Although the effects of cutis laxa slowly worsen over time, there are no inherent difficulties with wound healing, and patients may consider periodic surgical procedures to correct skin laxity.

Patients with Ehlers-Danlos syndrome, a disorder of collagen cross-linking, have fragile hyperelasticity of the skin, joint hypermobility, and aortic aneurysms. Because wound healing is poor, elective surgical procedures should not be performed. Elastoderma is a disorder of unknown cause characterized by pendulous skin laxity initially involving the trunk and extremities that progresses to encompass the entire body. Elective surgery is contraindicated because wound-healing mechanisms are unknown. Progeria, which is a disorder of childhood, involves laxity and irregular contouring of the skin and is associated with rapid progression and a shortened lifespan. Cutaneous features of Werner’s syndrome, an autosomal recessive disorder, include induration, plaques, variable pigmentation, and facial aging; alopecia, atherosclerosis, cataracts, and short stature are also associated. This disorder also has been linked to poor wound healing, and surgical procedures should not be performed to correct the prematurely aging face.

125
Q

Which of the following skin conditions is associated with a disorder of lipid metabolism?

(A) Acrochordons
(B) Dermatofibroma
(C) Keloid
(D) Sebaceous hyperplasia
(E) Xanthelasma

A

The correct response is Option E.

Approximately 50% of patients with xanthelasma palpebrarum have familial dyslipoproteinemia. Xanthelasma palpebrarum usually presents in the fourth to fifth decade of life and is characterized by multiple soft yellow periorbital plaques that may coalesce. Findings can be unilateral or bilateral. The lesions should be surgically excised or ablated with liquid nitrogen, trichloroacetic acid, or a carbon dioxide laser; however, the rate of recurrence is high.

Acrochordons are simple skin tags that usually develop in patients older than 10 years.

Dermatofibromas are common skin tumors that affect the trunk and upper and lower extremities. These lesions are typically no larger than 2 cm and are more common in women.

A keloid is an abnormal scar that extends beyond the site of the initial injury.

Patients with sebaceous hyperplasia have yellow lesions measuring 2 to 6 mm on the face.

126
Q

Aesthetic surgical procedures can be performed in patients who have which of the following disorders of excess skin laxity?

(A) Cutis laxa
(B) Ehlers-Danlos syndrome
(C) Elastoderma
(D) Hutchinson-Gilford syndrome
(E) Werner’s syndrome

A

The correct response is Option A.

Aesthetic surgical procedures can be performed in patients who have cutis laxa, a genetic disorder with variable expressivity that results from a nonfunctioning elastase inhibitor or premature degeneration of elastin fibers. Affected patients have coarsely textured, drooping skin. Because this condition is not associated with wound healing problems, rhytidectomy may be considered if the patient is otherwise healthy.

Ehlers-Danlos syndrome, or cutis hyperelastica, is an inherited disorder of abnormal molecular collagen cross-linking, and is thought to result in a deficiency of the lysyl oxidase enzyme. It is characterized by hyperextensibility and excessive thinness of the skin and hypermobility of the joints. Cosmetic surgery is contraindicated in patients with Ehlers-Danlos syndrome because wound healing is poor and subcutaneous hemorrhages are common.

Elastoderma is a disorder of unknown cause characterized by pendulous skin laxity initially involving the trunk and extremities that progresses to encompass the entire body. Elective surgery is contraindicated because wound-healing mechanisms are unknown.

Progeria, also known as Hutchinson-Gilford syndrome when seen in adults, is an autosomal recessive disorder of unknown cause and is characterized by growth retardation, craniofacial malformations, cardiac disease, and auricular abnormalities, as well as skin laxity, loss of subcutaneous fat, and findings similar to premature aging. Aesthetic surgery is contraindicated because wound healing is poor; in addition, premature death is associated.

Werner’s syndrome is a rare autosomal recessive disorder of unknown cause. Associated features include premature aging, cataracts, pigmentary abnormalities, and diabetes mellitus. Because these patients have arteriosclerosis and microangiopathy, cosmetic procedures should not be performed.

127
Q

A neonate has a 4 * 4-cm congenital defect of the scalp and underlying skull. The brain is visible beneath a gray membrane. Which of the following is the most appropriate initial management?

(A) Frequent application of silver sulfadiazine ointment
(B) Wet-to-dry dressing changes twice daily
(C) Biopsy of the wound margins
(D) Tissue expansion and coverage with scalp flaps

A

The correct response is Option A.

This neonate has cutis aplasia, or congenital absence of the layers of the skin and scalp that can also expand to include the skull. Conservative management is most appropriate; frequent application of silver sulfadiazine ointment and coverage with occlusive dressings will allow for wound healing. Some surgeons also advocate skin grafting over the exposed areas to prevent wound desiccation.

Use of dry dressings can actually result in desiccation of the dura and rupture of the sagittal sinus, a fatal complication. In patients with cutis aplasia, the wound must be kept moist at all times. Biopsy of the wound margins is not indicated because this patient does not have a malignant or premalignant condition. Tissue expansion is not required for this small wound, which will heal by secondary intention without reconstruction.

128
Q

A 45-year-old man has a painful lesion in the left conchal bowl. A biopsy specimen of the lesion shows chondrodermatitis nodularis helicis. Which of the following is the most appropriate management?

(A) Topical administration of retinoic acid
(B) Administration of acyclovir
(C) Administration of fluorouracil
(D) Administration of interferon
(E) Surgical excision of the lesion

A

The correct response is Option E.

Surgical excision is the most appropriate management of chondrodermatitis nodularis helicis, a painful erythematous nodule found on the helix, antihelix, or antitragus in persons older than 40 years. Although the cause of chondrodermatitis nodularis helicis is unknown, trauma, sun damage, and cold exposure may be mitigating factors. Men are more frequently affected. Because these lesions can become ulcerated, resulting in exposure of underlying cartilage, early surgical excision is recommended. Adjunctive removal of the underlying cartilage lessens the risk for recurrence, which is common at the borders of the resection. Other forms of therapy such as topical retinoic acid, acyclovir, fluorouracil, and interferon have not been proven effective. Intralesional injection of a corticosteroid has been helpful in some patients.

129
Q

In a patient who has a halo nevus, which of the following is the primary indication for surgical excision?

(A) Elimination of circulating antibodies
(B) Premalignant potential
(C) Prevention of leukoderma
(D) Relief of pain
(E) Resemblance to melanoma

A

The correct response is Option E.

Halo nevi, so called because of the distinct “halo” area of depigmentation surrounding the benign nevus, are typically seen on the trunk in teenagers and young adults. Histologic examination will show nevus cells surrounded by a band-like infiltrate of lymphocytes that may completely obliterate the lesion. Halo nevi have no known premalignant potential; however, because melanomas can also develop an irregular, incomplete halo, excisional biopsy should be performed for any halo nevus that becomes enlarged, asymmetric, or ulcerated or displays other characteristics similar to melanoma. The depigmentation around the nevus, known as leukoderma, is typically not painful. Although circulating antibodies to melanoma have been found in patients with halo nevi, this discovery alone is not an indication for nevus excision.

130
Q

A 50-year-old man has a keratoacanthoma on the left arm. He underwent removal of skin tumors on three previous occasions. His brother and father have had similar findings; the father subsequently died of a malignant tumor. Which of the following is the most appropriate diagnostic test?

(A) Bone scan
(B) Colonoscopy
(C) CT scan of the chest
(D) Panoramic radiograph
(E) Plain radiographs of the long bones

A

The correct response is Option B.

This patient has findings consistent with Muir-Torre syndrome, an autosomal dominant disorder that typically has its onset between the ages of 50 and 70 years. Multiple skin malignancies (eg, keratoacanthomas, basal and squamous cell carcinomas, adenocarcinomas) are characteristic. Because malignancies of the colon, kidneys, bladder, ovary, pancreas, and breast are also associated and may even be present prior to the appearance of skin lesions, diagnostic testing (such as colonoscopy) should be performed in any patient with multiple keratoacanthomas, especially in this patient who has a family history of similar findings. The other diagnostic tests listed above would not be effective in detecting malignancies associated with Muir-Torre syndrome.

131
Q

A 54-year-old woman undergoes excisional biopsy of a 6-mm papule from the right alar crease followed by direct closure of the wound. Histologic examination of a biopsy specimen of the lesion shows sclerosing basal cell carcinoma with positive deep margins.

Which of the following is the most appropriate next step in management?

(A) Observation and follow-up examination every two months
(B) Postoperative radiation therapy
(C) Reexcision with 5-mm margins and immediate reconstruction
(D) Reexcision with 1-cm margins and immediate reconstruction
(E) Mohs’ micrographic surgery and immediate reconstruction

A

The correct response is Option E.

This patient has many risk factors for recurrence of her basal cell carcinoma, indicating the need for surgical removal. The lesion is located within the “H” zone of the face (defined as the upper lip, nose, periocular regions, and temporal and preauricular areas) which is associated with an increased risk for local recurrence. In addition, because the sclerosing and morpheaform types of basal cell carcinoma are more prone to recur, pathologic evaluation of the margins is indicated prior to reconstruction, especially in this patient who has positive deep margins. Therefore, Mohs’ micrographic surgery is recommended to excise this high-risk carcinoma and provide tissue for immediate horizontal frozen section analysis. This technique is associated with a high cure rate in patients with recurrent or large tumors, poorly differentiated or aggressive histologic-type lesions, and tumors with positive margins and/or perineural invasion. Standard frozen section analysis may also be considered in this patient.

Although observation has been advocated in the past for the less aggressive variants of basal cell carcinoma (ie, not sclerosing and morpheaform variants), excision is now advocated because the recurrence rate has been shown to be high. Radiation therapy is appropriate only adjuvantly or as a first-line treatment in patients who are not surgical candidates.

132
Q

Which of the following lesions is most likely to contain malignant cells?

(A) Acrochordons
(B) Cutaneous horn
(C) Dermatofibroma
(D) Molluscum contagiosum
(E) Seborrheic keratosis

A

The correct response is Option B.

Cutaneous horns are most likely to contain malignant cells. These hard, cone-shaped cutaneous projections are typically caused by excessive epidermal growth and retention of keratin. Although cutaneous horns are most frequently associated with benign irregularities, approximately 20% are associated with premalignant lesions, and nearly 15% are associated with squamous cell carcinoma. Therefore, in patients with cutaneous horns, shave biopsy should be performed initially to exclude any underlying malignancy.

Acrochordons are simple skin tags not linked to malignancy. Dermatofibromas are benign, localized, fibrous tumors contained within the dermis that can develop following trauma. Molluscum contagiosum are virally induced papules commonly seen in children and young adults and may be associated with sexual transmission. Seborrheic keratoses are benign keratinocytic tumors seen in patients older than 30 years of age. Although these lesions often affect sun-damaged areas, they do not contain malignant cells.

133
Q

In children, Spitz nevi are most frequently found at which of the following sites?

(A) Head and neck
(B) Upper extremities
(C) Trunk
(D) Genitalia
(E) Lower extremities

A

The correct response is Option A.

The Spitz nevus was first described in 1948 when one researcher found that several children who had been previously diagnosed with melanoma were still alive at follow-up examinations as long as 13 years later. As a result, these lesions became known as benign melanomas. Although the diagnosis of Spitz nevus may be difficult to determine both clinically and pathologically, these are most commonly considered benign lesions. They are usually small, often less than 6 cm in diameter, and nodular, and may be multiple. In children, Spitz nevi are most commonly located in the head and neck region and first appear between ages 5 and 10 years. In adults, these lesions are most frequently found on the extremities.

134
Q

Administration of which of the following reverses the potential for development of premalignant lesions?

(A) Oxybenzone
(B) Padimate
(C) Para-aminobenzoic acid
(D) Retinoids
(E) Zinc oxide

A

The correct response is Option D.

Retinoids, which are natural derivatives of vitamin A, have been shown to reverse the formation and growth of actinic keratoses. Because approximately 5% of these lesions will undergo transformation into nonmelanotic malignancies, topical application of retinoids is recommended to reverse cutaneous actinic damage, resulting in a return to normal skin in the affected areas.

Para-aminobenzoic acid, padimate, oxybenzone, and zinc oxide are sunscreen ingredients that limit the effects of ultraviolet-A and/or ultraviolet-B light but do not treat potentially premalignant lesions.

135
Q

Which of the following is most characteristic of an amateur tattoo?

(A) Irregular penetration of the dye in the superficial dermis
(B) Large pigment particles (50 _m)
(C) Organometallic pigments
(D) Penetration of the dye confined to the epidermis
(E) Uniform penetration of the dye in the deep dermis

A

The correct response is Option A.

Amateur tattoos are most frequently characterized by irregular penetration of the ink within the superficial dermis. This type of tattoo is popular because it is inexpensive and easily created using ink and a needle. Blue and black dyes are generally used. Pigment particles within the tattoo are small, typically 5 _m or less.

Professional tattoos are more likely to have brightly colored dyes and large pigment granules (greater than 50 _m). There is a consistently uniform depth of penetration within the deep dermis.

136
Q

A 56-year-old man has a 30-degree flexion contracture of the proximal interphalangeal (PIP) joint of the right ring finger. He has had thickening in the palm at the base of the finger for the past five years. Which of the following structures are most likely involved in the PIP joint contracture?

(A) Central and spiral cords
(B) Lateral cord and knuckle pad
(C) Natatory and retrovascular cords
(D) Retrovascular and lateral cords
(E) Spiral cord and Cleland’s ligament

A

The correct response is Option A.

The central, lateral, and spiral cords each contribute to recurrent contracture of the PIP joint; the little finger is affected most frequently. The central cord develops from fascia between the neurovascular bundles and is continuous proximally with the pretendinous cord. It attaches distally to the tendon sheath over the middle phalanx. The lateral cord is adherent to the skin, while the spiral cord can occur as a continuation of the pretendinous cord or can arise at the musculotendinous junction of the intrinsic muscle. This cord straightens and courses less obliquely over time.

The natatory cord passes across the palm at the level of the web spaces and attaches to each individual flexor tendon sheath. Contracture of this cord can contribute to contracture of the PIP joint. Cleland’s ligaments are fascial structures located dorsal to the neurovascular bundle that help to hold the skin in position during flexion and extension of the finger. These structures are only an occasional cause of PIP joint contracture. The retrovascular cord most frequently causes contractures of the distal interphalangeal joint. This longitudinally oriented fascial cord lies dorsal to the neurovascular bundle and palmar to Cleland’s ligament.

Knuckle pads are a manifestation of joint contracture and not a cause themselves.

137
Q

A 73-year-old farmer with severely sun-damaged skin has 15 scaly lesions smaller than 1 cm on the face. He reports that the lesions have progressively increased in size and number over the past six months. Which of the following is the most appropriate management?

(A) Phenol peeling
(B) Cryosurgery
(C) Topical 5-fluorouracil therapy
(D) Topical tretinoin therapy
(E) Surgical excision

A

The correct response is Option C.

This patient has findings consistent with multiple actinic keratoses, or rough, scaly, discrete lesions of the epidermis that are premalignant precursors of squamous cell carcinoma. If the lesions are few and/or isolated, appropriate management can consist of cryosurgery, curettage, or surgical excision. However, these therapies are impractical in patients who have numerous, diffuse nevi on the face or upper extremities. Therefore, in patients such as this one, topical treatment with 5-fluorouracil for three weeks will effectively remove the keratoses. Any remaining lesions should then be excised surgically due to the patient’s risk for development of invasive carcinoma.

Phenol peeling can also be used to remove keratoses but is an invasive procedure with a longer and more difficult recovery phase. Tretinoin is substantially less effective than 5-fluorouracil and would exert only minimal effects in patients with diffuse keratoses.

138
Q

Which of the following lesions is associated with trigeminal nerve distribution?

(A) Blue nevus
(B) Ephelis
(C) Nevus of Ito
(D) Nevus sebaceus of Jadassohn
(E) Nevus of Ota

A

The correct response is Option E.

The above photograph depicts a nevus of Ota, a bluish gray facial lesion that occurs along the distribution of the ophthalmic and maxillary branches of the trigeminal nerve (V1 and V2) and predominantly involves the periorbital region. The cornea, nasal mucosa, and oral pharynx are rarely involved. Approximately 60% of these lesions are present at birth or develop by age 10 years, 80% are seen in female children, and 5% occur bilaterally. Because malignant transformation is rare, treatment is only indicated for cosmetic reasons.

A blue nevus is a small, well-defined intradermal nevus that rarely displays malignant transformation. Ephelis is a pigmented freckle that contains a normal quantity of melanocytes and an abnormally large quantity of melanin granules. There is no potential for malignancy. The nevus of Ito is similar to the nevus of Ota but instead follows the distribution of the lateral brachial cutaneous and supraclavicular nerves. The nevus sebaceus of Jadassohn is a yellowish orange, slightly elevated plaque seen at or soon after birth. It is a benign epidermal tumor that occurs most frequently on the scalp and face. Approximately 15% of these lesions develop into basal cell carcinoma.

139
Q

Sebaceous carcinoma most frequently affects which of the following anatomic sites?

(A) Eyelid
(B) Finger
(C) Nose
(D) Tongue
(E) Trunk

A

The correct response is Option A.

Sebaceous carcinoma arises from the adnexal epithelium of the sebaceous glands. It can be divided into two subtypes; the ocular subtype, which is most common, typically arises from the meibomian gland of the eyelid. This is a more aggressive carcinoma, frequently resulting in metastasis. In contrast, extraocular lesions are common to hair-bearing areas, and metastases are infrequent.

140
Q

A patient who wishes to undergo cosmetic surgery for correction of skin hyperextensibility has an inherited disorder of collagen polymerization that results in laxity of the joints and poor wound healing. These findings are most consistent with

(A) cutis laxa
(B) Ehlers-Danlos syndrome
(C) progeria
(D) pseudoxanthoma elasticum
(E) Werner’s syndrome

A

The correct response is Option B.

This patient has Ehlers-Danlos syndrome, or cutis hyperelastica, an inherited disorder of abnormal molecular collagen cross-linking characterized by hyperextensibility of the skin and laxity of the joints. Cosmetic surgery is contraindicated in patients with Ehlers-Danlos syndrome because wound healing is poor.

Patients with cutis laxa have coarsely textured, drooping skin resulting from a nonfunctioning elastase inhibitor or premature degeneration of elastin fibers. Because this condition is not associated with wound healing problems, rhytidectomy may be performed.

Progeria is an autosomal recessive disorder characterized by skin laxity, loss of subcutaneous fat, and findings of generalized atherosclerosis similar to premature aging. Aesthetic surgery is contraindicated because wound healing is poor; in addition, the lifespan of patients with progeria is shortened.

The mechanisms of action of pseudoxanthoma elasticum, an inherited disorder of premature skin laxity, include increased degradation of collagen and deposition of calcium within the elastic fibers. Rhytidectomy should be considered because healing is normal.

Werner’s syndrome is a rare autosomal recessive disorder of unknown etiology. Associated features include sclerodermic skin changes and accelerated atherosclerosis, which can limit or prolong the wound-healing process.

141
Q

Patients with erythroplasia of Queyrat have squamous cell carcinoma affecting which of the following sites?

(A) Arm
(B) Ear
(C) Nose
(D) Penis
(E) Scalp

A

The correct response is Option D.

Erythroplasia of Queyrat is an irregular, erythematous patch of full-blown squamous cell carcinoma or carcinoma in situ that affects the penis. It is a feature of the condition known as Bowen’s disease, which is characterized by widespread erythematous, brown, scaly patches. These lesions are composed of squamous cells that can completely replace normal dermis at the affected sites. Because approximately 15% of Bowen’s disease lesions progress to become invasive squamous cell carcinoma, surgical excision of an erythroplasia of Queyrat is recommended.

142
Q

A 59-year-old man has a 1.5-cm lesion over the fourth extensor compartment. A morpheaform basal cell carcinoma was excised at the site of the lesion seven years ago. There is no extensor adherence or adenopathy. Which of the following is the most appropriate management of this lesion?

(A) Observation
(B) Marginal excision
(C) Mohs’ micrographic resection
(D) Wide local excision
(E) Radical resection

A

The correct response is Option C.

This patient has a recurrence of basal cell carcinoma, a common, slow growing, nonmelanotic skin tumor that often goes unnoticed and undiagnosed. Basal cell carcinoma is six to 18 times more common than squamous cell carcinoma; sun exposure is directly associated with the development of this type of tumor. Approximately 3% of all basal cell carcinomas are found on the upper extremities. There can be as many as 26 different histologic subtypes of basal cell carcinoma, including nodular, superficial, ulcerative, micronodular, infiltrative, sclerosing, morpheic, and mixed types. The infiltrative, ulcerative, sclerosing, and morpheaform types are the most aggressive.

Several treatment options for basal cell carcinoma are associated with similar cure rates. Lesions smaller than 6 mm can be managed with curettage and electrodesiccation; radiation therapy, cryotherapy, and topical application of 5-fluorouracil are also effective. For most basal cell carcinomas (excluding sclerosing and morpheaform variants), there is a 95% cure rate when the tumor is resected with a margin of 4 mm or smaller. In contrast, aggressive lesions and recurrent tumors are best treated with wide resection with margins greater than 5 mm or controlled resection using Mohs’ micrographic technique. However, Mohs’ surgery will render more exact margins and is thus preferred over wide resection alone, even if the wide margins are examined by frozen section. Following excision, the wound should be covered with a skin graft, which will prevent dessication of the underlying structures and allow for histologic examination of the lesion without further risk for local tumor spread.

Radical resection is not indicated for locally recurrent morpheaform basal cell carcinoma.